You are on page 1of 308

© 1997, 1999, 2003

Inansfered in anyform or bya ma ‘ Property Changes in ideal Gas,


teiontthepriorpermision of Vapor Pressure,
SOLVED PROBLEMS.
SUPPLEMENTARY PROBLEMS.
metricShay
Total Hydrostatic Force on Curved Sata
Power and Bificlency «..
Bernoulli'sEnergyTheorem...
Energy and HydraulicGrade Lines.
SOLVED PROBLEMS.
SUPPLEMENTARY PROBLEMS...

Values of H for Various Conditions.


Contraction of the Jet...
Orifice under Low Heads
Venturi Meter.
481
AB2
483
483
483
484
485
Velocity Distributionin Pipes 8S
Shearing Stress in Pipes 86
486
Pysjor HeHeatarea re Proportions jons forfor Most 487
Rectangular Section.. 487
Trapezoidal Section. 487
489

SUPPLEMENTARYPROBLEM!
artes
“The cardinal objective ofthis book is to provide reference to)
Engineering students taking-up Fluid Mechanics and Hydraulics.
‘This mayalsoserve as guide toengineering studentswho will
betakingthelicensure examination given by the PRC.
‘The book has9 chapters. Each chapter presents the ms
principles and formulas involved,followed by solved proble
and supplementary problems. Each step in the solution is
carefully explained to ensure that it will be readilysunderst ood.
Some problems are evensolved in several method to give the
reader a choice on thetype of solution he may adopt.
‘Toprovide the reader easy access to the different topics, the
used in my
Most of the materials in this book have beenguided
review classes, The choice of these materia ls was by their
Liquids at 1 atm & 20°C. effectiveness as tested in my classes.
on Gases at 1 atm & 20°C, I wish to thank all myfriends and relativesenwho inspired me
or Pressure, in writing my books and especially to my childr and beloved
wife Imelda whois very supportive to me.
will appreciate anyerrors pointed out and will welcome
any suggestion for further improvement.
°
FLUID MECHANICS HYDRAULICS: eaction offuidsatest orin
‘uid Mechanics ia physical science dealing withth
enginee ring using fuids, Fluid
and devices in
‘motion, and with applicationsedtwomajor areasfdstati which deals with
" ngehanicscan besub dynamics, concerned with fluids incs,motion,
divid
Wis at vest, and fluid the flow of liquids or to low-veloity gas The term
Jiylrdynanics is applied to flows
Auhere the gas canbeconsideredasbe ingesse ntially incompressible,
Tomy mother lluminada, “Hydraulics dealswith theapplicationo f fuid mechanics toengineering devices
mywife Imelda, usually water or of, Hydraulicsde
"involving liquids, thro als withsuch problems as,
flow of fuids or o p e n channel s,thedesignofs toragedams,
‘anid our Children Kim Deunice,
Ken Dainiel,
‘and Karla Denise
‘TYPES OFFLUID
" Fluids aregenerally divided into two categories: ideal uids and real uids,
ealfluids
‘Assumed to have no viscosity (and hence, no resistance to sheat)
‘Incompressible
‘+ Have uniform velocity whenflowing,of fluid
‘+ Nofriction between moving layers
* No eddy currents or turbulence
Realfluids
«Exhibitinfinite viscosities
{Non-uniform velocity distribution when flowing
+ Compressible
+ Experience friction and turbulence in low
We mg=22(975)
We 2145 N
Since the massofanobject is absolute, itsmasswilstil be22 kg.

15
Js the weight of a 45:kg boulder if itis brought to a place where thj
‘The specific gravity of certain dueto gravity is395 m/sper minute?
Ib/fe and KN/m®,and (#) mass
Sine
gno05 BU, Imin
‘min 60sec

pa 1272kg/m?
Specific weight y= pg
#12742 981
‘fan object has a mass of22 Specific weight, y = 125 N/mm?
‘where the acceleration due to
that point?
a a
Gas constant, R =718.87Vkg=°K

eT
‘Airis keptatapressure200kPaabsoluteandatemperatureof30°Cin a5
iture of 4 °C and a pressure of184
IMercontine,
-
Whatisthemassofa?

Pro
[Arc oc 208 Lecrs) aPRstn vile 187 a ‘What istheuniweightoftheligain kN.
Determine thegas constant ofthis ee pa
and a volume of 0.995 liter of liter ata pressureof 1 MPa
elasticity (Es) ofthe liquids; ‘2MPa. The bulk modulus of
ear 7
ange surfacesare25 mmapart and the space between. them is filled
fa liquid ofviscosity u = 0.958 Pa-s. Assuming the velocity gradient tobe a
tine, what force is required to pull a very thinplate of0.37 m? area at a
tan speed of0:3m/f the plate is 8.4 mm from one ofthesurfaces?
u
17658sin15°= 00818<4(03)
U=5614 m/s
0.88 = x [2x(0.125)(03)] (0.125)
<= 2988Pa
2088,
8078570005. te the height to which water will rise in a capillary tube of diameter 3
5019 Pas Use o = 0.0728 N/m and y= 9810 N/m? for water.

‘An 16-kg slab slides down a 15* inclined plane on a 3-mm-thick film of oil with
viscosity 1 = 0.0814 Passec. If the contact area is 03mfind the terminal
velocity of the slab. Neglectair resistance.

Minmatar, Uses = 0914 N/mand = 40


“Solution
Capillary rise, k= = Aces8
“7
_(0518)(os140")
“e793 13.6)(0.002)
‘Terminal velocity is attained when the sum of all forces in the direction of Capillaryrise, h=-0.0059m (be negatvesnindicatescpilary
‘motion is zero, Capillary depression, k=5:9 mm
ae
ir transmitter operates at 2 impulses per second. If the device i
of fresh water (Es = 204 x 10? Pa) and the echo is received:

= (05) = secandtheotal
tance coveredis2,then

i will boil if the atmospheric pressureequals the vapor pressure.

ore water at 80°C will boll at 474 kPa,


HAPTER ONE , ‘CHAPTER TWO
Properties of Fluids u Principles of Hiydrostatice

|
Find the angle the surface tension film leaves the glass for = vertical
immersed in waterifthe diameter is 0.5 ineh and the capillary rise i 008 i
Use © = 0,005 1b/ft hapter 2
ciples ofHydrostati
Problem 1-39
What force is required to lift thin wire ring 6 em in diameter from a wa IRE OR PRESSURE, p
surface at 20°C? (6 of water at 20°C = 0.0728 N/m), Neglectthe weight of Is the force per unit area exerted by a liquid or gas on a body or
ring, With the force acting at right angles to the surface uniformly in all
Ans: 0.0278)

system, pressure is usually measured in pounds per square inch


tional il ry
and in the international metric system (SI),in Newtonsper square
al). The unit atmosphere (atm) is defined as a pressure of 1.05323,
(14696 lb/in’), which, in terms ofthe conventional mercury barometer,
to 760 mm (29.921 in) of mercury. The unit kilopascal (KPa) is
in pressure of0.0102 kg/cm(0145 1b/sq in).

ys LAW
Haw, developed by French mathematician Blaise Pascal, states that the
fon a fluid is equalin all directions and in all parts of the container. In
= 1, as liquid flows into the large container at the bottom, pressure
liquid equally upinto the tubes above the container, The liquid rises
evelin all ofthe tubes, regardless ofthe shape or angle ofthe tube,
eau
of ‘@ Ushaped
consists open
crs used. Ittheother tube withFilledonewith a
othe container and to the atmosphere.
touch as water,oil, of mercury,the difference in thedif iqud surface levels
wo manometer legs indicates the pressure
Conditions. For higher pressure differences, a Bourdon gauge,
iter the French inventor Eugene Bourdon, is used. This consists of a
metal tube with an oval cross section, bentin the shape of a hook, One
the tube is closed, the other open and connected to the measurement

provided depth ea constant faa longer pgym led


of a much ‘narrower:
as the pressure exerted by water at the botto
wth ater hag Pressure is the pressute at any one pointthat on the earth's surface from
teaches a maximum
45 the other examples(et ofheight
Figure
of 15 my its water wil exert the same pressure
2-2). Fluid flow ‘of the air above it. A vacuum
Pos ible Tw create a per
isa space has all matter removed
ire
spa pute 2-2), Hydrostatic force causes water some molecules are always present in the
{inthe siphon to flow up tnd and overover the the edge
ed until the Bucket i em advanced a vacuum systems, a small amount of gas: A
sati
‘suctionno hn
is broken,spon
A siphon is prety
arly us atl rpg tent
BSo e
laren. Even remote regions
‘can also be described as
of
a
outer
region
space
of
have
space where the pressure is Tess
‘the normal atmospheric pressure of760 mm (29.9 in)of mercury.
‘Normal conditions at ea level:

m™|
Pom = 2166 tb/f
=147 psi
‘= 209 inches of mercury (hg)
= 760 mm Hg
=10135kPa
notes
Figure 2 2: tlustration of Pascal's Law te Pressure
i feo point (1 & 2), whose difference in elevation ish, to ie inthe
are osm hasng seonal arena length of
a upon itshustbe in equlbriam.
s satrestall fores acting
Societe oe ale Free igus utce
0008 pe ‘coerstrate» 19abs Seema

oo 200 20 = 101.25 gage


‘pesoge
‘Apressure units in kPa
Figure 2: Relationship between absoe and gage pressures
"Note: Unless otherwise spected inthis book, the term pressresigies gage pressure.

MERCURY BAROMETER
‘smpe way to measurechanges in atmospheric
pressure, At sea760levelthe weght ipofthe aonosphere Figure 2 4: Forces acting on elementary prism
forces mercury hm (29.9) a elated
asstube,Higherelevationsyield lowerreadings, ucuid surface refers to Nquid surface subject to zero gage pressure or wih |
becauseaexerts
thinner the atmosphere islessdense
lesspressure thee, andthe “Sita
onthe meray. pressure nly.

thediference in pres ure Beton any toopins inahomogeneousfiak


totheprec thent weight ofthefai) 0 he era di ce
MANOMETER
‘A manometer is a tube, usually bent in a form of a U, containing a liquid
known specific gravity,the surface of which moves proportionally to changes 6 er ismost advisable)
Pressure. It is used tomeasure pressure from an end point, number in order, the interface of different
‘Types of Manometer points of equal pressure (taking into account that for a
us fluid atrest, the pressure along the same horizontal plane
fequal), Label these points with the same number.
dl from level to level, adding (if going down) or subtracting (if going,
DifferentialType-without an atmospheric surface and capable of measu ire heads as the elevation. decreases or increases, respectively,
only differencesoffressure. " ue regard for the specific gravity ofthe fluids
Piezometer~Thesimplest form ofopen manometer, It isa tube tapped into
‘wallacontainer or conduit for the purpose of measuring pressure.
‘Aluin thecontainer orconduit rises inthis tube to forma freesurface ,
Limitations of Piezometer:
+ Largepressuresinthelighter liquids require long
+ Gas pressures can not be measured because gastubes
can not form a fe

eal
th of liquid of 1 m causes a pressure of 7 kPa, whatis the specific gravity)
liquic?

7 = (981x5) (0)

(8) Open manometer () Dierentat manometer

p= @81x1.03)0125)
p=1263kPa
Problem 2-3 4
Uf the pressure 23 meter below a liquid is 338445 KPa, deter above an oil (s = 0.75) surface in a closedtankis
mass density p , and specific gravity s. mine its unit wel frthegage pressure 2mbelow the surface? |
1
Solution
(@) Unie weighty ote: Pi = 101.325 KPO
Seatsais <7) <7
y= 14.715 KN/m?
(+) Mass density, p
ae
&
p= 14715 10%
91,500 kg/m?
(©) Specific gravity,

Prat
1,500
1,000
s=15

Problem 3-4
HE he pressure ata point in the ocean 60 kPa, whats the presure 27 mates ove thebottom ofthe tankcontain inggu eds 90
below this point?
ae dtphighfm reads 103 kPa. Determine the pce weightof
Solution
‘The difference inpressure between any two points ina
liquid ispps =yh
Peapityh
= 60 + (981%1.03)(27)
P2= 392.82 kPa
‘CHAPTER TWO
Principles of Hydrostaics

“Whathada
would constant specie weight of 122 N/nand were incon
be the eight ofthe atmoyphere ifthe atmospheric pressure
evel) is 102 kPa?
Solution:
Helgi ofaecghere ko7

Height of atmosphere, = = 8,360.66 m

Problem 2-43 (CE Board May 1994) eee


Assuming
| approximatespecific weight of air to be constant at 12 N/m’, what is
heightofMount Banahaw if mercury barometer atthe base of ° ig
ectoritestex
‘mountain reads 654 mm and at the same instant, another barometer atthe top
the mountain reads 480 mm.
Solution

(lao ~ (eh Mer


(©9810 13.6)0654)~ (9810 13.6)048) = 12h
=193453 m
Problem 2-15
‘Convert 760 mum ofmercuryto (a) oil ofsp gr0.82 and(b) water.
* Solution
(@) aa *hancaySa
Sat

Problem 2-16 (CE Board May 1994)


‘A barometer reads 760 mmbig and a pressure gage attached to a tankrea
figure shown,

os = 168.1 kPa abs.

‘A hydraulic press is used to raise an 80-KN cargo truck. If oil of sp.gr.0.82


‘on the piston under a pressureof 10 MPa, whatdiameter of piston is required
Solution
‘Since the pressure under the piston is uniform:
Force = pressure x Area
80000 = (1010) 3D"
D=01m=100mm
Solution
Force on the top:
Em px Area
(6500 = px § (700% 20")
p= 0016904 MPa
P=16904 Pa
bpmrh)
16,904 = 9610
he 1.723m.
Weight = yx Volume
= 9810 $0.029.723)
p= 13622 kPa
Problem 2-21 22-309.6 F = (981 x078)(4.6)
The figure shown shows a setup a vessel containing « plunger ang
‘eylinder. What force F is required with
to balance the weight of the cylinderi
‘weightof the plunger is negligible?

‘press shown is filled with oil with sp. gr. 0.82. Neglecting
ie two pistons, what force Fon the handle is required to suppor
(pressure head) when the tank contain
water = (y + 2phy) em of gasoline
y+ 2a =
y=27.06em

n
ae
(0.075)
(0.025)? 526(wn CintRhe Figuvre,e A) ly=4m, Determine the value of,
Ae the tank sho hi =3m and
[2 Mo= 0}
F(0A25) F(0.005)
(0425) =1.1kN1(0.005)
F= 0.0654
ose 000 2 5
i Px 654N
| i “3
o
F
, »
‘a sa ro] [ewer :
t Sa
BD ofhe lever arm
6 Problem 2 - 23, ‘Summing-up pressure header
i from 1 to in meters of wat
i Pus n(ossy x= 2 hy
| Y Y
rd
BE 0+0.84In-(4-3)=0
y fy= 119m

Sincethe gage reads “FULL” then thereadingis equivalent to 30cm ofgasoline


Bolution
d from
rs hea
‘Sum3-up pressure
110 in mete ofwater:
FL +1q358)+1508)= 22Y
A
o+1475= BS
7
B 14.75 m of water
y
=1475(981)
pa 1h7 kPa
ls empty the water surface is
int A and the mercury ofsP gx.

AtoBinmetersofwater:
Pa 3008) +15-yI3.9) = Pe
7
3 39-6y=
gar
where ps0
y= 0324 m_

Solve for in Figure (2):


umuppressureheadfromAto 2in metersof water:
7 + OS
Pa =2
y-0.15(13.55) a
O+y-20=0-
y= 203m
Principles of Fiydrostaics
Woy-= A Eg.)
mn the funnel is filled with water to B, point
"with the same value as point 2 moving up to 2” 1 will move down to 1! Figure (6):
invup pressure head from 2 nin meters ofwater
Sump pesurehenrom B02 PE + (0.2 sin 0 + y + 0.2)(13.6) - (r+ 02) = a
ea
0+ 080+203+%-271x-203=0
7
Poc o de nn
WS6y-x=8.183-2728in0
om
Ea.)
261-080
x= 0081 m=3.1 cm [id6y-x=136y-3)
New reading, R= 15 +2 Ba aie- a
=15+2@.1)
New reading, R =21.2em

teeat point m in the figure


‘The
22 seatenofwanemer:lo tcc
+has a pressure of 30 kPa. If an open
S
too n e ae ‘determine the deflection of mercury.

fuel gaiaee
sees from 20

Bes7 +y08yo201-yn9= Bo
he +246+3-13.6y=0
y= 0.626 m.

Figure (b)

~ Sum-up pressure head from 2 to m in meters of water:


Pa yag6)-2= Pa
P24 Pn
7 aa
In Figure @: z
‘Summing-up pressure head from 1 to 3 in munof water:
Siridins mercury. 16 12 7 + 1528-Ras6)= 22
Poured into the righthand eg 7 1
Ultimateneighnthewoeps? R=1124mm
In Eq,
Take 2e- 20
x= 11438 mm
Ultimateheightsin each leg:
Right-hand leg, y= hx
=1528+ 11438
Solution» Right-hand leg, hx = 267.18 mm_
Solving forh, (sefigure b): Leftchand leg, hi =R +x
Volume ofwater = =1124+11438
he 15:28m=1528.mm_ Left-handleg, y= 125.62 mm
‘Since thequantity mercurybeforeand after water is poured 2-32
‘remain thesame, then;
1208) =R+4+1204x 1 gage reading of -17:1 kPa,
R+2r=240 > (1) the (a) elevations of
liquids in the open
meter columns E, F, and G
PL +307) + 4(1)- nf.6)
7
+21¢4-16y=0

Deflection of mercury
‘Sum-up pressure head from 1 to 5 in metersofwater;
Be: esa7)r
BL 4- -hass)
+444+4 =P=
ay + 101-16
y=0.614 m

‘manometer attached toa pipe shows shows a deflection of 150mm with


Column & Pen‘Calcuara a nerc bsowrlineothefthe
ryure420atheaoncente ene of he pipe crying
cntpipe.
‘Sum-uppressure hea from 1 toc in meteofwater, late the press
PPe snyory=
PL BeFe
i(07)= Be
at thn(07)=0
hy=25m
Surface elevationn = 15 ~ hy
Surface elevatio = 15-25 =125m
Column F
‘Sum-up pressure head from 1 tofin meters of water;
PL
Te 4 ea 3(07)- = PL
= PL +0.45-0.15(13.6)= e
Y
ay +21-n=0 PL +045-204=0
ig 0357 m 981
Surface elevation = 12+ hy pin 156KPa
‘Surface elevation = 12 + 0.357 = 12.357 m
Tor the configuration he
petit = mercury, k=O
beta ‘Sum-up pressure head from.
i — To in meters of water;
Be ehe
Pose x+h-¥136)-2-
tee15= Be
Ps
BiB -15.01+01036

Bezuppreseurehead tn toin telers of ater,


Ph sxe henassyex-15 Be
A.
Y
where Ft Pe =276m > trom (a
276=15+0554
Sum-up absolute pressure head.
‘fromB to 2in meters ofwater,
Ps a7a6jak= AD + 09x-1651=0
#H -952-h= HH Bai
x=1381m
= 2208 m
Then,x + y=2842m
‘Sum-upabsolute pressure hea fromB to A in meters ofwater, ‘Problem
Fe -a7a36)+07~ Ps the manometer setup shown,
ht ‘4 -952+07 = Ba.
aa
42085 kPa abs

lution
340.68" y+17
x-y=10m > Eq.(1)
Sum-uppressure head from A toB
in meters of water;
Pa 50.6085) + y= 2
7
Pa PB ay.y+057 >F9.Q)
Solution roo
‘Sum-up absolute pressure head from Substitute xy = 1.02 in Eq, (1) to Eq. 2):
1 to 2.in meters of water;
Pt yos)= 2
pre a
402101 2
oar" oar
ysterm
2-39
A differential manometer is
Itached to a pipe as shown, 4s intially 250 pe If
Calculate the pressure
difference between points A
andB.

‘Sum-up pressure head from A to B in meters of water;


7A. ¥09)-01036) + 01008) + y05)= 2
Pa Pe ~91036)-01009)

Figure (2) Figure (6)


In Figure a, sum-up pressure head from A to B in meters of water;
74 -06-025(36)+025+21= Pe
7
Pa FS =165mofwater
7
(CHAPTER TWO
Principles of Hydrostatics
| In Figure b,pa = pa +40
| ‘Sum-up pressure head from A” to B in meters of water;
PH(065-1)-(025+291354 235+) = 227
Pa 14440 .064x-34-2720235+ r= P8
7 Y

‘i Newmercury deflection = 250 + 2x = 250+ 2(162) ‘Sum-up pressure head from A to B in meters of water;
New mercury deflection = 574mm 2s soa) -omss)-02408) +025-03(.012)= Fe
Problem 2-4 Pa. = 10598 mofwate
In the igure shown, determine the differencein pressure between points A and pa pa 981(1.0528)
Pa py" 10.32 kPa

2-42 (CE Board)


“Assuming normal barometric pressure, how deep in the ocean the point
“where an air bubble, upon reaching the surface, has six times its volume than it
‘dt the bottom?
‘Solution
‘Applying Boyle's Law
(assuming isothermal condition) ae
frie peVad - %
r= 1013 +9.81(.03)h
prim 1013 +10.104 i
YnV
p= 1013 +0= 1013
Va= 6
(101.3 + 10.1041) = 1013 (6 V)
10104 k= 101.3(6)-1013
hes 5013 m
H is inflated at sea level, where the atmospheric pressure is 1013
he temperature is 21 °C, to 445 KPa. Assuming the tie does not
that is the gage pressure within the tire on the top of « mountene
the altitude is 6,000 m, atmospheric pressure 1s 47.22 KPaa,
ture is 5 °C, and the

2-47 i ‘
tube shown is filled with oll. Determine the pressure heads at B and C:
At sea level: of water. |
Absolute pressure of air, py = 101.3 + 445
‘Absolute pressure, py = 5463 kPa
Volume of air, ¥; = V
Absolute temperature ofair =21 +273 = 294°K

lire did notexpand,


Absolute temperature volumeofait,
ofair, T:= 5+ 273 =278V;=°K V
PM aVe y
Tees
54630) (4722+ pv
472ES24 9 =516578 “Problem 2-48 é
p= 869.35 KPa or the tank shown in the figure, compute the pressure at points B, C,D,and E
“in kPa, Neglect the unitweight of air, ‘Ans: po 49; pc po = 49: pe™ 21.64

ee
Problem 2-49 2-52 i
A glass Utube open tothe atmosphere alboth ends is shown. ifthe U-tu al tank contains water ata height of 55.mm, asshown. nah
contains oll and water, determine the spelgravity ofthe ol ‘ylindrical tank containing cleaning Aid (sg = 038) at a
‘ Ans: 0.8 ‘po = 13A kPa gage and pe = 1342 kPa gage.Assume the cleanin
‘prevented from moving tothe top ofthe tank. Use unit weightof
9 N/a. (a) Determine the pressure px in KPa, (6) the value off in mm
Ans: (a) 12.88; (6) 10.2; () 101

Problem 2-50
‘A glass 12cm tll filled withwater is inverted. The bottom i open. Whats
Pressure atthe closedend? Barometric pressure is 101.325 kPa,
Ars 10015 KPa
Problem 2-51
In Figure 13, in which fluid will a pressure of700 kPa first be achieved?
Ans: gh
p91 2-53
differential manometershown is measuring the difference in pressure two
Petri fom SE
er
the two pipes.
ene
pies. The indicating liquid is mercury (specific gravity = 13.6, ly is 675
‘01 460 N is exerted on lever AB as shown, ‘The end B is connected to)
|which fits into a cylinder having a diameter of 60 mm, What force Foa
‘he larger piston, ifthe volume between C and Disfilled with water?
‘Ans: 1583 KN
Hydrostatic Force
460
Surfaces
HYDROSTATIC FORCE ON PLANE SURFACES
_ptessure over a plane area is uniform, as in the case of a horizontal surface
in a liquid or a plane surface inside a gas chamber, the total

Problem 2-55
iropen tube open tubes attached toa tankas shown, If water rises toa
‘of800 mm in the tube, what are the pressures pand ps of the air above water fase of an inclined or vertical plane submerged in a liquid, the total
Neglectcapillary effects in the tube. syste can be found bythe following formula:
Ans: pa =3.92 kPa p
(CHAPTER
‘Total Hydrostatic Force on Surfact
OF F (yr):
3 - 1, taking moment of force aboute),5, (the intersection
tion ofthe plane area and theliquid surfac
Fy= [yar
where dF=yysinodA
Peysin® Ay
sin 8 AY %p= [prseinoaa)
ain yp =rsine [yaa
From calculus, few =[s (moment of inertia about $)
AY ye ms

Since yi the unit pressure atthe centroid of the plane area, py, the By transfer formulaofmoment of inertia:
‘may also be expressed as:
Lent AY?
is a¥?
1,+
a AY,

Submergedunderlye ofdiferentquisor ‘the gage pressure atthe liquid


bua noteo,3-3seateropp. Se ‘Problem 3- 15, ih
(CHAPTER THREE
‘Total Hydrostatic Force on Surfaces

Redangie

amar Qg=F 8
‘ArasKer
pets il
2
eR THREE Da 4 (CHAPTER THREE
Hydrostatic Force on Surfaces DRAULICS “Total Hydrostatic Force on Surfaces
HYDROSTATIC FORCE ON CURVED SURFACES
‘@ASE I; FLUID IS ABOVE THE CURVED SURFACE.

“where:
"A= vertical projectionof submerged curve (plane area)
oa pressure at the centroid ofA
ote: The procedure used In slving Fs the same are tetpresented in Page 73.

MT: FLUID BELOW AND ABOVE THE CURVED SURFACE


DAMS
Dame ae structures
oe oes oe block the flow of a river,
Samedame dive th fwof iver war isa peta
stream, or other waterwa
Oe ae as
power.
energy ofmake
ges. Many damn ofharnessinland thewaterwe then aga eiaee
falling waterfrrigation,
fo drinking, and crop
Dams also hold ld waterwater for
fe ponerse
and. pro
aoe

PURPOSE OF A DAM
an are built or the following purposes:
1 Iigaton and drinking wter,
Powersupply (hydrodt)
Navigation
Food control
Mali porpocs 5-3:boat Passin g troigh Canal Lack,wateCanal locks are a series
ee a
of gates designed to
8a
ears ae fom ane ef ant ,
‘Saces open and water flows though
on ether side
the water level is equal ream
the shices; the downst gate opens, andthe Doat continues on atte new

‘OF DAMS: pressure


4. Gravity dams use only the force of gravity to resist watertheir weight
thatis, they hold back the water by the sheer force of
this, gravity dams must
pushing downvvard. To doreservoir consist ofa mass so
hheavy thatthe water ina cannot push the dam downstream
or Hpit over, They are much thickerforcesatthe base than the top—a shape
thatreflects the distribution ofthe of the water against the dam.
‘As water becomes deeper, it exerts more horizontal pressure on the
dam, Gravity dams are relatively thinA near the surface ofthe reservolr,
where the water pressure is light. thick base enables the dam theto
‘Mwithstand the more intense water pressure at the bottom of
‘Figure 3-2: Seton of3 dam used forhyrelectc reservoir,
‘Total Hydrostatic Force on Surfaces
‘A buttress dam consists of a wall, or face, supported by several
puttresses on the downstream side. The vast majority of buttress dams
fare made of concrete that is reinforced -with steel. Buttresses are
typically spaced across the dam site every 6 to 30 m (20 to 100 ft)
depending upon the size and design of the dam. Buttress dams are
sometimes called hollow dams because the buttresses do not form a
solid wall stretching across a river valley.

Figure 3 - 4: Gravity dam


2 An embankment dam is a gravity dam formed out of loose rock, earth,
oF a. combination of these materials. The upstream and downstream
slopes of embankment dams are flatter than those of concrete gravity
dams,In essence,they more closely matchthe natura slope of a pile of

3, Arch dams are concrete or masonry structures that curve upstream into
‘a reservoir, stretching from one wall ofa river canyon to the other. This
design, based on the same principles as the architectural arch and
vault,transfers some water pressure onto the walls ofthe canyon. Arch
dams require a relatively narrow river canyon with solid rock walls
capable of withstanding a significant amount of
‘These dams do not need to be as massive as gravity horizont al thrust.
dams because the
‘canyon wallscarry partofthe pressure exerted by the reservoir.

Figure 3 -5:Arch dam


5, Weight or permanent structures on the dam
4, Hydrostatic Uplift
Uy Va
ey Va
Bi, Horizontal Force
4, Total Hydrostatic Force acting at the vertical projection,
ofthe submerged portionof the dam,
Fayha
2, Wind Pressure
3, Wave Action
44, Floating Bodies
5, Barthquake Load
Solve for the Reaction
‘A, Vertical Reaction, Ry
Ryn th,
Rye Wit Wee Wy We OOS
1B, Horizontal Reaction, Re
Ree dh
ReP
| Momentabout the Toe
‘A. Righting Moment, RM_ (rotation towards the upstream side)
RM = Wixi + Waa Wx + Were
B, Overturning Moment, OM (rotation towards the downstream side)
OM=Py+ Ujz+ Uz
With reference to Figure 3-8, for 3 I Location ‘ of Ry (Z) .
f ‘ade nthe shape oftheup Purposes of llustation, an asumptionw
Consider 1 unit (1m length ofdam (perpendicular tothe sketch)

where:
‘= unit weight ofwater = 981 KN/m: (or 1000 kg/m?)
‘c= nit weight ofconcrete
yea2dy (Usually taken as235 KN/m?)
Use (4toget the stressat point where Ry is nearest In the
‘above, use (+)to get gr and (-) toget qu. A negative stress
vestressan apositivestresindicatesteilstress.
je soil cannot carry any tensilestress the result of Eq. 3~ 14 is invalid ifthe
TihYRNp 8 9/6 ShoalsKappes ale
Apwillposi
beused. i "
‘Total Hydrostatic Force on Surfaces
where:
‘y= unit weight ofthe uid
Ripe dsco
vered by the Greek scientist Archimedes thatstates that “any Vo volumedisplaced. Volume of the body below the liquid surface
ne
f se ina
afi cedis aced uponwn by anupurnrd
pnrd fre buayCynant for)fore) eqneu on, fee prolonsinbuoancy, nti heforces acting and apply conditions ofa q
Tis principle, also known a the law ofhydrovttis, applies to both floating
sbmerged bodies, and to all uid and
SFanslder the body shown in igure 3- 9 immersed ina fluid of unit weigh
The ‘horizontal components of the force acting on the body are ailty,ih
vertical projection of the body opposite sides is the
‘of the body is subjecttoa verticalindown
's equal to the weight of the uid above it, and the lower faceward
is
force which,
subje
upward force equal to the weight of real or imaginary liquld above it’ ct to an -
upward force acting on the body is the buoyantforce The net

If the body of height H has a constant horizontal cross-sectionalarea such as


Wertical cylinders, blocks, etc.
(CHAPTER THREE
‘otal Hydrostatic Force on Surfaces
‘STABILITY OF FLOATING BODIES
body is acted upon by two equal opposing forces. These are, the
ly weight WV (acting aits center of gravity) and its buoyant force BF(acting
‘centerof buoyancy that is located at the center of gravity of the displaced
Nquiay
Whenthese forces are collinear as shown in Figure - 10 (a) it floats in an.
Upright position. However, when the body tilts due to 3wind or wave
‘enter of buoyancy shifts to its new position as shown in Figure 3 - action,
10 (b)
the:
and’
the two forces, which are no longer collinear, produces a couple equal to 1V(),
The body will not overturn if this couple makes the body rotate
Originalposition as shownin Figure 3 - 10 (band will overturn if thetowards its
situation
is as shownin Figure 3- 10 (),
The point ofintersection between the of the body and the line of action of
the buoyantforce is called the metacenter. The distance from the metacente Fgute3- 10 c): Unstable postion
to the centerof gravity (G) of the bodyis called the metacentric height (MG).r (M),It
«an be seenthat a body is stable if M is above G as shown Figure 3.10. Figure 3 ~ 10: Forces ona Noating body
“AH UASTABTE TFs Below as shown in Figure 3 -10 (c). If M coincides with
G,the body is said to be just stable, MOMENT AND OVERTURNING MOMENT

OF A FLOATING BODY:
W “== weight ofthe bod}
buoyant force (always equal to fora floating body)
‘center of gravity of the body
Eean
{Gentroid of the displaced lg
center of buoyancy in the tilted position
‘= volume displaced
‘metacenter, the pointof intersection between the line of action
the buoyant force and the axis of the body
Figure 3 10(2): Upright postion ater of gravity of the wedges (immersion and emersion)
Figure 3 - 10 (): Stable postion orizontal distance between the cg’s of the wedges
‘Total Hydrostaticoes
Force onSurfaces 0S
int due to shifting of BF = momentdue to shifting of wedge
BE@=FO)
Use () ifs above Bo BE=Vo
Use (4) if Gis below Bo Feyo
= MBo sind
Note: Mis avays above
Vo MBosind=y 0s

VALUE OF MB,
‘The stability ofthe body depend on the amount ofthe righting moment which
in turn is dependent on the metacentic height MG. When the body tts,
center of buoyancy shifts to a new position (Bo). This shifting also causes th
| wedge oto shift to a new postion e. The moment due to the shiting of
| buoyant force BF(s) is must equal to momentdueto wedge shift F's).

‘Wedge, volume = v
igure 3-11: Recanguorbody

Consider a body in the shape ofa rectangular parallelepiped length Las shown
in igure3-11;
* Volume of wedge, » = ¥8/2){(8/2) tan 8]L
Volume ofwedge,» $13" tan®
Forsmal values of, 5= $8
“Total Hydrostatic Force on Sunaces
(1B?a
tanox3
Butfor small values of 0, sin @~ tan
p=

Note: This frm canbe apple to any section.

Since the metacentrc height MG is dependent with MBs the stability of a


floating body therefore depends on the
section. It can also be seen that the bodymoment
is more
of inertia of the waterlin
stable in pitchingthan in
ng because Ue HOMENE Feria Th pitchingis greater han that in rolling,
| Vo= BDL here Listhelath perpendiculartothe fue
| moment v= 94B/2)1(/2) tan lL
oo pan
‘The righting or overturning moment on a lating body is: eee aes
rom geometry, = #722752
ee = ed
4B/2)sec8 +(B/2)cos0

on) Er]
(CHAPTER THREE
‘otal Hydrostatic Force on Surfaces
fa pipe of diameter D and
{be subjected to a net pressure p.
sine the tangential stress in the
Jet us cuta section oflength
on are the total pressure F due to
and this i to be
d by T whichis the total stress of the
wall.
Dbutsec?@=1 + tan?o g equilibrium condition;
Stee uno) o4) Feari
(2 nto
B (2,
Fe ph=pDs
- == B
M,ap
B?
ap (FS )
2 T= SrAwe
T=Sr6x)
pDs=2x(Sr(sx 9]

determine the longitudinal stress, let us


‘STRESS ON THIN-WALLED PRESSURE VESSELS ut the cylinder across its length as shown.
[eFn= 0}
‘THIN-WALLED CYLINDRICAL TANK FeT
Fepa
tank or pipe carrying a fluid or gas under a pressure is subjected tensile Fep$Dt
forces, which resist bursting, developed across longitudinal and tan

— OD
TS. Ant
‘Ava 2DE
T=SxDt
pED?=SixDt
(CHAPTER THREE
‘Total Hydrostatic Force on Surfaces
Mi spherical tank of diameter D and thickness # contains gas
“of pte stress atthe wall can be expr under a
essed as:

F= ald)(o) = arb &


“ThelocationofFisatthecentroid ofthe pressure diagram. :
Salcsites teh
submergedtonn
ined er vera ceomr op edge fused
in ud wtheh
‘p= internalenoa
Avca
:D=diameter pressurein they
anthecen
pipe sianya the h etcls
terecangur sufce(nc
ofthepipe
Problem 3-2
A vertical triangular surface of height d and horizontal base width b
‘submerged in a liquid with its vertex atthe liquid surface. Determine the tots
force F acting on one side and its location from the liquidsurface.
Solution
Peyha
haja
Asti iets 4
FeyeGds sind wn
wrrte
yorerla
=e
Using the pressure diagram for this case is quiet complicated. With the
shown, its volume can be computed by integration, Hence, pressure
un seas touse only ifthe area is rectangular,with one sehorizontal,
(Gb(2473)
end ma-4
tical rectangular gate 1.5 m wide and 3 m high is submerged in water with
write ‘edge 2 m below the water surface. Find the total pressure acting on one
yn Rdrd/i2= 34
Using the pressurediagram:
= Volume ofpressure diagram Feyha
Fa}Anse height Riew15+2=35m
Fe 4 (bx wi)= fro
Fis located atthecentro ofthediagram, which s % ofthe alitude F=98185)(0.5)@)]
P= 15451 kN
from the base. :
Problem 3-3
vertical circular gate or radius ris submerged in a liquid with its top edge
:
‘ushed ontheliquidsurface. Determine themagnitudeand locationofthe
forceacting on onesideofthegate. cee
ae
y=15-0214
‘Using the pressure diagram:
= Volume ofpressuredagram 1hA
ro(2%.3)a9
= (5122 2+ 40)
i =3m=7
F= 15754
F=1575081 F = (9810082)2)04115)9)]
P=SA3KN
FrISAst in
1, | 0590)
Ay FESO
€=0167m
spe diae
= 3467 m from the oll surface

3-6 (CE Board May


vertical rectangular plate issu
Locationof F fer such that its top edge is
A= A=
Ar= 4000) =457
AnAv¢ Ar=105y
lay=zay)
TO5yy=61(15) +45y(1)
(worcompleto getthon usingte fom) Fom1hA
Fo= (tex 0:8)4/4)(H(d/2))
Fo=Olteb &

Pega ™ (te * 08)(d/2) + Yo(d/4)


pan = 065 tod
Fav (0.65 Ye d)(0(4/2))
Fy= 0325 7b?
7 (CE Board May 1994)
eal circular gate a tunnel 8 m in diameter has oil (sp. gr. 0.8) on (CE Boord May 1992)
land ir on the otherside, If oll is 12 m above the invert and the air press tank 2 my dll and 8m depwth onasvet
kPa, where will a single supportbe located (above the invertofthe tun ‘of ol (p. gr = 0.8). The air above in kg acting onthe
"to holdthe gate in position? 6/08Keer. Deternine thtlanormal force
Solution

Foa™ toa RA
Fea O81 x 080)8) x § (8)? Fi pw
Fai=3,156 KN,
Par™ 03 kg/ca® 8,000 kg/m?
Fy = 8,000(2x x 2) = 32,000kg,
ya6ti=7m
40! =05m R=paA
a= (1000 «08)8) +8000
16'@ = 10400kg/m
F210400(2x x6)= 124800x kg
zed-e=35m
Fua™pace=40% $8) Solve fore
Far=2001 KN R=whA
124800 = 1000. 08)(2x 6)
‘Thesupportmust be located atpoint Owhere
Faaszero, Since Fa > Fa,must be below Fathe moment due to Fu
iv =Bm
en 2 HOMO?
[2Mo= 0} Ay” Gxxo\td)
Fly) Fu(t=9) ¢=023077 m
G.156)3
1560055-y) = 2011(4-y
-y)=4- yen3-e=277m
y
5493-1568y=4-y F=F,+F:=156800nkg > Total normal force
y=262m
Guaprer Taree 4 CHAPTER THREE
“otal Hydrostatic Force on Surfaces iat Hyonuis Pestonuraees ONE
Fy=hiw thy
(156,800
¥=363m)y = (32, 000n)(7)
> Location+ of(124F from
,800n)(2.77)
the bottom

a l
Using the pressure diagram:
a 2) = 22m pa9siksh(a5)
Peia7i
2m
p= Ae
ay
where 5 =

Liter
6m #5)? a
i006)i
=0 6
n “ase
1
s=05+e= 05+ 7

harshsei
Pressure Diagram
= 8000(8)2x) = 125,000 kg shi
(ose
P=P.+Ps=156800nkg > Total normalforce
[y= Pinas
(1568005) y = 1280005)+ (28,8002)
¥=363m_ > Location of Pfrom the bottom
cularBeaeandth : ota
ubergeeafin gl its toponedge
the l o wieei
fushed
ball
Problem 3-9 Fae on the upper half, ol fce acing
In the ifthe
figure shown, stop B will acting
break force on it reaches 40
AN. Find thecritical water depth
‘The length of the gate
mPerpendicular to the sketchis 1.5

:ate
i
= 1cars
Rete, 05756r
{i long damretains 9 m ofwater
in the figure, Find the total nA
peanforce acting on the dan and 3542/3
focation of the center of pressure i = 467m
from the bottom, A=14()(261)
‘A=1305 m?
F = (9810 x 0.83)(4.167)(1.305)
P= 4427 N
P4427 kN
Solution
Peyha lem 3-13
F=9381(45)(@0)010392)) ‘An inclined, circular gate
F=13,769 KN ‘with water on one side is
6 the figure,

1h(@0(10392)*
B510392)45/sin go)
= 1732m
y= 1400392) -1.732
y=3464m
y= 410392) = 3464m
Solution
Problem 3-42 Payha
The isosceles triangle gate shown in ht =2+05sin 60°
the figure is hinged at A and weighs
11500 N. What isthe total hydrostatic i = 2488
force acting on one sideofthe gate in 9812433)(0?
KiloNewton? P=18746 kN
#0565"
oth wall at B. Compute (a) the total force on the gate due to seawater, ce * asx3972)
feaction at B, and (c) the reaction at hinge A. Neglect the weight ofthe gate, €=015m
x= 18-015
916m
EM,=0)
Fés)-RaQ2)=0
218.25(1.65) =2 Re
Ry=180kN
( BFu=o}
Ru+Fsind-Re=0
Ray 180-2185 sin3269"
Rau = 5894 KN
[er=9)
Ray=P cos =0
Ray™ 218.25 cos 33.69"
Rave 1BL6KN
Ram WRay? + Rae
Ra (1816)? + 68.98)"
Ry= 1909KN,

Ebsco the: magenta


location of the total
drostatic force acting on
2m 41m gate shownin
the figure.

F= 081 x 1.03)(4)105)8.6))
F= 21825 kN
3-16 (CE November 1997)
the magnitude of the force on the inclined gate 1.5 m by 05 m shown
Vthe Figure 001. The tank of water is completely closed and the pressure gage
ye bottom of the tank reads 90,000 N/m. Use 9,800 N/cu. m. for water.

-——,
Figure 001

Papas
Pan 2ihtp
g™ O811.26)8) + @B1)(15) + @810.80)1) +32
Fee IPT
Fa 91.6450 x4)
F=733,16kN
Solving fore aia
Solve for f and Pa~ Peg yh
Peyha $0000 p= 98000265)
723.16" (81x4.26)k (24) = 4030Pa
i =7414m 2164000 (0515)
J = Ki /sin60°=7.414 / sin 60"
‘Pa48,0225N.
7 =8561m
ead. OO?
Ay” Rxaj@5e
¢=0.156m
z=2-e=1844m
‘Therefore, is located 1.844 from the bottom of the gate. 3-17
shown in the figure
Jgate is3msquareand A and rests on a smooth floor at
is hinged at gr.
ilofhavingsp. of082 standstoa heightof1.5m_
We the hinge A. The air above the oil surface is under a pressure of 7 KPa
atmosphere. If the gate weighs 5 KN, determine the vertical force F
3-16 (CE Board) 7
Pins 20 mm in diameter are used for supporting flashboards atthe crestof
dams. Tests show thatthe yield pointofiron to be 310 MPa (extreme
ress). Neglecting the dynamic effect of water on flashboards and:
static conditions, what is the proper spacing, S, ofthe iron pins, 80
the flashboards 600 mm high will yield when water flows 150 mm deep
the top ofthe flashboards.
lution
cs :

Solution
P=pgA i ¢ ; R-04s
Pos Par * tds
Pag=7 +981(082)256)
es
P=2759[G)3)]
P=248.34kN
Paya
24834 = (81:08) G3)
i =343m

Tm=aam
‘Moment capacity of one iron pin (20 mm 2):
(i= Me/t}
en. A i HO 10 Fen (3)
(Bx3)485)
ee M=243473.43 Nemm
x= 1655:m M=026347 EN-m
[EMa=0) ‘Moment caused by F (considering $ m width of flashboard):
Pla) + WU.06)-Fe12)=0 Mim Pry
212F= 24834(1.655) +5(.06) ayia _whereA=065
F=196.37 KN F=981(0.45)106 5]
¢ 116 “Total Hydrostatic
2SForce on Surfaces C
F= 26095 3-20
yn tS ee 120 °C, A in the figure reads 290 kPa absolute. The tank is 2m wide
= Je. Hons? dickstothe figure, Assume atmospheric pressure to be 1 bar. Sp fof
"Ag (065)045) ny = 13.6, Determine thetotal pressure acting onsideCD.
£0067m ©
y=03- 0067 =02%3m
MraExy=M.
261950.23 02407
$~0394m=354mm
Problem 3-19
‘The semi-circular gate shown in
Figure 28 is hinged at B.
Determine the force F required
tohhold the gate in position.

T= -
290 (981 x136070) + @81)h+175
k=22m
‘Total force on side CD:(Note: 1 bar = 100 kPa)
Paya i= 175-100
P= 62.4(6:02)%n(8))
P= 13,019.89 Ibs

Fre ¥em29)@)
Fam A2BM49)2.9)2)
Fs 825kN
FeAth
P= 6675 kN
4g earner mee
“Total Hydrostatic Force on Surfaces
Problem 3 - 21
‘The funnel shown in the figure is
full of water. The volume of the
lupper part is 90 liters and the lower
partis 74 liters. Whatisthe force
tending topush theplug out?
ee
Taney
0.082 m
2=08-e
2=0718m
T=Exz
T= 4897x0718

its base horizontal and is hal-f


m on each edge, hasfiled
cubical box, 15 nder illed with
‘water, The remai of the box i with air under a gage press ure of 82
[Pa One of he verticaltopsides is hinge d at the top and is fee to swin g inward,
o what depth can the of this toentbox be submerged in an open body of fresh
In the figure shown, the gate AB fer withoutallowing any water er?
rolates about an axis throughB. The
width isto12themeters. torque T
Jsfateapplied shaft through
Determine the torque T to keep the
sate closed.
1
G)-F 075)-F:025)=0
Fi=puA
Fy, 82{(15)(15)] = 145KN
Fx = 47.30)075)0.5)
Fra4ldkn

Fy= 91h (1.5)05))


y= 207k
x07 +e Solution
Paya
7 oie P= 9.81(A)in(L5)*- x01)
ap Shea Fe 1541 KN
ay” (005
gn 01875 Location of F:
i cote, 405-400
x= 075+ 91878
7 Hono
0208 m
In Equation (1)
0.1875 5(075)-4.14025) =0
2207(a7 + S272) yp 4+ 0208 = 4.208 m below thews
16.55h + 4138-138375-5.175=0 ‘Problem 3-25
16.55 i = 199.412
T= 842m

the gate just to come up to the


‘vertical position.
dA,
Ye (@S1H)G)C) payy
4.905 12 KN A 2s dy
Fe=981ha.5\1) By squared property of parabola
P= 14715h KN 2 3
[EMo=9}
Ful/3) + WO8)-B05/2)= 0
4.905K (h/3) + 5(06)- 14. 715h (078) =0 eaify:
L€Q5)9-11.04h + 3=0 x-2V73
Solve h by trial and error
= 0.2748 m aE = ry (2 2Yy/3) ay)
ME = 2317?dy
Problem 3-26
In Problem3-25, find h when the force againstthe “stop” is a maximum,
‘Solution
[Mo =0) rn]
Fs eye
F\l/3) + W6) + PCL5)-PAL5/2) #0
4905i?=147154
(i/3) + 5(0.8)+ PLS) P= 231081} 197-02]
(075) =0 Pa 1si3kN
P= 1.09-7:358h +3
Gm327-7358=0 Locat
W=225 Fy [ox
i
.
HIB y= ferns)

cos [yay
y=one[a/n7"| s
= 01608 2/7) [972-07]
3,=214mbelow the ws.
is triangular in cross-section with the upstream face vertical, Water is
withthe top. The dam is 8 m high and 6 m wide atthebase and weighs
fons per cubic meter. The coefficient of friction between the base and the
reyent the dam from sliding, The dation is 08. Determine (a) the mayimam and minimum unit pressure on
cific gravity of concrete is foandation, and the () factors of safety against overturning and against
land the coefficient. of friction
‘between the base of the dam and.
the foundation is 0.4. Use 15 as
the factor of safety against sliding.
Is the dam also safe from Sp. gr. of cone, ane ™ TS
‘overturning ?
Sp. gr of cone, fane= 241000
“SE
Solution
Consider1 m length of dam sider 1 m length ofdam
We=teVe W-n¥
We= 120) (24/600)
Wo= 14a by W=576y —wherey= unit wt ofwater
pay
Fay 2250) Feyha
Fe 10125 = 1x1)
Re= E> 10.1257 F=32)
Ry=W. Re=P=32
Ryo Md by Ry=W=57.6r
My RM= Wi)
=57.6(8)
RM= 23047
OM=PQ8/3)
=3218/3)
OM =8533y
FS" OnRM
= RM-OM
= Welb/2) ay
aT) = 204-837
ey - Sm m< <B/2
= 14.8(2.687)1(2637/2) =33>1 (Safe)
FS, 10:125(1.5) e=B/2-%
e=3-2519 =0481 m<B/6
pea co)
Using (4 gr-13947 KPa sollpressureatthe toe
Using (: qu*-4888 KPa, > prossureat the heel
2857.67)

Problem 3-30 (CE Board May 1992)


A gravity dam of trapezoidal cross-sectionwith one face vertical and
‘bases Zan igh and hasahickness oFmathe top. Waterupsteam stands egtecting trycrostatt-uptit
mbelow thecrestofthedam. The specificgravity of masonry is 24 |, Consider 1 m length ofdar
A. Neglectinghydrostaticuplift: |. Forces
1. “Find the base width Bofthe dam so that the resultant force will ut Wim ne Vas (rx 2.4)104)22)0)1
extremeedgeofthemiddle third nearthe to. Wi=21127
2 Compute the factors ofsafetyagainstsliding and overturning:
Use u=05. Wa (rx 24) (B-4)(20)(0)]
B, Considering uplift pressure 10. vary uniformly ffom full hydrostd We= 24By-967
pressure atthe heel0zer0 atthetoe: FeyhA=7 (102)
1. Find the base width B of dm #0 that the resultant force will act F=2000
theextremity ofthemiddlethirdnear the toe.
2 Compute te mastapine compre swe st IIL Reaction
R=th=P
“against thebaseofthe Ry 2007
RyashWiee
= 211.2y + 24By- 967
Ry= 24By + 1152
Be 11475 m
Factors of Safety:
Factor ofsafety against siding:
uk,
-- (05124011175) + 11823)
2007
75, = 0.9585,
Factor of safety against overturning
RM
OM
= 16(11.175)? y+ 832(01175)y 165.4
1585357
P27
IV, Momentabout the toe Considering hydrostatic uplift
RM= Wi -2)+ Wal ZB-4)) ‘Uplift force, U= Ys 20))(8)(1) = 10Bw
= 211.2y(B - 2) + (24By- 967)(3 (B-4)] Rye Wit Wy
= 211.28 4224+ 168% -128By + 2567, = 24By + 115.27 - 10By
RM = 16B% +63.28 - 1664 Ry=14By + 115.27
(OM = F(20/3) RM = W(B-2)+ Wil (8-4)
= 2007(20/3) RM = 168% + 83,2By - 166.47
OM = 1333-337 OM = F(20/3) + U2B/3)
V._ Location of R: = 2007(20/9) + 10By (28/3)
OM » 6,67B% + 1333.37
Ry¥=RM~OM ;
Ryz = RM-OM
(14By + 115.2)(8/3) = T6BYy + 83.2By-166y - (6.678% + 1333.33)
Since the resultant force will passthrough the extreme edge of 46682 + 44.88 - 149973 = 0
the middle thirds near the toe, ¥ = B/3. Then,
pw 8 Ve) 44.65(-189979)
(24By +38-487
8B+ 115.2)8/3)=
=168% + 16B% + 83.28y- 16647-1338.
63.28)14997737 = 2466)
85+ 4488 149973 =0 B=13766m
¥=13:766/3 459m
= B/2- =22913m
ye $@-2m
Ry = 14(13.766)981) + 115.2981) Wars
Ry = 3020734N =23512(6)())
WW) 376 kN.
= 302073f;,, 622943) Win Va
4° ~"33766 Pe 13.766 = 2351/42)6)())
gr -498.87 kPa Wi= 188 kN.
qu OKRA,
wid 42) =3,
29 (2/3)(2) =1:333 m
‘Problem 3-31 (CE Board May 2002)
‘The section of a concrete gravity
pure
<dopth ofwater attheupstream side
ism, Neglect hydrostatic uplift
‘and use unitweightofconcrete
‘equal to 235 kN/m, Coefficient of
friction between the base ofthe
‘dam and the foundation is06.
Determine the following: (a)factor
ofsafety againstsiding, (b)the = 376(8) + 188.338)
factor of safety against RM = 1578604 KN-m
overturning, and (€)the OM=Fxy
‘overturning momentacting agaist = 176580)
the damin kN-m. OM™35216KN-m > overturningmoment
2M
ae OM
1378608 =3908
wy 353.16
‘THREE = ‘Total Hydrostatic Force on
Hydrostatic Force on Surfaces
3-32 (CE Board November 2001) 333 (CE Board May 1986)
ection ofa gravity dam is Jsecton of the masonry dam is as shown. ‘The specific weightof water
jahown in the figure. Assume KN/ox? and thatof concrete is 2354 KN/m?. Assuming uplift pressure
PRjdrostatic uplift to vary Tineatly from
tnlformly from fll ydostaic im hydrostatic
Uplift rom the heel tozero at at the fel fo zero at
thetoe. Determine the total Raia. HHocation of the drain,
reaction per unit length atthe 6” 2 inethe (a) locationof
baoeofthe das. Use pg F resultant force(factor
concrete = 24. gant ding If ‘|
offetion is 075,
Solution actor ofsafety agaist
(a thestress at
heel andat thetoe, and ()
unit horizontal shearing
Consider 1foolength ofdame at the base,
Ren Fayha
= 62406031)
Ra 1i2220ibs ie
Ry= Wit Wat W5-U »
Wien
= (24x2928oa
Wi= 224640bs
Wie Va
= (24x29 2-2 ay
Wa44,925 Ibs
Woe Vor (624 x24) 424)40)0) ;
Wo= 718648 bs
Wi te a= (624) 410060) q
Wy = 56,160 Ibs
Usy— Yum (624) 460,70)
U> 131,040Ibs
Ry = 224,640 + 44,928 + 71,884.8 + 56160 - 131,040 k-—— 32m —9
Ry= 2665726le | 30
R= [(412,520) + (2665728) = 289,269 Ibs per foot
R M , 68390012
© FS on “ar778
F5,= 1.81
Wy= 23.54 [45.2)62)()] = 3,183 KN (d) Foundationpressure:

ndeail
We 2354 [7)62)0)= 8569 KN
coilmanana
Siege B
Vic 9810048\60(2)] = 1.2263KN a*H(28)
U=Bee14490.5)28.2)1) =5,690kN BR 669)
mana},
Eras981051800) 4" 383 Shs 382
Stress at the toe, (use “+")
il, Reaction
Ry = F = 12263 KN bist
Ryn Wit Wet Wt WU. Stress the
Tis sesons 1263-5690 qeunieae
T= TUTSEN
‘(¢) Unit horizontalshearingstress, Ss
IV. Moment Se fe
+ WW4(36.53)
RM = WiGG473) + Wi(295) + W(17.23)3(17 ew
83(8473) + 8569(295) + 1591 39) + 1226.36.59) ae es :
RM ==31683,900.12kN-m
uG047) 7)
OM = F80/3)060+/3)+
=12263 549004 e
E e
EMSSee
Location of Ry
Bz =RM-OM
23,2013 ¥ = 683,90012-377,758
¥=132m
|(@) The resultantforceis132m from the toe.
a,
eel
Race
Hydrostatic Force on Surfaces. (CHAPTER THREE
“Total Hydrostatic Force onSurfaces.
nother way of solving ¥:
Tieyha jee the unit pressure is always normal
Fy == 9816142)
9924kN he surface and a normal to the circle
through its center, then the total
yrlte F should also pass through the
mer of the circle O, therefore the

4Q) /12
are
€=0067m
=1+0067
= 1067 m t
“Therefore; Fisting1.067 m blow B
Fv Weightanco
Monsists of a cylindrical
jurface of which AB is the
Ar=%xQ)=3.14 mt hse supported by a
Aructural frame hinged at
A=s+oid , The length of the gate is
A=1144 mn? 10 m Compute the
Vasco = s(11.14) Magnitude and location of
Vanco = 44.56 m0? the horizontal and. vertical
Fy=9.81(44.56)
Fy=437.413 kN
Location fF
ae
ete
=a,3a” s@)
Be
= 089m
‘1114 ¥ = 8(1) +3.140.819)
¥=0957m
‘Therefore; Fy is acting 0.957 to the right of A
be
= 981(4.33)1108.66)}_
Fy) = 3679 KN
yo $666) =2887m
“Therefore:Fisating887m above O
Fvny Vase
Vasc = Vaonc= Vion
= 5 (6.66) 10-4109 [60° ae] x10
Vasc 12590 Fy= 9.813.261)
Fyv=9.81(12590) Py = 31.98 KN,
Fy=1235 KN Fe PPA?
‘Moment about O due to Fy and Fy = 0 = (07658)? + (61.98)?
FvQ)= Fay)
1235 x= 36792877)
m
‘Therefore; Fv is acting 857 m to from 0

Brobiem 3 - 36 (CE May 1999)


Calculate the magnitude of the
resultant force per meter length due to
water acting on the radial tainter gate
shown in the figure.

Fam pg
Fe ™ (624 x 25)(8 x 1) = 780Ibs
Fv=1 Vasc
Fy= 624 @)M0)] = 1225 tbs
P= (Er 6
B= (780)? +(1225)? = 1452 tbs
F
(CHAPTERTHREE Srtenvi
“otal Hydrostatic Force on Surfaces _ a ‘Total Hydrostatic Force on Surfaces
3-38 '3- 39 (CE Board November 1993)
Determine the magnitude of the the figure shown, the 1.20 m
horizontal and vertical cylinder, 1.20 m longis acted
‘components of the total force per by water on the left and oil
‘meter length acting on the three: sp. gr. of 0.80 on the right,
quarter cylinder gateshown. line the components of the
mn at B if the cylinder weighs
2 kN

9.81(1.2)(1.2%1.2) oem
Fin = 1695 KN
ean En=vi
Fr = 981p2x (06)%.2))
Fy = 6.657 EN.
FoxyA= (81080612412)
iy

Fn= 6781N
Fram Wa (981 x08)[x (0.6)41.2))
Funyha
F2=532kN
= 981812). Fx O}
Fu=58.86KN Fin-Fra- Raw = 0
Ray = 16.95 -678
Ray = 10.17 KN
[eR =0)
Ruy Fut Fin We0
Ryy= 19.62 6.657 -532
Ray 7.04 KN|
y= yWol
= 981[42)(1) + 075[x(27(1))
Fv= 170.98 KN
(CHAPTER THREE.
‘Total Hydrostatic Force on Surfaces
Fy = (81 082)0.00628)
‘An inverted conical plug 400 mm diameter and 00 mm long closes a 200 Fy= 0.0505 KN
diameter circular hole afthe bottom of a tank containing 600-mm of oil ha heh
eqegrgietey aie eee
f * 0.0635 KN = 63.5 N downward
Solution
cerry
‘ny diameter horizontalcylinder 2 m long plugs a 1m by 2m rectangular hole
the bottom of a tank. With what force is the cylinder pressed against the
‘of the tank due to the 4-m depth of water?

Iy=2x (1 c0s304)
y= 1.732m
I= 4—hs
y= 2.268 m

Rem
be | ote Vin Ad
‘Area, Ay = Area ofrectangle DEPG - Ay
‘reno segment = 22L9 «1 sin60"
Bem ‘Area of segment, Ax = 0.09059 m?
Brrae
81 082)n037045) ‘Area At = 1.268) - 0.09059
P. ‘Area, Ay 2.1774 m?
Rey: ‘Vi = 2.174(2) = 4.355
Vi= Viren= Vom F,= 9.814.955)
wDfua?e200."|-c0ye09 Fr=4272KN
RahoWr
‘Ve 0.00628 me eae
AenotspentAx BOLE ya 0 Tynyha
P= 98161242890)
‘Area ofsegment,Az = 0614 m= Fip= 25456 kN
Va 0.6142) = 1.298 m? e
f= = 981(1228) i .
Fi=Fy= 1205KN Vat (Amun Ano)
Net force = Fy -F;~F, Vass = [eneay? + £9934(429)]c1)
[Net force = 42.72 - 1205 - 12.05, Vosnna = 40.1
Net force = 18.62 kN Fe=981(401)
Fy = 393.38 KN
Problem 3-42
In the figure shown, determine
the horizontal and vertical gate shown is a
components of the total force circle 25 m wide.
‘acting on the cylinder per m of the force F just
its length,

Funyha
Fy = 981(1)@5 2)
Fy=1 Vac
Fy=981[@2)-02520)125)
Fy=2105KN
pve for zand x: Forces duetool:
Since the surface is circular, EMo = 0 due to Fy and Fy Fo paw A
Fy@)=Fu@/3) Fyo = (@.81 x 080)(7 - 1.273)x ¥an(3)?
21.05(2) = 4905/3) Fyo = 6354 KN
2=155m
xe2-2=2-155 Fone
x= 045m Vj = Volume of imaginary oll above the surface
{2M = 0] = Volume of halfeylinder ~ Volume of % sphere
FuQ2/3) + Fe(@)-FQ)=0 Vo= Yan77) - 44 x0)?
2F = 49.05(2/3) + 21.05(045) V, = 70,686 m?*
F=21.09 kN Fyo= (981 «0:80)(70.686)
Fyo™ 554,74 kN
Problem 3-44 Forces due to water
the cylindrical tank shown has a Fray Page A
hemispherical end cap. Compute Figy = (@.81 « 08)(7) + 981(1.273)] x Vex?
the horizontal and vertical 4" Figy = 953.19 KN,
‘components of the total force due ~}
to oll and water acting on the Fiwy= Weight of real and imaginary oll above the sur
TRamTisphere “F WeIgNC OTFEAT WATT MOVE TNE SUM
Frys 81 « 0.8)x Yon(Q)X7) + 981 = ¥$ =(3)>
Foy = 1,054001
Total horizontal force, Fy, = Fig * Fin
‘otal horizontal force, Fi= 6354 + 953.19.
‘Total horizontal force, Fy = 1,588.59 kN >
‘Total vertical force, Fy = Fuy~ Fvo
‘Total vertical force, Fy = 1,054.01 ~ 554.74
Total vertical force, Fy = 499.27 kN
Trnatnerway to solve for te total verial force, Fe
Fy = weight offluid within the hemisphere
Fran Vet teVe
Fy= @Slx08)[}«4x @))] + 981}<$xG))]
y= 499.27 KN
Pressurized water fils the tank shown in the figure. Compute the ine the force required to open the quarterclinder gate shown, The
“hydrostatic force acting on the hemispherical surface. ght ofthe gates 50 KN acting1.2 m tothe right ofO.

Solution
Convert 100kPa tots ao
aa
head,lg D ee
t= the gate has circular surface,
Y total water pressure passes
hg= 2. gh point O whichis alo the
3a cation of the hinge, therefore the
hq = 10194 ent due to water pressure about
= 10194-5
= 519m t= 0]
F(25)
F=24kN= 5012)+ Fx)
F= Weight of imaginary water above the hemisphericalsurface
Pte Ve
Vi = Volume ofcylinder + Volume ofhemisphere hemispherical dome shownis filled with oll (s = 09) and is atached tothe
Vom xQ76.194) + 1 $202? by eight diametrically opposed bolts, What forcein each bots required to
Vo= 82.025 m* ithe dome down, if the dome weighs 50 kN?
P= 981662025)
P6017kN
Fou (@81«038)(x(0.805)%5)
~ $ (080576)
Fu = 91 KN
Fa paw
Fu 2014 (161)]
Fay =40.72 KN
Solution
Fv y Vinaysateane
Fy (9.81x0.9)fn(2)(8) -3x(2))
Fv=739.66kN

Pru 739.6650 8
sin the steel
Frau ™ 86.2 kN

Problem 3-48 «g,= 2816000) = 6131.25 kPa


Determine the force F required 1 301)
to hold the cone shown. 5i=613 MPa
Neglect the weight ofthe cone.
3-50
termine the required thickness of a 450 mam diameter steel pipe to carry a
ximum pressure of 5500 kPa if the allowable working stress of steel is 124

Ey
ee
124 1000 = 9450)
2
£9.98mm say 10 mm
‘Deormin th sires atthe wall of 200mm diameter pipe, 10 man thickund FON ae
presi
pressure lepth of of 20 m in in salt salt wat water.
of 150 m ofwater and subr ymerged to a depth = 210510900)
54.996(6000)
wee {$= 200.23 mm say 200 mm
2 Pm “Pale a4
9-4150)kPa- 9.81
981( (1.093)(20)
Jyp=126 = 1.26 MPa ‘thin-walled hallow sphere 3.5 m in diameter holds helium. gas eat 1700 kPa,
12690200) «>go aap ine the minimum wall thickness ofthe sphere if its allowabl stress is 60
2410) a

Problem 3-52 Wall stress, = 2


has a 6 mm walthic
‘A 100-mmiDMPa,steelwhatpipemaxima kness. For an allowabl e te conn = 1700(85x 1000)
‘tres of8 m pressure can the pipe withstan d? rn
=:

ee ee Its sides
4 © é in position means oftwo steel hoops, one atthe top and the otherat
p= 96 MPa 9,600 kPs bottom. If the tank is filled with water to a depth of 2.1 m, determine the
ile stress in each hoop.

‘A wooden storage vat is 6 m in diameter and is filled with 7 ofol, s =


‘The wood staves are bound by flat steel bands, 50 mm wide by 6 mm
whose allowable tensile stress is 110 MPa, What isthe required spacing 0
bands near the bottom of the vat, neglectingany intialstress?
saaion
sear stioo = AL
“Allowable tensile stress of hoops, $;= 110 MPa
(Gross-sectional area ofhoops, Ax = 50(6) = 300 mm?
Pipe diameter, D = 6 m= 6000 mm.
‘Maximum pressure the tank (at bottom), p = yailh
" GHAPTER THREE (CHAPTER THREE
‘Total Hydrostatic Force on Surfaces “oval Hydrostatic ForesonSurfaces 155,
[2M 0)
210) = F23) ‘A cylindrical container 8 m high and 3 m in diameter is reinforced with two
T= 0.3833F > 8q.(1) ops 1 meter from each end. When it is filled with water, whatisthe tension.
Feyha
Fa 9813 )1Q)21)]
F=43.26 kN
Inq.)
Ty» 03839(48.26)
Ty= 1658 KN (tension in thebottomhoop)
(2Fi=0)
20+ 20=F
20 =F-2:
27= 4326-21658)
T,=5.05 KN (tension in the top hoop)
ret A
jh{ein
etlpotion
elindil tankopen a hep fled wit ig. ode
by means of two steel hoops, one atthe top andthe other a
F=9816/2)60))
tenon! Determine the rato ofthe stress in the upperhoop to that in theIo Pee
‘hoop: 21,(6) = F (13/3)
Tr= 13F/36
Solution T= 13(041.76)/36
Ratio= Ti / Tr Ty 34008 kN
Pen [Macy =O
216) = F/3)
a(t) F(2h/3) Ti" 5F/36
Ta=F/3 T= 5(041.76)/36 %
T= 1308 kN
[EMboioo = 0]
2TH) = F/3)
T=F/6 plem 3-58 (CE Board Noveriber 1982)
fe Tolindsial tank with is axe vertical is 1 meter in diameter and 6 m high. Itis
ratio= £18 ~o5 Hid together by two steel hoops, one at the top and the ater atthe bottom.
liquids A, B, and C having sp. gr. of 1.0, 20, and 30, respectively fill this
tank, each having a depth of 1.20 m. On the surface of A there is atmospheric
sessure. Find the tensile stress in each hoop if each has a cross-sectional area of
th
Stress, Ss= Ty, 35316
Az 1250
‘Stress, S:"2825MPa > stress in bottom hoop.
[eFu= 0}
2h+2%= Fi + ht i+ hits
27 = 0.72y + 1.4dy + 1.Ady + 432y + 2167-206)
Tye tady
T= 14409810)
y= 141264
Stress, 1 Ty 141264
Ay 1250
Stress, 5;=11.3MPa ‘> stress in top hoop

3 but of 0.98mt cro-ecton in the inverted postion, allowing lls open


P= mt yoo to rest on the bottom of the bigger tank. Determine the maximum tension
Pym 12y + (992)0.2) =26r st vertical millimeters on the sides of the bigger tank. Neglect the thickness of
pes pst yohc the metal forming the inner tank and assume normal barometric pressure,
Pe 367 + (93)(1.2) =7.2y
4701290), 186 0
¥4(1.2910.20) = 0727
Fo= px(.2)Q)
Re 1.2y(1.2)0)= Ly
B= Alps P20)
Fy= YQ.6y- 1.2y(1.2)(1) = 44
Fe, a(l-2))
Fu=3.6y(1.2)(1) =432)
Be 4p0-200)
Fe=W72y-3.6)(12)0)= 2160
[Mop ~ 0}
360M) = (08) + (18) + BQ) + AG)+ G2)
7.2T= 0.72408) + 1441(.8) + 1-A4y()+ 4320) + 2160.2)
he 3.6
f= 3.6(9810) = 35516 N
CHAPTERTHREE
‘Total Hydrostatic Force on Surfaces _1E
3-60 (CE Board November 1977)
taeof water 0.86 - y+) 0880 3 1g having specific gravity of092 i floating on salt water of sp.

Bs
© 1.86b + 0.93h = 2831 Volume of ice above the water surface is 1000 cu, m., what is

ze
2b h= 3084 of the ice?
b-1522-05% > 54.)
Irvin pe Val Let V= total volume of ice
‘pi 101.325 kPa (atmospheric pressure) Vo volume displaced
Vi 093.05) = 2.8365 mé Vo = V- 1000
po= 101.325 + 981k
Vs= 0.99.05 -) Wee too V=(081%0.92)(V)
Wee = 9.0252 V
101.325(2.8365) = (101.325 + 9.811)[093(8.05 -b)]
309.04 = 309,04 - 101:325b + 29.921 981bh BE~ Yewwaee Vo
2992 ~101.325b-981bh=0 BE = @81-103)(V- 1000)
BF = 10.108 - 1000)
29,90 -101.325(1522 -0.5H) -981(1.522 -0.5i)h=0 [R= 0) |
29,92h--154.22.+ 50,66-14.99h + 4.90512» 0 Wee BF |
4.905Ie + 65.65h - 154.22 = 0 Y= 10.1043(V - 1000)
pu SSE EP ETHAN) yg 1.0791 = 101043
en V= 9,364 cu, m.
=05027m
H=b+h=252m ere solid body fleating on a homogeneous liquid:
For homogeneous
“The maximum tensile stress occurs atthe bottomof thetank. V2 Vay, = TO,st
p=yH=9812512)
eee Ta
p=24.937kPa = 0.024997 MPa then; Zi
Vast8
ioe ose=0
Vesaotca
yonisem
D-15%0"1599mn
‘3-61 (CE Board May 2003, Nov 2002, May 2000, Nov 1992)
27 = 0.024997(1539)(1) block ofwood 0.60 m x 0.60 m x hi meters in dimension was thrown into the
T=192N yerand lots with 0.18 m projecting above the water surface. Thesame lock
thrown into a container of a liquid having a specific gravity oF 0.90 and it
pnts with 0.4 m projecting above thesurface. Determine the following:
(a) the valueoth, ; J
(0the speci gravity ofthe block and ‘
(6) theweight ofthe block.
In Water:
Draft= Sp ss
-ou
038= Set jobiem 3 - 63 (CE Board May 1993)
Swood ht = = 0.18 > Eq.(1) meer es 1) ‘ody having a sp. gr. of 07 floats on liquid of sp. gr 08. The volume of the
ly above the liquid surface is what percent of its total volume?
{In another liquid tion
fos Vom Steir Viniy
s, y Sigua
04 = Sood hos Vo $F Vis =0875Viay
ea
Scoot t= 09H = 0.126 > Eq. (2) wil
eer iq (S » 0.9) Since the volume ofthe body displaced (below the uid surface is 0875 or
15% of its total volume,then the volume of the body above the liquid surface
rat
t-018= 09 - 0.126
H=054m > beightofthe block
Substituteto Eq): sm 3 = 64 (CE November 1997)
Soon(0.54) = 0.54 ~ 0.18 block of wood 0.20 m thick is floating in sea water. ‘The specific gravity of
| Stout 0.667 > Specific gravity of wood oc i 0.65 while that of seawateris 03. Find the minimum area of « block
*h will support a man weighing 80 kg,
| Weight
ight ofblock == oes(81 Va
© RERREELGner-ooom »
06 7)(06 x 06)(054)

Problem 3- 62
stone weighs 460 N in air. When submerged in water, it weighs 300 N.
the volume and specific gravity of the stone.
oben, (eFv=0)
‘Weightoftone= 460 N BE = Wan + Went
‘Weight fstonein water =300 N Yes Von = Wa Yeas Vest
Buoyant force, BF=460-300 = 160N (1000» 1.03) Veons = 80 + (1000 0.5) Vanes
BF= Yee Vice Vii = 02105 m= Area x02
160 = 9810 (Vaen) ‘Atea=105 square meter
Vanoe = 0.0163 eum.
‘THREE i CHAPTER
“Total Hydrostatic Force on Surfaces ai ‘Total Hydrostatic Force on Surfaces
3-65 (CE November 1997) em 3- 67
| A aube of wood (6g, = 0:60) has 9-in sides, Compute the magnitude uniform block of steel (¢ = 7:85)
irection ofthe force required to hold the wood completely submerged in wat float at a mercury-water
werface_as shown in the figure,
Solution What isthe ratio of the distances
Weight of wood = (62:4 x 0.60)()? = 15.795 Ibs ‘snd for this condition?
Buoyant force whencompletely submerged in water:
BE =624(g)° =26325 Ibs
Required force = 23.325 15.795
Required force = 10.53 tbs downward

Problem 3 - 66 (CE Nov 2000) ae


‘The block shown in the figure
weighs 35,000 Ibs. Find the
value of h.

ba vere
A be the horizontal cross-sectional
1 ofthe block.
BE + BE, = W
Ye Vow Ye Vow = 1. V
9.81(A x a) + (981 « 13.56)(A x b) = 0.81 «7.85)1A(a +B]
a+ 1356b= 7.850 +7856
571b= 6852
a/b= 0894

BE, Ya Vo 3-65 (CE May 1998)


BF, (62408)(12123) a Sk steel plate is attached to one end of a 0.1 m x 03 m x 1.20 m wooden
BF, =21,565.44 Ibs le, what is the length of the pole above water? Use sg. of wood of 0.50.
21,565.44 + BF; = 35,000
leglect buoyantforceon stel
BF, 1343456 bs
TasiSe=2 290245
1343456
ho 1455
3-70
Neglecting the buoyant force on steel: jpoden buoy (6g. = 0.62) is 50 mm by 50 mam by 3 m long is made to floatin
BEvod = Wa + Wnt Be water (6.g, = 1.025). How many N of steel (sg. = 7.85) should be attached t6
1000004 «03x y)=5033+«12)
1000(0.5)0.1% z
es bottom to make the buoy float with exactly 450 mm exposed above the
y=07m
held-y
5
sy
h=12-077 h
h= 043m BEset + BFwood = Woveos = Wot = 0
BEset = Yow Vito
BEwet ® (9810 x 1.025) Vien
BF = 1.05525 View NI
BFa tm Vo
BFsmy = (B10x 1.025)(005)4255)
Problem 3- 69 BF= 641 N
Ifa 5:kg steel plateis attached to one end of a 0.1 m x 03 m x 1.20 m wood Woot Yas Vaan
pole, what isthe length of the pole above water? Use sp. gr. of wood of050 and Wows ® (0810 x 0.62)(0.05)43)]
that ofstest 78 oo
‘Solution Wt * Yt Vo
Wout * (1000 x 0.8)(0:1 «03 1.2) Wat ® (9810 x 7.85) Via A
Woo 8 Rg (FET wae Wnt 77008.5 Vint
1005525 Vg 641-4562-77085 Vau=0 me Y Yow
Wa
Pet Ail: fea
Vans = 0.000276?
Bry = 30d n= 810085)000270
BF. = 1000 V5 tam a 21.
MP ae:
Wem 1000785)
Vs = 0.000697 m* Vs=5 fe an
BF, = 0637 Ke
tergiegisan piece of lead (op. gr. 113) i tied toa 130 ce of cork whose specific gravity is
edtoe
745 m oO They fotust submerged inwater, Whatstheweight ofthe lead?
e124
We+ Wis BFc+ BF:
Wee ¥eVe
We= (1 0.25)(130)
We=325 grams
BFe= ye Ve
BFe = ()(130)
BEc= 130gem
Wien
We= (x13) Vi
Wi=13
B= We.
BRi=() ev.
9254113 Vi= 130+ Vi
Vi=9d7ecc
(2) Lod fastened ote teinder__(8) Lea is paced ins the eynder
Misters |Lead is fastened outside
BFe=1~ Vo
Problem 3-72 (CE November 1993) BFe= 9811 0715)
Anhallow cylinder 1 m in diameter and 2 m high weighs 3825 N. (a) How BFc=1156kN
KN of lead weighing 110 KN/m? must be fastened to the outside bottom of BEM 1 Ve
cylinderto make it float with 1.5 m submerged in water? (6) How many kN.
lead ifit is placed inside the cylinder?
BFL =981¥;
Wo=nvi
We= toy,
ER-9
B+ BF=We+ Wh
1156+ 981V, =3825+ 110%.
Vix 00720?

Wi +3825 =1156
W.=7.735KN
‘A stone cube 280 mun on each side and weighing 425 N is lowered into a tan
‘onthining « layer of water 1.50 m thick overa layer of mercury. Determine t
[Positionofthe block when ithas reachedequilibrium.
Solution
Ws 425
BR ywVou
BEy= (810 x 13.60.2842)
BFy= 1045981.
BEy= tw Vow
BE= 9810{(0.28)(028 - )]
BEiy= 769.10.28 ~3)
BE = (624 « 1.4)((2.2)%22-W)] + (624 x 08)2.27¢0))
[ry =0) BE = (624 « 223] (3.08 - 14h + 08h)
BEy + BEw=W. We (624 x 1.6)[227.2)] + 624 0712270.))
105981 x + 769.1(0.28 - 3) = 425, We {62.4.2 240953)
9690;7t3 209-654
x= 00216 m ier
pee, [62.4 x 2.27] 8.08 -1.4h + 0.8%) = [624 « 2291253)
‘Therefore; the block will float with 21.6 mm below the mereury surface, 308-1.4h-+ 08h = 1.76: 07
hn 0917 ft
Problem 3-74 lem 3 = 75 (CE May 1997)
‘cube 22 fet on an edge 100-mm diameter solid cylinder is 95 mm high and weighing 375 N is
Te and upperbalofog = umersed in a liquid (y = 8.175 KN/m?) contained in a tall metal cylinder having
07, It rests ina two-layer ymeter of125 mm. Before immersion, the liquid was 75 mm deep, At what
fluid, with lower sg. = 14 will the solid cylinder float?
and upper sg. = 08.
Determine the height tof
the top of the cube above
the interface.
poclen beam of sp. gr.0.64 is 150 mm by 150 mm and is hinged at A, as
in the Figure. At what angle @ will the beam float in water?

(@)Before immersion
Solve forthe draft in figure (b):
ft
nVomW Weightof beam, W’= Pun Vee
(8175) Vo=375
Vo = 0.004587 m>
Weight ofbeam, W = (9810 0.68)(01576))
Weightof beam, W= 706.32 kN
Vo=458,716 mm?
4 (100)? x D = 458,716 Buoyant fore,
fant force, BF = 9,810{(0.15)2 x}
Draft D = 584 mm Buoyant fore, B= 2207252
When the solid cylinder is immersed, the liquid inthe tall cylinder rises [EMa=0)
BE(G-05x) cos 0 = W25cos 8)
to volume of liquid displaced
‘equals the total volumeofreal .andTherefore, the volume of liquid disp
imaginary liquid above the original, 220,725x(5-05) = 7063225)
Vannt= Vo 5r-052= 8 oe
4 (125)16) = 458,716 05x-5r+8=0
99738 mm = LANES? 40518)
(05)
From Figure b: x=2m
B+i=D+y sino= =1/3
y=75 +3738 -584
y=53.98 mm
‘Therefore; the solid cylinder will float with its bottom 53.976 mm above
‘bottom of the hallow cylinder. ©
CHAPTER THREE 4
“Total Hydrostatic Force on Surfaces:
3-77 (CE Board May 2003) 13-78 (CE Board November 1993)
the figure below,i is shown thatthe gate is 1.0 m wide and is hi going from salt water (p. gr. = 1.08)to fresh water (op. gr. = 1.0) sinks
oto of thse gate. Compute the following fan ancl after burning 72730 kg, of coal rises up by 15-24 cm. Find the
{@) the hydrostatic force in kN acting on the gate, 3 displacementofthe boain sea water in KN.
(Wthe location ofthe center of pressureofthe gate from the hinge,
(©the minimum volume of concrete (unit weight = 23.6 kN/m?)
keepthe gate in closed position.

FeyRA=9.810)@x1) eeoy,
F=1962kN osm
y= 4@)=0.607m 4 1
[EMA =0}
Fry=Tx25 Foxe (0)
19.62(0.667) = 25T
yee hhave to assumethat the boathave a constant cross-sectional area A below
From the FBD of the ‘water surface and use Ywaur ™ 1000 kg/m’.
‘concrete block: te
Bea BAW
tow Vos We
poe
BE =YeVonw 981 Vou W= (1000 1.05)64(0)]
We Yam Vows ™ 23.6 Von We 1080AD > Fa.)
$.232+9.81 Vow = 23.6 View
Veauc= 0.3796 m3?
CHAPTER THREE
‘Total Hydrostatic Force on Surfaces

For any floating body; Buoyant force = Weight


W= 1000[4(0+0.07602)) Se) Solving for displacement in sea water:
W=10004(0 + 0.076 esa Vor= W
gure (): (64) Von = 24,000 x 2.240
BE,= W- 72730 Voy = 840,000 88
1O00{A(D -0.0762)] = W-72730 > Eq, @) Solving for dapace mentin rsh water
eee Ve
(Esha) = 24,00» ni
From Eq. (1) and Eq. (2):
Wem Vachebe an
11080AD = 10004(D + 0.0762) Foure (a)
1020D = 10000 + 76.2
D=254m_ (draftin sea water) Lot irbe the difference in the drafts in fresh & seawater:
Von = Vou = Area xt
From Eq, (1) j= 86430868 840,000
We 1030A@258) 32,000
W= 2616.24. h= 0768
From Fa, G) Drain fresh water, D= 34 076= 34.764
10004(254 - 0.0762) = 2616.24 -72730
2463.8 = 2616.24 - 72730
A=87.B mt jem 3- 80 (CE Board November 1995)
der an arbitrary shaped body with a submerged volume Vs and a density
Therefore: submerged in a fluid of density p; What isthe net vertical force onthe body
W= 2616267723)
W= 12248 kN (81/1000)
W=1,248,529

Problem 3-79
A ship having a displacement of 24,000 tons and a draft
centers a harbor of fresh water. If the horizontal section ofof 34thefeetshipin atoce
Waterline is 32,000 sq.ft, what depth of fresh wateris require float the ship?
Assume that marine ton is 2,240 Ib and thatsea water and freshd towater weight64 gas pressurized to 111
I ‘pef and 62.2 pcf respectively, erica balloon, 9 m in diameter is filled with helium
a rope to the ground. Neglecting.
Pa ata temperature of 20°C, and anchored by tension
dead weightof the balloon, determine the in the rope. Use R = 212
°K for helium gas and Yu= 11.76 N/s?.
We=4562N (from Figure3-1)
6 tao? BF = S06. 025(0.0574)
RT 212(273+20) ie BF =25.138L_
Yatom 1.787 N/m’
Pian {2Mo= 0} cos 6) -BE[L/2) cos6] =0
Joh W(L.5
we 45.62(15)-25.138L(L/2)~0
[erv=0) R572 = 68.48
BF-W-T=0 1=233m és
BE Yar Vastion T sin = 2/L ie
BF 1176@81.7)
BF= 4488.8 N
WeYan Vin
W=1.787 @81.7)
maesain- so arep ae los Peo y
J circular cone is 100 mam in diameter ‘and 200 mm high and weighs 1.6

Problem 3- 62
‘The buoy in Figure 3-1 has 80.N ofsteel weight attached. ‘The buoy has lodged
against a rock 2.m deep, Compute the angle 6 with the horizontal at which ‘The required downward vertical force is:
buoy will lean, assuming the rock exerts no moment on the buoy. = BE-W.
Solution BE
BE = (98Yess10Ver» 08)
e
(x/3)0.1/2(02)1
BF=4.11N
Feail-16
F=251N
Note; ‘This force F= 251 N becomes
“constant no matter how deep further
the cone issubmerged.
‘As=0425nr2 (shaded area)
Let V = volume of wood
From geometery: In water:
‘As Anco=Avimg Sy ER =0)
042507 = 14726, Ys sind BE,-W-F=0
6,-sin0= 267 9e10V-W =40
_ 40+W
ve 0 by tial and error v > 50)
Try 0= 170°
170"(x/180")-sin70"=276 (267) igo
ee)
Try 0= 166°
166%(s/180")-sin166" = 2655 (267) a)
(i071
Try 0 165.44" c
16644°(5/180") -sinl6544°= 267 OK. oai0.132222-10
hery 52+1.3W-W= 100
= 1-()c08 0/2) W=160N
hm 1 (1) cos (16648°/2) From Ba)
ee 9810
v=00204 n°
seh y= WaWha 3
Unit weighty 160
force of 100N to Keep it immersed in glycerin (sp. gr. = 13). Find the weigh Unit weight, y=7843 N/m?
and sp.gr. ofthe ofthe wood.
oes = oe Taood 73oe
Sp. grs= 08
‘Since the volume of ol remain unchanged;
‘A tectangular tank of internal width of5 Veaaia= Vestn
‘m, as shown, contains oil of sp. gr. = 08 (0.5)6)(.25) = (05)6)(H) -0.1278
‘and water. () Find the depth ofoh @) W=1301m
Ifa 1000-Nblock ofwood is floated inthe
oil, whatis the rise in free surface ofthe ‘As shownin Figure b, ifthe oil-water interface drops by a distance of y the
‘water in contact with ir? free surface of water will rise by y/2, since the cross-sectional area of the
right compartmentis twice thatofthe let compartment
Sum-up pressure head from oll surface to watersurface in m of water
(0+ 1301(08) + @-y)-4-y/2=0
1.04081-3y/2=0
3y/2 = 00408
{y/2=0.0136m or 136 mm
Therefore; the free surface of water will rise 13.6 mm,

Tin open cylindrical tank 350 mm tn diameter and TEA RigR 1s THEE
“Verticallyinto a body of water with the open end down and floats with a 1300 N
lock of concrete (sp. gr. = 24) suspended at its lower end, Neglecting the
ght of the cylinder, to what depth will the open end be submerged in water?
Figure (b)

(@)Depth ofol: (Referto Figurea)


Sum-uppressure head from ollsurface @ to water surface @ in m of water: [EF = 0)
m
Buenos) +3-4= 22PaY Be + BFqi~W=0 Eq.(1)
Bow * Your Vase
u me)
0+08h-1=0
hn 125m
"(Rise ofthe water surface: (Refer toFigure })
BE=W.
a VomW
(9810 x08) Vo = 1000
Vo= 0.1274 ms
BFgt = Ywaes Vo
BF = 98101§ (035)*H]
BF) =943.83
‘CHAPTER THREE
“Total Hydrostatic Force on Surfaces
From Ea.)
‘5A17 +993.83h - 1300-0
= 0803 m
Applying Boyles Law (taking pan = 101325 kPa)
Before insertion:
‘Absolute pressure in ar,py = 101.325 kPa
Volume of air inside thecylinder, Vi= # (035)%(0.18)
Volume ofair inside the cylinder, Vj = 0,0173 m)
After insertion:
Absolute pressure in air,p= 101.325 yh
Absolute pressure in air, ps = 101.325 ++ 9:81(0.803)
Absolute pressure in air,ps = 109.2 kPa
Volume of air inside the cylinder, V>= $ (0.35)? x
‘Volume of air inside the cylinder, V2 = 0.0962x
eee sre: Low Tice
Ips Va= pa Vay
101:325(0.173) = 100-2(0.09592)
x= 167m. WeightoF chain = 17 kay
rhty= 18 sty ofsteel = 7,790 kg/m?
y = 18-167 +0803 flumeofsteel (chain)= 12/7790
y=0833.m lume ofsteel (chain)= 0.00154 mypermeter length
‘Therefore, the open end is submerged 0.933 m below the water surface.
BR =o)
BE, + BF-Wi-Wa=0
Problem 3 - 88 (CE Board) BFi= tm Vo
A cylindrical buoy 600 mm in = (1000 x 1.081 (0.694096),
‘moored in salt water to a 12 mdiame ter and 1.8 m high welghs 205 kg. It
length of chain weighing 12 kg per m of BF, = 27958 kg
length. At high tide, the height of buoy protr
What could be the length of protrusion of theuding
buoy
above water surface is 08 BES" YoVenn
Density ofsteel is 7.790 kg/m’. Use density of water =ifthe tide dropped 21 m
1000 kg/m’. (1000 x 1.08(.00154(0)}
BF, = 1.5961
Wie 12h,
279558 + 15861 - 205-121 =0
L=716m.
Depth ofwater, H= +096
Depth ofwater, H= 812m
Depthof water, H’= H-21
Depth of water, H'= 602m Sc J
ra |
Draft, D= H'=L" W= yous View z y
Draft, D= 602-1?
PA=9)
BF+ BPs-Wi-Wa=0
BF = (1000 x 1.08))13.(06)°D} Buoyant Force:
BE = Yue Via
BF= 291.23 (602-L) BF = (9810) $ (0.15)?
BF), = 1753.18 - 201231" BF = 138.69N
BP's» (1000 x 1.03){0.00154(L")] o
BP» 1.5861! Depth of poo:
Woe 12 Work done by W= Work done by BF 7
Wid+h) = BEG)
1753.18 - 291.231+ 1,586L'- 205 - 121 = 0 '58.25(43 + h) = 138.698
[5.3m h=311m.
D = 602-513 = 0.89
=18-D biem 3-50
eee
y= 0:91 m (length ofprotrusion) hhydrometer weighs 0.0214 N and has a stem at the upper end whichis 279
in diameter, How much deeper will it float in oil (6p. gr. = 0.78) that in
eohol (sp. gr. = 0.821)?
Problem 3-89 (CE Board)
‘A wooden spherical ball with specific gravity of0.42 and a diameterof 300 In alcohol:
is dropped from a height of 4.3 m above the surface of water in a pool BE=W
‘unknown depth. The ball barely touched the bottom ofthe poo!before it bega (0810 x 0.821)Vou = 0.0214
tofloat. Determine the depth of the pool. Vou™ 2.657 x 10% m* ‘
Vo. 2,657 mm*
Ino:
Br=w
(0810 078)Voo=0.0214
Vou 2.797 106 m?
Voc 2.797 mi?
eS echo s=0m1 —O45=070
AV= 2797-2657 =140mm?
AVo= $2797 h=140
= 229mm
A plastic cube of side L and s P. 0.82 is placed vertically in water.Is the eck of wood (sp. gr. = 0.64)is in the shape of a rectangular
stable? 10cm square bse. Ifthe Block oats in salt waterwih ts square
Solution
‘The bodyis stable if M is above G. bet
Draft, D= 981
Draft D=082L ‘The bodyis stable when Mis toon 106m
Gand unstable if Mis below
ebb? | With smaller value of H,the
i) (Lx1(082L) ter M will become higher
‘MB, =0.102 1 G making it much stable. When.
cases, M will move down
GB,=1/2~D/2
GB, 0.091
Since MB, > GB,, Mis above G,
‘The body is stable,
GB,= H/2-D/2
Problem 3-92 Draft, D= $41 = 06214
A solid wood cylinder ofspecific gravity 0.6 is 600 mm in diameter and 120 ‘GB, = 05H - 0621H/2
‘mm high, If placed vertically in oil (sp. gr.= 0.85), wouldit be stable? GB, = 0.189H
Solution MB 7)
Draft, D= 2:
spar “00100?
1=200mm Me “Goya
Draft, D= $42 (1200) MBS = 10) 13419
Draft, D = 847 mm_ 10621)
TeI
MB,B= =

MB, = 4000)¢
+(300)?(847)
‘MB, =26'56mm
GB, = 600 - 4647)
GB. =1765
Since MB, < GB, the metacenteris below G.
Therefore, the body is unstable,
Since MB, < GB, Mis below G and the cone is UNSTABLE.
Initial metacentric height, MG = MB, - GB,
Initial metacentric height, MG = 7749 - 117.85
Initial metacentric height, MG =-38.96 mm

‘vertex down. If the specific gravity of the wood is 0.60, wouldit be stab rectangular scow 9 m wide,15 m long, and 3.6 m high has a draft in sea water
Determine also its initial metacentric height Of 2.4 m, Its center of gravity is 27 m above the bottom ofthe scow. Determine
the following:
Solution (j) The initial metacentric height,
(b) The righting or overturning momentwhen the scow tilts until one side is
Veaoa™ 4 (850)2(1000) just atthe pointof submergence.
Vent ® 128,281,700 m?
Solution
Vo™ 22 Vanes (@) Initial metacentric height:
Vo = 0.6 Vat

By similar solids:
Mesa « (200)!
vo UD
Vase -(2y
Ting D Initial metacentric height, MG = MB,GB.
D=#34mm Initial metacentrc height, MG = 28125 -1.5,
x 50 Initial metacentrc height, MG = 1.3125 m
‘$434 1000
x= 2952 mm.
MB,
1
We

40952)" =7749mm
‘MB,= 76,969,020
rom the Figure:
GB, = 750 -3D/4
GB, 750 - 3(843.4)/4
G8,11745mm
(CHAPTER THREE
“Total Hydrostatic Force on Surfaces
tic height, MG = MB.-GB, ‘Along longitudinal axis (rolling):
tric eight, MG = 261 -15= 141 m B=10m
ince MG > MB,, the moment is righting moment. o,~ 82).
Righting moment, RM = HV (MG sin 6) wp|* 2 ‘| where = 0
We BE =o 0?
We bal Lalaeyea) Sezraniat pHi aa
Righting moment, RM = IMTS) eee
Metacentric height, MG = 545 ~ 3.235
Metacentric height, MG=2.215 m_ (the barge is stable in rolling)
Along transverse direction (pitching)
discinaios
barge floating in fresh water has the form of a parallelepiped havit
sualbyaatipe Lawabasal
Foe askvenaGuMeherbones Mel iveeee oteae Baa.
ere
7
aboutits longest and shortest centerline, and determine whether or not th 2
barge is stable, MB, 731133)
20
Solution Metacentric height, MG= 4902-3235

C3 temo ton
PAD VZITA: biem 3-97 (CE August 1973)
rane barge, 20 m long, 8 meters wide, ad 2 meters high loaded at is centerof
tons, floats on fresh water with a draft
with
1120 meters its center20ofshor
rollerhasweighing
road and gravity located along its vertical axis a a point
1.50 meters above its bottom: Compute thehorizontaldistance outto one side
padorn which
the centerine ofthefrom
it had ited bargethethrough
center which the crane
ofthe deck, andcould
tip theswing
bargethewith2-4on
the
Solve forthe draft, D: 20-meter edgejusttouching the watersurface?
GF=")
yVo=W
9.81 [10 30 « D] = 4,500
D=153m
GB.=4-D/2
GB, = 4~1.53/2
GB,=3.235 m
eto198
“Total Hydrostatic Force on Surfaces

=so0 kg e=dsind
We (20900) 981 2 =0552sin 1131 108 ma
We 176.58 kN GB,=14-d= 0.848 m
r p= 2120| fs =|
2
e
of ef * an?a3t PA
Lm 4

MB, 120) p z ] oy
MG= 4509-0848 =3685m
x=MGsin6
BE=1Vo) x= 3.685 sin 11312 = 0723 m
BE= 981[8 «1.2 20)
BE= 188352 KN = Wr [eMe=0)
(BA x= Wall +2)
Weight of barge, Ws= BF Wy 1,883.52(0.723) = (176.58)+ 0.108)
Weight ot barge, Ws= 1,883.52 176 = 7.604 m > Florizontal distance from the centerofthe deck
‘Weightof barge, Ws = 1,706.04 kN 58
Tilted position: Problem 3-98
"A wooden barge ofrectangular cross-section is 8 m wide, 4 m high, and 16 m
Jong. Its transporting in seawater (5 = 1.03) a total load of 1,500 kNincludin
“its own weight and cargo. If a weight of 75 KN (included in the 1,500-KN) g,is
shifted a distance of 25 m to one side,it will cause the barge to go down 450
mm in the wedge ofmmersion and also rise 450 mm in the corresponding wedge of
‘mersion. ‘The barge floats vertically (on an even keel) before the shifting of the
‘weight. Compute how far above the waterline is the center of gravity of the
loaded barge.
Solution W = 1500
for the draft, D: 3-99
BE=W ‘Waterline section ofa 1,500-KN barge is as shown. Its center ofgravity i815
(081 x 1.03)[8 « 16 D]= 1,500 ove the center of buoyancy. Compute the intial metacentric height against
D=1i6m
In the Tilted position:

T= Tonge * lange * laste


1 Ye (128) + yy (OMAPx2 + EO)
T= 67653 m‘
(BF =W)
9.81 Vo = 1,500
Vo = 152.9 m*
[2Ms = 0} MB, = 67653
visors one
IMG = MB,~ GB)
MG = 4425-15
(MG=2925 m > initial metacentric height

b= (h +058)sin642°
ADSI + 0.58) sin 6424] + 752867) = 150000518)
i= 2947 m > distanceofG from thews.
PER THREE CHAPTER THREE 4 gy
‘Total Hydrostatic Force on Surfaces

{in a tank is pressurized to 80 cmHg. Determine the total force per meter
‘on panel AB.
‘Ans: 482kN
vertical rectangular gate 2:m wide and 1.2 m high has water on one side
‘surface 3m above its top. Determine the magnitude of the total hydros
force acting onthe gate and its distance from the water surface.
‘Ans: F= 846 XN; y," 35

‘A vertical semi-circular area of radius r is submerged in a liquid with


diameter in the liquid surface. How far isthe center of pressure from the lig
surface?
Ans: 05
the figure shown, the 8-ft-diameter cylinder, 3 feet long weighs 550 lbs and
onthe btm fs tek it 3 ee eg asYo ealet
4 feet, res
Trapezadal erosesection 9 i wide at the Dotiont and Sm wide at te i ‘the magnitudes of the horizontal and ory Carats=
Determinethe following: (2) the weight ofoil, t)the force on the bottom of & ce that will keep the cylinder touching the tank at A.
vat, and (c) the force on the trapezoidal end panel. ‘Ans: Fu 749 1b >
‘Ans: (@) 1002 KN;(#) 7528 Fy= 2134 Ibs v
(9230
Problem 3 - 103,
Freshly poured conerete approximates a fluid with sp. gr. of 240. The fig
shown a wall poured between wooden forms which are connected by six bolts
[Neglecting end effects,compute the force inthe lower bolts.
‘Ans: 19,170 I

Problem 3 - 106
ompute the hydrostatic force and its location on semi-cylindrical indentation
BCD shown. Consider only 1 meter length of cylinder perpendicular to the
‘Ans: Fy = 1095 KN @ 1.349 m below D
F,=205 kN @ 0.531 mto the left of B
CHAPTER THREE 4.99
“Total Hydrostatic Force on Surfaces

‘Problem 3 - 109
‘The I-m diameter solid cylinder shownis 8 m long perpendicular to the figur "A block of wood having a volume of 0.034 mand weighing 300 N is suspended
| and rests in static equilibrium against a frictionless wall at O. Determine t {h water as shown. a wooden rod oflength 3.4 m andcross-section of200 mm?
‘unit weight ofthe eylinder. “rattached to the weightand also to the wal. Ifthe rod weighs 16 N, determine
‘Ans: a= 105°

Problem 3 - 108
per cubic meter and water weighs 9790 N per cubic meter. Coefficient
fiction between the damand foundationis0.55. Determine the factors of safe
‘against sliding and against overturning, and also the soil pressure atthe Problem 3-110
landtoe. Assume hydrostatic uplift varies uniformly from full hydrostatic hea “Two spheres,each 1.3 m in diameter, weigh 5 kN and 13 KN,respectively.
atte heel ofthedam to zero atthetoe. Consider 1 length ofdam. ‘are connected with a short rope and placed in water. What is the tensi
Ans: FSo= 2.20; FSs= 1 rope andwhatportion ofthe lighter sphere protrudes from the water?
‘pnt ™ 85:2 KPa; ge = 2002 ‘Ans: T= 1.74 KN; 401
THREE
Force on Surfaces

Weighing 125 pcf is 1 ft squar and 9 inches deep floats on a strat


Homposed of a 7-in layer of ewater above a layer of mercury
live the position ofthe bottom ofthe block.
6F 260 Ib is applied to the center of mass ofthi(#)s block
Ifa downward v
, what is the
[sition ofthe bottom ofthe block?
‘Ane: (a) 08 below
(0/467 below me
Problem 3 - 112
Would a wooden cylinder(sp. gr. = 0:6) 660 mm in diameterand 1.3 m long
stableif placed verticallyin oll (sp. gr. = 0.85)? Fcc elec lene
Ea ace et ae ave relative

pa eres ema eet


ee
sara
Se
Ans: Not stab dis moving with a constant speed (uniform elect), the conusott im

Problem 3- 113
A rectangular scow 7 ft by 18 by 32ft long loaded with garbage has a draft
area
feet in water. Its center of gravity is 2 ft above the waterline. Is the
stable? What is the initial metacentric heigh t?

Horizontal Motion i
Problem 3-114 Consider « mass of fluid moving with a ina acceleration «as shown inthe
igure. Considering a particl e in thesurfa ce, the forces acting are weight W
A cube of dimension 1 and sp. gr. 0.82 floats horizontally in water. Is the = Mg and the fictitio us inertia force (revers ed effecti ve force, REF) wl
stable? ‘equal to Ma, and the reactio n N which must be normal to the surface .
z
Ans: Stab
der a mass of fluid accelerated upwards or downwar ds with «
eration ofa as shownin the Figure. The forces acting ata pointkbel ow
surface are the weight ofthe liquid above the point, y¥, the inertia foree,
and the pressure force F= pA,then,
f= 0}
P= Mery r+
Mapve Ly =—
‘Therefore; the surface and all planes of equal hydrostatic pressure must t q 2
Inlined at this angle@ withthe horizontal. P-LL ver 1 iw. w
: re
Consider a mass of fluid being acelerated upwards a an ncination «witht pe AY
horizontal so that = acosa anda,=asina. P= < (anes Ah)
poi +a/s)

Use (+) for upward motion and (- for downward motion.


Note: spostivefr acceleration and negate for decteration.

OTATION (ROTATING VESSELS)


‘liquid mass i rotated about a vertical axis constant angular speedof
‘(in radians per second) , every particle experie nces a normal aceleraion
2
which sequal to X= 0x where xs the partic’ distance fom the axis of
taion, This acceleration causes an inertia force (centrifugal force or reversed
effectivefore) whichisequal o Maroryox,

Use (+)sign for upward motion and () sign for downward motion.
i i
‘where o is theangular speed inradian per second.
NOTE: 1 rpm = x/30 asec
in Figure 4-1 (b, the relationshipr between anytwo points in
{n the parabola canis
given by (squared property of parabola):
From the force polygon:
tang = cr
teng= = W/ai0"s
Wao x

‘Where tan is the slope ofthe paraboloid any pointx from the axis of rotation,
"utp sunFAcE conprrions or closed cylindrical containers more thanhalf-full of guid, rated about
For open cylindrical containers more than half-full of liquid, rotated abl gesearn te
its vertical axis (4> H/2);
(6) When = 6m/s?
tind e 3Rr 6
Problem 4-1 om 3145"
‘An open rectangular tank mounted on a truck is 5 m long, 2 mimu wide and 2.5:
thigh is fillned with water to a depth of 2 m. (a) Wha t max m horizo x= 25 coB145"
‘acc eler atio can be imp ose d k out
on the tan with spi lli ng yy water and @ x= 400875 <5m
det erm ine the acc ele rat ing force on the liquid mass ? (0) If the acce ler Via» Y(4.0875)(2.5)(2)
Increased to 6 m/s, how much water is spilled out? Vo 10.22 mi?
Vecana * 2)2)(8)
Solution Verna= 20:
(@ Vogts * Veit ~ Ve
20-10.22
Fae

‘completely filled with gasoline (sp. gr. = 0.82) andaccelerated horizontallyofa


the
“The figure shows the water level under maximum a when no water is and at the front wall
ating at the rear wallmass.
Iin/st. Find the total force ting
spilled out. Find also the accelera force on thefluid
tand= 9§ =02
tand= 4 =02
g
a= 02981)
a= 1.962 mys? y_3
4981
% y=123m
‘Accelerating Force, F = Ma
‘Mass, M= p(Volume ofliquid) R= 12234075
Mass, M = 1000{5 x 22} i =1973m
‘Mass, M = 20,000 kg Fw YHA
‘Accelerating Force, F = 20,000 x 1.962 Fra O81 » 082)(.973)§ (5)
‘Accelerating Force, F = 39,240 N Far = 28,05 KN
or; Fron = 7H A
= Frewan~ Fnetnat Fron = O81 « 082)(075) [5 (L571
F=981(3$ )25@)] -981( 12 1.520) Fun =10.66KN
F=39244N
‘Accelerating Force: Also:
F=Ma Vas oepan = Varna
Mass, M = p(Volume) 40.2)2)= (1/2)x2@)
x16 E42)
F = ((1000 « 082)1§ (5°41 @) 2=16/x > Ea.)
F=17390N. Substitute z and x2to Eq (1)
Fe 1739kN (1.6/2) -16= 4.1% > multiply by x
F = Pru - Pat
F= 28.05 -10.66
P= 17.36 kN.

Peyia Solve for aand y:


200981 F (202) F=Ma= 10400
i =54m M= Mae + Mat
-M=1,000{(1.82)(1.82)(0.91)] +3425/981
yeh -1=41m M= 3363.42 kg,
By similar triangles: 10400 = 3368424
a= 3.02m/s
PUA,
dr-u-4tx EQ) Went
tno= Ost
‘CHAPTER FOUR
RelativeEquilibrium of Liquids
Amen 541 me
981” Ost
y=029m £091 m (OK) Vane =5.11(15)= 7.665Bi
a
h=091-y
h= 062m. Vies = 12.4695 - 7.665
P= 9810 (0.62/2) (0.62 « 1.82} Vige = 4.7985 m>
P=3432N
pat = 8.6985 -4-7985
Vapans = 39 >
Problem 4-5
‘Anopen trapezoidal tank having a bottom width of 3 m is 2m high, 15m A> 6 (CE Board)
and has its sides inclined 60° with the horizontal. It is filled with water to
depth of 15 m. If ee vessel 3 m in diameter containing 2.4 m of wateris beingraised. (a) Find the
a
st, how much we Ga i l e d out2. ee ee ire atthe bottom of the vessel in kPa when the velocity is constant, and (b)
ia ee sd the pressure at the bottom of the vessel whenitis accelerating 0.6 m/s?
Solution ards.

For vertical motion:


p=(48)
h=24m
tno a$= 81 (a) When the velocity isconstant, a= 0,then.
= 24.64" path
p= 98124)kPa (pressureatthe bottom)
p=23544
Vis Vor ~ Vie
Veag = 24P52 (1.5) x 1.5 When2= 06m/s? (uses for upward motion)
Vere * 8.6985 m? p=981d +(t+ sae)
View ™ Vasco Vase
Vanco = 22530 (2)(1.5)
Vanco = 12.4635 m?
Vase = Aase(15) Bereta ed Weis toed
Anse = %2 (AB)AE)sin @ nenthe motion is(a)
a= 180° 60° -24.64
(¢) Downward motion with a positive acceleration (use ”-" with +8 m/s)
p= 081% 08)8) (1-3)
poe kPs
(a) Downward motionwitha negative acceleration (use "” with a= m/s")
p= 981% 08)9) (sh)
ay=12sin 15°
ay = 0.31 m/s?
(@) Whenthe motion is upwards: lindrical water tank usd in lifting water tothe top ofa tower is 1.5 m high.
tne 9 the pressure at the bottom of the tank is must not exceed 16 KPa, what
981+ (O31) {imum vertical acceleration can be imposed in the cylinder when it filled
= 6538 ih water.
lution
() When he motiondownward: ‘we parked.
3ai- (031) Ena
6 =6.955"
fem 4-10
Problem 4-8 to its diameter is halffilled
‘open cylindrical vessel having a height equalaxis
‘An open tank containing oil (sp. gr. = 0.8) is accelerated vertically at 8 m/s% ith water and revolved aboutits own verticalso thatwith a constant angular
Determine the pressure3 m below thesurface ifthe motionis (a) upward with ‘of 120 rpm. Find its minimum diameter there can be no liquid
positive acceleration, (®) upward with a negative acceleration, (¢) down
‘with a positive acceleration,and (4) downward with a negative aceleration.
that there's no liquid spilled, base of
rt ‘coincide with the =..
Te pean atadepthsgrenty.7=H(1+2) Fe cewoeginie: Sesto ae
‘ slr inalhaul theeight of :
Bo at ae : : thereforeequal to
of the iscylinder.
Bathe paraboloid
aaa
(@)Upward motion with a positive acceleration (use “+” with a= +8 m/s?)
P= 81x 08)0)(1+3%) 7q Pe ens
prQ7enre ee m0
X. with a negativeacceleration(use
(©) Upward motion . “+” with a =-8 m/s!)31 Hee
2g ; D2

P= @81«08)8) (1+58) [ Cee er M4 tt 7


prasekPa o=darad/see $
| CHAPTER FOUR
Relative Equilibrium of Liquids eladve Buiaofguise 219
pq Gr@/2 Vegas = Va a = Va at
20981) Vert = ¥(0.67(1.63) = n(0.6)%(0.7)
Vegas = 0.3m x 1000 lit/m?

‘Anopen cylindrical tank 1.6m in diameter and 2 m high is fll ofwater. ‘open cylindrical tank, 2 m in diameter and 4 mhigh contains water to a
‘rotated about its vertical axis at 30 rpm, what would be the slope of the wa
surface attherimofthe tank? ‘of 3m. Its rotated aboutits own vertical axis with a constant angular
Solution If@ = 3rad/sec, is there any liquid spilled?
Slope = tan What maxsoum valu of o(nrpm) canbe imposed without pling any
ox If @=8 rad/s, how much water is spilled out and to what depth will the
Beer water stand when broughtto rest?
‘What angular speed «(in rpm) will just zero the depth of water at the
center of the tank?
If@= 100 rpm, how much area at the bottom of the tank is uncovered?

i
Problem 4-12 (CE Board November 1978) ©" 3rad/sec
‘An open cylindrical vessel 1.2 m in diameter and 2.1 m high is 2/3 full of water. n= OFF
Compute the amount of water in liters that will be spilled out is the vessel is 20981)
rotated about its vertical axe ata constant angular speed of 90-rpm,, h=046
h/2=023<1m
- no liquid isspilled out
Vr=im
The maximum @ so that there isno liquid spilled is such that
h/2=1morh=2m
\

h/2= 0815 > 07 m (some liquidspilled)


(ie=8nd/uc
Pos 4
+ the vortex ofthe paraboloid is
hiredSlo he nk eaginay)
h/2=168m>1m
2031) T wae
‘Area, A= 132

some quispilled but thevorexofthe | y=188m


since k < 4m.
‘paraboloidi inside the tank By the squared property
Vapi = Vain~ Vaoat of parabola:
Varay = Veatis oie.
er
Vaur ina= ¥8x(1)? 8.26) as
Vue teuy = 5.121
ee
Vaseoaan = 51920) 804
Vegan * 3.142 20°
Voges = 5.121 - 3.142, ‘Area, A # 1(0.283)
Vegas * 1.979 m0?
Anothersolution:
‘ater level will estat h/2 from top.
yah/2-11
| yn 63-m
y= 063
| Vipant 72 y
Voys = x 0)97°09 .6?3)
| Vi = 1. 0
(@) The vortex touches the bottom when =m
| oy?
j 4" ean Solve for h (by squared property)
| = 886% 2* A1)y? (05) 2
| = 846rpm sated
} (€) Wheno = 100rpm ha 025K
Aeek
@ = 100(n/30) = 3.33x rad/sec
ae h=538am y
pees 533-22@08)n
||
aa 0 =1023red/secx 2
| ee
| 09765 rom
k= 558>4m
(CHAPTER FOUR
Relative Equilibrium of Liquids
UO) Vee Veins Virotnofcite
Viar=
12 LOO) = (1) (8) - Tas [4x 06S)
(1}86.93) = Yor O505). 6 6.33 -4 29-4)

pif.
‘Problem 4-15 (CE Board November 1993) h=175-275-623
4 1.90 m diameter closed cylinder, 275 m high is completely filled wit = 10975
having sp. gr. of 0.8 under a pressure of 5 kg/cm? at the top. (a) Whatang
speed can be imposed on the cylinder so that the maximum pressure af
bottom of the tank is 14 kg/cm’? () Compute the pressure force exerted = 4884rad/sec x 2
on the side ofthe tank in kg, a 0= 466.84 rpm
Bayh
B= m-279/2
it =173.625m
F =800(173.625)(2n(09)2.75)
P=2.28x10"kg

r e n i n
e a et
r
Problem 4-16 (CE
e ce
See s 1985)

a
Solution
fee oF
‘ince 75% of thetotal volume i spilled out,
the paraboloid will be formed a partoutside
7x(095)= 5969 m “the vessel (Le, with its vortex below the tank)
eng Vegas Var= O75IRr%(1.2)) +
‘Unit of oll, = 1000(0.8) Var = 09n0*
Unit weightof ol, y= 800 kg/n® = 0,0008 kg/em?
@ nace But Var"Viernes
O9n= YenPh- exvy
2g
Solve for: er=eh-ry > 20)
pifar= 5/0.0008 = 6250 cm By squaredpropertyofparabola:
ily= 625m
ER FOUR
ve Equilibrium of Liquids

> Fa.)
In Bq. @)
j : —
TE
18m Ph
uaesaa > Itiph
Seoeeae
‘open vessel, 500 mm in diameter and filled with water, is rotated aboutis.

‘angle of 40° with the horizontal. Compute the speed of rotation in xp.
Nats
buty=h-1.2
es
ua
Liha i (24h aaa
oe gees ae
“The slope of the paraboloid at any distance “x” from the axis is given by:

Finally:
atta) weeds
‘oat
SaE 2(981)
ee
saan Saget
FaTT serscas and 18 m deep i fled with water
‘An open cylindrical tank 1 m in diameter 3 m high is fll of water. At w! ‘An cylin tank 1.2ansminat 60diameter
revouton minute. How much iqud
pr bottom?
‘speed (in rpm) must it be rotated to dischaand
rge 1/3 ofits content. date abouis gon
{s spilled and what is the pressure atthe center ofits
Solution
Let ybe the height ofthe paraboloid.
‘Since the volume of the paraboloid represents the volume of water spilled,
2g
‘Volume of paraboloid = % Full volume of cylinder (= 60 x (2/30) = 2x rad/sec
Yan O5Py= 4x x (0.57 @)
7=12/2=06m

Vigan Ya
Vigan
Vigan ¥x(0.6)(0724)
= 0409 m?
FOUR
tive Equilibrium of Liquids
PPro at the cenier W/2=3.67 m>1m
(part ofthe paraboloid is above the vessel)
yo1s-h =18-07%
ay

y= 1076 ‘Verify the position ofthe vortex (See Page 207)


p=981(1076)=10555%P— Hee
2D
we
(1)

oe
2
ee -em>734m
Problem 4-20 -s.the vortex is inside the vessel
‘A closed cylindrical vessel, 2m in diameter and 4 m high i led with wi
8 depth of 3m and rotated about its own vertical axis at a constant any Vaignay = Vat ns
speed, «. The ar inside the vesselis under a pressure of120 KPa, Vaxdy = 2P()
(@) Ife= 12 rad/sec, whats the pressure atthe center and circumte ty=2? > Eq(1)
the bottom ofthe tank?
(@ Whatangularspeed willjust zero the depth of water atthecenter? By square property of paabol
(©) Ife = 20rad/see, how much area atthe bottom is uncovered?
‘Solution yr
eel y 38.Q)
Substitute x2in Eq, @tg ()
2
(Eyy=27
sP=2h=20-34)
y=383m-<4m
Pressure atthe center,(a
pum hi * Pa
Inedey
y= 4-383 047 m
pr =981(0.17) +120
‘pi = 121.66 kPa (pressure atthe center)
Figure (e) Pressureatcircumference, (at©)
(a) Refer to Figure ): pate * Poe
@=1rad/s in= hth
N17 +734=7.51m_
pe=981(751) +120 5
ps= 193.6 kPa (pres at the cireu ence)
7 sure rhfer
= Caray?
ee

ab
Badretegenervat
en th vortex the panbad
cle the Voor he t
is p= CPO?
Vaie na= Vai init) 2081)
Ys= ne F
1m
ke204m
BH=2RD >Eq.(61)
ab} In Figure (0):
By squared property of parabola: Vee ony * Vie
2A
oee A aA) = Yatye Ya?
Mh 2exin-xye Ea ()
asf 9m.03 By squared property of parabola:
aaa. |
uo

na
Substitute x? to Eq, (41) niaZy 2 9G (62) pare
wet yw 91463)
‘Simpli
Substitute xand 20 Fg (1)
ap 2 yiy)-T ye) multiply bothide by hi?
Dee ye-ye
Butn=4¢y2
Bee (4 y-¥e
Dia i6+8 y2-¥?
8.=2@04)-16
yeaim
= oar
2081)
@ = 12528 rad/sec x 2
= 1196rpm
x2 = 0152
‘Area=x= x(0152)
‘Aren=048 mi (aea at thebottom uncovered)
CHAPTERFOUR
Relative Equilibrium ofLiquids
Determine the position ofthe vortex:
‘vertical cylindrical vessel, 15 m in diameter and 3.6 m high is 3/ HE.
(6* 13) and is revolved about its vertical axis with a constant 2D 0?
2(05) 53m
"speed, The vessel is made up of stel 9 mm thick with an allowable te Since h = 1386 >7.2, the vortexis belowthe vessel,See Figure ())
stress of 85 MPa and has a small opening atthe center of the top cover. (a)
‘angular speed is 210 rpm, whatis maximum the stress in the walls? (b) To.
‘maximum angular speed can the vessel be revolved? Vaea ™ Varian
(09) = Yn? ys ~ Veerye
Solution 1SPaxty-xm >E9.()
(@) @=210rpm«n/30
0 7rrad/s By squared property of parabola
OP atereee ee
now
2
OF
P= Pein Fy 0)
Fy. @)

SabstiatewPandaF EO)
182%yy) y) > multiply both sides byh/
18h yey
Buty. =36+ yo
18k= G6 yp-y2 yy
18k = 1296+7.29+
72y2=18h-1296 > Fa.)
7.2ys=1.8(1386) 12.96
y= 1665 m
y= 1386-1665
y= 12195 m
p=981(13)(12195)
p= 15552 KPa
Solve for hn:
hohe = (15550500)
209)
08? (7)*(0.75)? S= 12,960 kPa
S.= 1296MPa (maximum wall stress)
2" 2081)
k= 1386m
(H) For maximum value of o, S;= 85 MPa 82x 10= pa(18x10°)
5 10° = 2510.3) (1500) 26)
20) pr= 4555 kPa
F hy=7998m
yonh-7998 pata
455598116)
Inka.) y= 29.02,
72{h-7938) = 184-1296 h= he p/y-27
54h =362896 k= 29.02-27-1561
= 10424m i= 1071m
Haat
wey 21081) 1071
_ 27075)? = 16: rad/see x
eke 2981) = 153. rpm (maximum allowable angularspeed)
© = 603rad/secx30/x
o=576rpm OT
Problem 4-23
‘ pet fagatwater-pump-
Problem 4-22 1500 rpm. If the casing is full of water, what pressure isdeveloped by rotation?
‘A118 m diameter closed cylinder, 27 m high is completely filed with gly: Solution
having sp. gr.of 1.6 under a pressure of 245 kPa at the top. The steel plate
‘which form the cylinder are 5 mm thick with an ultimate tensile stress of Presure head, © =
MPa, How fast can it be rotated aboutits vertical axis to the point of bursting?
ia ott
@ = 1500 %/30
(0 = 50x rad/see
5)2
A ._ (60n20)?.8(01).7 = 7074.
Pa 7A mot water
shy
p= 6,990 kPa
Problem 4 - 24 (CE Board)
‘A conical vessel with sides inclined 30° with its vertical axis is revolved pay
‘another axis 1m from its
‘ust it make in order own and parallel. How many revolutions per
that water poured into it will be entirely discharged Solving forhe:
therotative effect?
yay
Solution ign
nea wx? t
. wrx?
a
The waterin the vessel will entirely be nw ee 1257-057)
discharged ata speed when the 20981)
paraboloid is tangent tothe cone a the n= 231.27 m
vertex, hence, the inclination, 0,of the
paraboloid atx= 1 m is 60° or its slope is ! p= (81% 0822)23127)
tan ef 60°, 1
1 pone p= 21865 1865 kPa
Fromthe formula: |
a 1 Problem 4 - 26 4 a
ends 2
' Lim
1 m i} A hs Use whom vtticalstn
st ae 0mm apartapa is filled withwith
are meray
mercury fatoa
tan 6"= 2 (1) ee the midpointof the horizontal section. What angular speed © will produce a
ee 2
@=4.12rad/secx eee
‘ pressure ofabsolute zero in the mercury at the axis?
= 38.6revolutionsperminute
Problem 4 - 25 (CE November 1992)
‘A75 mm diameter pipe, 2m longi jus filled with ol (pg, = 0.822) and he 015,
‘capped, and placed on a horizontal positon, Itis rotated at275 rad/sec
about 5
r=035m
‘vertical axis 0.5 from one end (outside the pipe), Whatis the pressurein Sie rue Ce
atthe far end ofthe pipe? {s absolute zero, then gage
pressure atthe center is
£760 monlig, therefore f= 0.76:
y= 076 +0A5

@= 2817 rad/secx 2
@ =269 rpm
600 mm apart is filled with mercury
Ustubewhose vertical stems areItis axis th
rotated abouta verticalrotated
ff200 mom inthe vertical stems,stem. Howfas
{ts horizontal base 400 mm from one t should it be so
tho difference in the mercury levels nthe stems i 200 mm?

gr as shown in the figure,the sum ofthe height of water in the verticalstems


tefore and after rotation must be equal
2yi +225005)
ptyni > 840)
By squared property ofparabola:
| ot oF , 09%
| zoan (4? 0271- OR: Se
| @=572rad/secx 2 Poa ot0)
0 S461pm ‘Substitute to Eq (1)
it dy 125
| nos
Problem 4-28 Pr?
glass tubing consistof 5 vertical stems which are $00 mm apart connected to cag
water to adepth of 500 mm inth
single horizontal tube. The tube is filled. ithabout 205)?
Yertical stems. How fast should ite rotated and axis through the midi 025- 50
2981)
stem to just zero the depth ofwater in thatstem? o=443rad/secx 2
o=4231pm
sit Ab
“Minimize Idifferentiate Eq, (1) with respect tox and equate to zero:
'A75 mm diameter pipe, 2 m longis filed with water and capped at both ex Be
rm a54ax tan60"<0
tis held ona plane inclined 60° with the horizontal and rotated abouta ver x= 068m
‘axis through its lower end with a constant angular speed of 5 rad/sec. a= x sec60"
Compute the pressure at the upper end of the pipe and (b) determine (4= 0.68 sec60® = 1.36 m
‘minimum pressure and its location in the pipe,
‘Solution Inky ruosay + 170-068tn"
TT
Sincethere isno initial pressure henidton
head at
the lower endtheofpressure
Inthepipe the pipe will yee sSan.as)
Fase ke located1.6 fom thelower end (long the pipe)
remain equaltothestatic
pressure headof173m, and
therefore the vortex of the Problem 4-30
‘paraboloid will be 1.73 m above contains. 150 mm of
the lower end. ‘A eylindrical bucket 150 mm in diameter and 200 mm high that the bottom of the
‘water. A boy swings the bucket on a vertical plane so
Ducket describes a circle of radius 1 m. How fast should it be rotated so that no

h=1z74m
(@) Pressure atthe upper end:
Poppe =
Pomme = 981(1.274)
Pope = 12.497 KPa
(6) Minimum pressure
Pan 1
Solve fork?
Weayte
Pa173-xtan60?
at
yA
yl7ie
We 1274x8+(173-xtan6o") > Eq. (1)
From Figure
c=W
CF=Ma,
w.
Capote

we
gore
wry previous chapters deals only with fluids at rest in whichthe only significant
02 (0925) =9.81 perty used is the weight of the fluid. This chapter will dealwith fluids in
Sees Jpotion which is based on the following principles: (a) the principle of
= 31.13 rpm ervation of mass, (8) the energy principle (the kinetic and potential
gies) and () the principle of momentum.
Problem 4-31
‘A-cubical tank is filled with 2m of oil having
wing sp. of 08. Find the force
‘on one side of the tank when the accelerationsp. isgr.o5 f m/s(a ) vertically upwa of Bul pas ing throughweight flow ral
{oy vertical y COWAWaRT, > lw rate (ex. kg/3e),
fof time This is expressed as a. mas
Solution KN/sec)and volumelow rateorflow rte (ex. 0/5, it/s).
(@) Pang
Felyhg(l +a)
F= (81 <08)@)(1+5/981/2)2))
F= 47392 kN
FapgA
Ee tylig(t-a/gA (Q= discharge in m/s or f/s
Fr Os 09a) 5/9812)
P= 15392 ‘A= cross-sectional area of flow in m?or f°
‘v= mean velocity offlow in m/s of ft/s
‘p= mass density in kg/m? or slugs/°
‘y= weight density in N/m?or Ib/

DEFINITION OF TERMS
tay or todim
Pid Floweonedibenenial, nif oUre dien
usta eonal continuous
onansoa amir
{rt or and rationo t
‘rotational.
"Steady Flow
‘This occurs when the discharge Q passing a given cross-section is const flow is said to be turbulent when the path ofindividual particles are
time, Ifthe flow Q at the cross-section varies with time,the lowis unsteady, egular and continuously cross each other. Turbulent flow normallytuation occurs
the Reynolds number exceed 2,100, (although the most commonsi
‘when it exceeds 4000).
Uniform Flow
‘This occurs if, with steady flow for a given length, or reach, of a stream, mninar flow in circular pipes can be maintained up to values ofRe as high 35
average velocity of flow is the same at every cross-section. This usually oce unstable, and the
10,000. However, in such cases this type of flow is inherentlyflow.
‘when an incompressible fluid flows through a stream with uniform disturbanc e will transform it instantly into turbulent On the other
Section. In stream where the <rosssections and velocity changes, the 4. itis practically impossiblefor turbulent flow in straightpipe touppersistat
sald to be non-uniform. Walucs of R, much below 2100, because any turbulence that is set will be
‘amped out by viscous friction.
Continuous Flow ‘One-Dimensional Flow
‘This occurs when at any tine, the discharge Q at every section of thes ‘This occurs when in an incompressible fluid, the direction and magnitude ofthe
the same(principleofconseretion ofmas) velocity aall points are identical

"TwoDimensional Flow
‘This occurs when the fluid particles move in planes or parallel planes and the
streamline patterns are identical in each plane.

‘Streamlines
Continuity Equation ‘These are imaginary curves drawn through a fluid to indicate the direction of
Forincompressible fluids: motion in various sections ofthe flow of the fluid system.
‘Streamtubes
“These represents elementary portions ofa flowing fluid bounded by a group of
streamlines which confine the flow.
"Flow Nets
‘These are drawn to indicate flow patters in case of two-dimensional flow, or
LaminarFlow ‘even three-dimensional flow.
‘The low is said to be laminar when the path of individual fluid particles do n
cross oF intersect. The flow is always laminar when the Reynolds number
les than (approximately)2100
energy possessed by a flowing fluidconsists ofthe kinetic and the po
‘energy, Potential energy may in turn be subdivided into energy due to
‘or elevation above a given datum, and energy due to pressure in the fluid.
‘amount of energy per pound or Newton of fltid is called the head.

Kinetic Energy
‘The ability of the fluid mass to do work by virtue ofits velocity.
eonee
where:
"z= position ofthe fluid above (+)or below (-) the datum plane.
pe fluid pressure
‘p= mean velocity of flow

low Energy, E
total energy or head in a fluid dflow is the sum of the Kinetic and the
otentialenergies, It can be summarize as:

JERAND EFFICIENCY
sis therateat which work isdone. Forafluid ofunit weighty (N/m)and
moving arate of Q (an?/s) with a total energy of E (m), the power in N-m/s
“(ioule/sec) orwatts is:

Note: 1 Horsepower (hp) =746 Watts


1 Horsepower (hp) 350fb/see
11 Watt= 1 N-m/s=1Joule/sec
‘The Bernoulli's energy theorem results from the application ofthe principle
‘onseroaton ofenergy. This equationmaybesummarizedas follows:

Energy Equation without Head Lost:


Ifthe fluid experiences no head lost in moving from section 1 to section 2th
[Neglecting head lost in fuid flow, the values that we get arecalled idea!
theoretical values. With reference to Figure 5-3:

nergy Equation with Pump:


Pump is used basically to increase the head. (Usually to raise water from a
er to a higher elevation). The input power (Pap) of the pumpis electrical
ergy and its output power (Paapw) is the low energy.
"
(Characteristics of HGL
«HGslopes downward inthe direction of flow but may rise
sure
to.changes in velocity or presion,
+ Forunifo rm pipe cross-sect HGLis parallel to the EGL- sure |
For horizontal pipes with isunif orm diameter, the drop in pres een
between any two points also equal to the head lost betw
Energy Equation with Turbine or Motor: points
whi in
mechanical work work which
‘Turbines or motors ‘extract flow energy to do jo mechanical
converted into electrical energy for turbines, 4 nergy Grade Line (EGL)
energy of fly (he
representation ofthe total datum
Energy grade line is graphical from the planes
ante Kinetic and potential energies. Its distance
e
2

Characteristics of EGL of flow, and it will only rise


‘with the presence ofa pum .
«The dro p of the EGL bet wee n any two points is the head lost between
those points. e cross-section, EGL is parallel tothe HGL
«+ For uniformpip
‘© EGILis always above the HGL by an amount equal to the
velocity head,
3/2g.
+ Neglecting head loss, EGL is horizontal.

ENERGY AND HYDRAULIC GRADE LINES


Hydraulic Grade Line (HGL)
Iydrailic grade line is the graphi
‘Also known as. pressure gradient,energy
of the total potential of flow. It s the line thatcc
the water levels in successive piezometer tubes placed atintervals along :
pipe. Ils dista p
: nce fom thedatum planes P+ hyaraucarade Ines.
Figure: 5-7; Tustin showing the behavior of energy and
9, CHAPTER FIVE
Fundamentals ofFuid Flow

Wnty on wwe
ijnVolune
aa Taare $00 iTa
$ (0.075) (05) = § (0025) W=192625N
9 45.n/6 (velocity ofthe jet) PE=1,92627

Power, P= QyE
Ay. I7 ;
wi fhrougha50-mm diameter nozzle
2g 20981) ‘Solution
BOAT my Kinetic energy flux = Kinetic Energy per second = Power
Power, P = 0,002200(9,810)(1.0321) : Power, P= QyE
Power, P= 22.37 watts (poweravailable inthe jet)

‘A turbine is rated at 600 hp when the flow of water through it is 0.61 m’


‘Assumingan efficiency of 87%, what isthe head acting on the turbine?
p= 10.186 m/s
‘Solution pm 10186)2 5256
Given: Power output = 600 hp 2981)
Bien, =67% = 0.029610 x 0.85)(5.288)
pen eaetl
00 699.655 hp
Power input 2.
Power input = 514483 watts to what height a vertical jet of ‘waterwater could
Neglecting air resistance, determine m/s?
Power input = Qy HE rise if projected with a velocity of 20
514483 = 0.61(9,810)HE
HE=8597m

Problem 5-7
A standpipe 5 m in diameter and 10 m high is filled with water. Calculate
potential energy of the water ifthe elevation datum is taken 2 m below the
ofthe standpipe.
pipe carrying oll of specific gravity 0.877 changes in size from
tion 1 and 450 mm at section 2. Section 1 is 3.6 m below section
jessures are 90 kPa and 60 kPa respectively. If the discharge is 150
termine the head lost and the direction of flow.
lution
Q1=Q.= 015 m/s
05 = =849 m/s
F015)
eae015
“Teking @ as datum: 2 eee
rn fow
Watersdownflowing in an open channel ata depth of 2m and a velocity of 3 m/s, pate
fn oe lags Sa
°°" a@an @aixom™ *°
flows a chute into another channel where the depth is 1m and th Fy 14.135
velocity is 10 m/s,
Papeete e ees
Solution

PL
0.9437 Laer oO
(0.81«0877) 2

Since Ey > By the flow is from/to2


‘Head Lost, HL = Ey ~ Ez = 14.135 - 10.62
Head Lost, HL = 3515 m

[Neglecting fiction (headlost):


Eis Ey
ue
ee oe
Wan eee
zon *?** a
| Problem 5-12
| Ol flows from a tank through 150 m of PA(0303)0)
180 min diameter pipe
dlscharges into air as shown“andin then
the
ceeLge 0575
anedenysteon2)
Figure. Ifthe head loss fom point 149
Point 2 is 600 mm, determine the
Pressure needed at point
lt/se ofol flow. 10 cause 17 6 29m
Solution capes twough«s0-mam-iameter vert. Calculate didandthe ely of at
Q=0.017 m'/s fostough he vent. Assume the fw tobeehincompressible
Energy equation between © and @:
Ei2 Hlaa™ Ey 2
ten tine B+ Bee
oY 2
O* ie +20-06=
2
S001)
a g(015)" 9439
FY'=10.65 m ofoil
y
pr 10.65.81 « 084)
i= 87.76 kPa

Gas is flowing throu igh a square conduit whose section gradually changes
150 mm (section 1) to 300 mum (section 2), Atsection 1,the "Assuming the flow to be incompressible:
'm/s and the density of gas is 1 kg/m? while atsection 2 the velocit
velocit
y of flow
y of flow is
‘m/s. Calculate the mass flow rate and the density ofthe gas at section 2. Foostes)= F (0.057) + $ 0.065)43.5) + §(0.6)Rah/at
a h/t = 0.0553 ays
Considering the air abovethe tank:
[t= Qu
i $10.05 )04 = § (067 dh/at
rey a a8)
F05)F04 = § (0.670.055)
‘4= 7.963 ays (velocity ofair flow)
ee M=pAw
= 11(0.15)0015)])
M=04575 kg/sec (mass low rate)
ee
(CHAPTER FIVE
Fundamentals of Fluid Flow
AA liquid having sp. gr. of 2.0is flowing in a 50 mm diameter pipe. The Q1= Q.= 003 m/s
‘head a given point was foundto be 17.5 Joule per Newton. ‘The elevation
the pipe above the datum 3 m and the pressure in the pipe a 8g?
is 65.6
Compute the velocity offlow and the horsepower inthestream at that point 2g” 3gD
Solution Be, 4008)" 5 9,0365 m
2g we8i02)"
Totalenergy, E= 22 +B az MOE ota7 m
1
£=175oule/Na(1N-m/loue (045)'
2g 77(9.81)
Ja z Equation
Energy Equation betweenA and B:
Eq~HLja+HA-HLis™ Ea
v0
ea
ee 2)
BC
ae
P aya HA Hann BE + BE
: - 10(0.147) = 0+ 0+ 60
0+ 0+ 10- 2(0.0465)+ HA
a ii
¥=14.7 9 mysn (velocity of flow) pe es
ae 015150)
roweroutput=Qy HA = 00381065 6)
= [5 (0.05)104.79) x (0810 x2) «175 Power output = 20.horsepower (rated power ofthe pump)
"= 9970.92 watts» (1 hp/746 watts)
Power, P= 13.37 hp Pressure heads at 1 and 2:
‘nergy Equation between A and:
E4-HLaa= Ey 5
bien 16 (CE Way 1994)
‘The pump shown draws water from reservoir A at elevation 10 m and lifts it t 1d PA ego tyse Se + Bh a
reservoir B at elevation 60 m. The loss of head from A to 1 is two times Oey 28
velocity headin the 200 mm diameter pipe and the loss of head from 2 to B 0+ 04+10-2(00865) = 00165 + #1 +0
fen times the velocity head in the 150 mm diamete
horsepower of the pump and the pressure heads at1r pipe.
and
Determine the rat
2in meters when P2986 ofwater
discharge is 0.03 m/sec. ci
nom poe Energy Equation betweenof 2and B:
Ey Hse Es
2
S242 +-Hhas= + vst PB ss,
2g, 2 ii
047+ P2 +0-10(0.187)=0+0+ 60
y
P2. = 61,323 m of water
Y
Problem 5 - 17 (CE November 1986) j
‘A pipeline with a pump leads to a nozzle as shown. Find the flow rate
Pump develops an 80 ft (244 m) head. Assume headlost in the 6-inch (152:
Pipe tobe five times its velocity head while the head lst in the inch (102 80" ass
Pipe to be twelve time its velocity head. (a) Compute the flow rate, (1) ska ae
the energy gradeline and hydraulic grade line, and (c) find the pressure head 0+0+21.3-773.96 Q' + 24.4 -9160.13Q = 24508 G+ 0+ 244
the suction side, 1238489 = 213
amya440) = 00415 mys > Discharge
onitly (0) nergy and Hydraulic grade tines
aned
2, M OON 246m
x? (9.81)(0.152)'
= 20.0815" astm
25” Femnaxmt ">
a T2 hi(0.08
e 15)?s
ae — aw as2m 2g 7(9.81)(0.0762)*
73.96 #133 m
Solution
(@) Discharge
Q=Q=QW=Q
Energy Equation between A and B:
Eq Hly+ HA-Hla™ Es
SA> + PA 424 Hl +HA-Hla= 2 +
2g Y 23
HA=244m
Hy=5 2
2g 25805
ED,
19
Foss
HL,=77396
az 2
Hla = =1 1222 = Bae22
2
~? Fesei?
Energy Equation between 1 and 2
nergy Hls=
Equation between A and S (neglecting head lost and taking point 2 as datum)
Ex Er E.-HE=B:
Ph
Pa ayn
Teas Z
ast Be
78h e + n npn Mo
Phan wh 1 Pe
Pe
at 2 oY *
oF
Fy 70286 m 057, sag pe ge A 0
Hesnos* 981
x081(06)* 9.81
0+04213-133~0266+ 2S +152
Y He= 967m
PS < 450m Power, P= Qy HE
7 = 0.5(9810)(3.647)
= 17,8885 watts « (Ihp/746 watts)
(Or from the figure shown above, the pressure head at $is the vertical dista Power, P= 23.98 horsepower
from the pipe to the HGL.
PS 0 19.704-152 5-19
'A.20-hp suction pump operating at 70% efficiency draws water from a suction
mm ratareter on
diameter is 150 mm. ‘The velocity in the 150 mm line is 36 m/s. IftheApressure
‘at point A in the suction pipe is 34 kPa below the atmosphere, where is 1.8.m
elow B on the 150 mm line, determine the maximum elevation above B to
ater enters « motor through a 600-mmdiameter pipe under a pressure of
It leaves through a 900-mm-<diameter exhaust pipe with a pressure o lution
‘kPa. A vertical distance of 25 m separates the centers of the two pipes at & m=36m/s~ 0
‘sections where the pressures are measured. 1f 500 liters of water pass the moto
‘each second, compute the power supplied to the motor. 1
ve 66m
a4
Solution Qa= §(0.153.6)
1410 (Qe 0.0636 m/s
Q= Qr= Q:* 0.0636 m’/s
man 0.0696
40027
na 01 =2025m/s
25m

we
a 2° 021m
aaah
Q=Q:=05 m/s
Powerosper™ Qy HA
ExHle=Hly-Hlan=HLn=Ee
DAE 5 PA 5 xy-3-2-10- 004m PM 4 PH 4 2y
ar Zante. y
SA2 2 BE 4c 2 2
Og + PAtoh tae
tet HA-HL=
HA-HL= 2
SE 4 Poy Pa 10 4 PY oy
= PA og 2a 15 15 = 10428
4 +0+2591-3=066+0+08+h)
O05" 40416
021+ SF ar + 40-15 550 -10,2 8g
= 15.19 m, Fosias sat °° Fon)
Q=000106745m/s
ge Qu 0106745
Ay 70005)"
ua 27s
and 16 m above the pump. Assuming frictional losses
the hydrant, 2m in the hydrant, 10 m from the hydrant ofto 3thembase
from the p
ofthe noza og Te
2g” 2981)
= meTOT
‘Solution
For the pipe shown in the Figure 0, = »;= 1.2 m/s. Determine the total head lost
between I and 2,
wam=12m/s
Energy equation between 1 and 2:
B-HL=Ey
2 2 B
Mey
gy tee ape
221s 900 mm, find the
Since oy =r pressureat point 2
Dense Pan Solution
7 1 Qi = Q: = 0.056 m?/s
280
2804 200
r 49-Ht= 224,
oar aa ‘Energy equation between @ and ©:
HL = 3.375 m Ey-Hlia™
2
4 Peay Hl
2g
waterjet atthe rate of 10m/s, Neglecti
jet a the highest pointofthe projectile? sacra ~Ss2m ool
Solution p= 45753 kPa
Solving forthe velocity oftheetatthesummit (highest point,A)
90
lem5 24
‘A So-mm diameter siphon dischargesheadoll (sp. gr. = 0.82) from a reservoir (elev.
open air (lev. 15
20 ‘m)into open m), “The eae
loss from the reservoir (point 1)to the
yew

Since the flow is continuous:


[Qo= Qa}
‘Ao ™ AaD
$ (0.05) (10) = An 6),
‘n= 0.003927 mt
ter pipe shown (@) assuming
head loss and (#) considering a
3 ostof200 man,

lution
(a) Assuming no head loss:
Energy equation between 1 and 3: Energy equation between ae
Ey~HLia-HLay = E © and © noglecting head lost:
BL2 4 PL 4 2y-Hlsa-Hlay = 22 + PDs Eek:
2 2
2 oY ae ES of >:
2 My Aye Ms Bee:
0+0+20-15-24= 82° so+15 ag oy ag eae
WOODY 0+0+20= 2 +0+249
Q= 0.00912 m/s 2g
Qm 9.12 liysec
Energy equation between 1 and 2:
22 sam
2g
Ej-Hlia™ Ba
2 Bact oe
Bey 2 oe
= 10 m/s
QrArnom $ (015710)
ag oy $227, Q= 0177 m’/s=17Ys
(0.0O
0912)
0+0+29-15~ M
3(005)"O7 981xPa082 +2 (W) Considering head loss of 0.2m:
Fy-HL= Ea
pr=-37kPa 2 Ben
M2 Pain +
‘Absolute pressure at = 1013+ (37) 2 2g
‘Absolute pressure at © = 643 kPa 2
0+0+30-02= 2 +0+249
2g
2
2 249m
2g
= 9805m/s
Q=Armr= $ (1510805)
(CHAPTER FIVE 271
Fundamentals of Fluid How
Water flows frely from the reservoir shown throu 50-mm diameter pip
the rate of6.31 it/sec. Ifthe head lost in the systemighisa11.58Jo ule/N, detern
the elevation of the water surface in the reservoir ifthe discharg e end {a
elevation 4m. flowing inthe 75mm diameter pipe
150.6 m/s.

11 =06m/s
| [=Qa
t $F (0.075106) = (0.025)
m=54m/s

Bee
|
ete,
s
HL» 11.58 N-m/N=1158.m oaega between @ and @:

BE ee Bae
Energy equation between @ and ©; 2% 28 a
ese 062, Fty yon 2081)SH so424
ey Bene, ae 28)
2" y 2 y Ph 3,868 m of water
00+ 2-1155= 0M?
170.05) 5 9,4 i
21 = 16.11 m > Elevation of w.in the tank ‘Summing-uppressurehead from
BL rars-mnag=
3.868 + 0.75 - 13.6h= 0
hh= 0.3995 m= 3395 mm

172 Fundamentals of Fuid Flow
= lution
| ‘A‘mim
Atel horizontal pipe gradually reduces from 300 mm diameter section to 100 i
epathemesonokon
section is 70 kPa. If the flow rates 15 Iters/sec of water, compute the
peSreepee
i) Energy equation between 1 &3
hongplat3)
Jost between the two sections. ee
"Solution Ob Pine Bie
ig Po tae
Qe 0.015my
0 15 ms — 100 mm 0+0+10= 522 +0+0
= ee y= 1d m/s i
Lashed | Q= Qs $(0.225)414)
Qi=Q.= 0.015 m/s Q= 0857 mys
Energyequation between @ and (t) Pressure at the throat
fear nergy equation between @ and ©:
Brg Ss Bey
gy ee
+0+0

possible rate of discharge through this tube, and (b)the corresponding pr


atthe throat.
Ans:-9 mm/s
the mean velocity of the ais?
Fluid having sp.gr, 0.88 enters the cylindrical arrangement shownin Figure 5 =
10 at section A, at 0.16 N/s, The 80-mm-diameter plates axe 3 mm apart.
Problem 5-31 Assuming steady flow, determine the average velocity at section A and. at
‘The piston of a hypodermic apparatus shown in Figure 5 - ‘ection B, Assume radial low at B.
‘at 6 mm/sec;air leaks around the piston at 20 mm’/sec. Whatis the a ‘Ans: 01 = 147 m/s; 02" 246 em/s
‘speed of blood flow in the needle?
Ans: 498 mm/s
a3

Figure 5-8

i Figure 5-10

Problem 5-35
‘The water tank in Figure 5 - 9 is being filled Uwough section 1 at 6 m/s an If ajetis inclined upward 30° from the horizontal, what must be its velocity to
through section 3 at 15 L/s. If water level h is constant, determine the reach over a3-m wall ata horizontal distance of 18 m, neglecting friction?
‘Ans: 16.98 m/s
velocity 0
Ans: 7.97)
5s Problem 5-36
ofe [Neglecting air resistance, determin e the height a vertical jt of water will ie if
“ projected with velocity of 21 m/s?
‘Ans: 225 m.

09mo—
(CHAPTER FIVE
Fundamentals of Fluid Flow

High velocity water flows upan inclined plane,as shownin Figure 5-11.
are the two possible depth of flow at seetion 2? Neglect a losses
‘Ans: 0.775 m &
Chapter 6
| Fluid Flow Measurement
There are numerous number of devices used to measure the flow offluids. andIn
fany of these devices, the Bemouli’s Energy Theorem is greatly utilized
Figure 5-14 Ixiditional knowledge of the characteristics and coefficient s of each device is,

observation, usually by
“The actual discharge may be accomplished by series of the
‘measuring the total amount of fluid passing through device for a known
period. The theoretical value can be accomp lished using the Bernoulli's
‘Theorem neglecting losses.

Coefficient of Velocity, C,
‘The coefficient of velocity is the rato of the actual mean velocity to the ideal or
‘theoretical velocity which would occur without losses.
any
& Fluid Flow.

:
|
2
Coeficets for Vertical Edged Crear ©
Te concent of onsacton the i fhe acl are fhe on
_ Coefficient of Contraction, C.

eo nr oe ig ah wc he
“Table 6-4: Discharge
Deecharging nt Ar a15.6C (60"F)
‘Head ‘Diameter In mm
7 fm) &25_| 1250 16.75 [25.00 50.00 7
ozs} oen7 07 0.616 0.609 ~o.601 0.603 E
‘043 0635 |_0.619_|_o.610_|0.605 ‘0.603 0.60010:
a BT ST ‘0.500 0s98_| 0397
iz] oat 0609 [0597 "0596
ia [0617 007 ‘0.599
24 [0614 0.605 ‘0.558 0596 0
Relationship between the Three Coefficients 305 [06130004 0.597 0596 0595:
[36 oi 0.603 (0.597 0.595 0.595 |
‘Actual discharge, Q=CxQ, > Bq (1) 427 [ost 0.603. (05960595 0.584
458 ‘0.610 0.602 (0.596 0.595 0.594 #]
a 0.596 [0395 [0594 f
pie‘Q= Actual area, ax Actual velocity, » 762 0.08 0.501 05s6—[—ose [ose |
915 —|_0.607_|- 0.00. ai
Q=GAx Gu, 1220 [0.606 0.600 03595 |
- 1524 0.605 [0598 0595 os94 0.583 4
eee 1830 00s 0599 ‘so 0593 [0583
but Av=Q1
= GG) FH @
From Equations (1) and @)
HEAD LOST
‘The head lost through Venturi meters, orifices, tubes, and nozzles may be
“The coefficientofdischarge varies withReynolds Number.Its not constant expressed as:
‘given device. Table 6 1 gives thecoefficients for vertical sharpedged orifice.

eno coat0
csc86 en
ar jen
‘Tack late owns
Figure 6 4: Orie cements
2
(42) Ses one EB ae
2
A) 39 7

(-(a(ees) the orifice or nozzle takes off directly from a tank where
er than Ay, thenthe velocity ofapproach is negligible and
Ay is very mutch
Eq. 6 5 reduces

ome Forgas) C'>)| 20


2
7
Consideringhead
- lost beten 1 and2
22 He Het ee se
ay oY
‘This equation simplifies to:
“An orice is an opening (usually circular) with a closed perimeter through which
upstream face of the orifice may be roundedlengthof or sharp. An orifice with
prolonged side, such asa pieceof pipe, having atubes suchtwoas. orculvert three times its
Since Yauai™ Ce Pomerat ™ 2 ameter, is called a short tube, Longer s under
e althoug h they may also be treated as
rear ee)
fembankments areusua lly teated asorific
shortpipes.
‘shape, orifice may be circularis, most square, or rectangular in cross:
Squaring both sides and arranging terms: incular sharp-crested orifice widely used because of the
simplicity ofitsdesign andconstruction.
“The figure below shows a general case of fluid flowB,respect through an orifice. Let px
and ps be the air pressure s in the chamber s A and ively and vs be the
‘locity ofthe stream normal to the p l a n e of the orifice (velocity of approach).
Consider two points 1 and 2 such that = v4 and ex = and writing theenergy.
equation between these two points neglecti ng losses:
orifice, there is an appreciable difference between the discharges|
previous analysis
‘Consider the rectangular section oflength L and height D as shown in the
with both the surface and the jet subject to atmospheric pressure:
theoretical discharge through an elementary strip oflength L and heightdh is

dQ. (Lal gh
= LWr a
Contraction of the Jet oa by
The figure shown represents a cross-section of fluid flow through a = +
sharp-edged orifice fom a reservoir to the atmogphere, "The fluid fl See ae
| they cannot make an abrupt change in their direction and move
Talc pt Sedase Sees set dpspad a= fret [2] yt
orifice. The phenomenon is referred to as the contraction of the jet. The ft
omSpot
thejt where the contacon cassis called the Ye conrad wi
|Ree Toate ncn hai ef ee anc ©) fom Que § PRL (Herm)
once

VENTURE METER
‘Veturi meter is aninstrument used in measuring thedischarge through pipes. It
consist ofa converging tubeAB (See Figure6-3) which isconnected to the main
and
pipeatthe inletatA,and endingina cylindrical section BCcalled the throat,outlet
' diverging section CD which is connected again to the main pipe atthe
D. The angle of divergence is kept small to reduce the head lost cause by
turbulence asthe velocity is reduced.
eo ere te
‘S
BO CHAP TER six
ris ion Measurement
Gu. “The theoretical or ideal discharge “Q,” can be found once v: oFvis KNOWN:
factual discharge "Q” is computed by multiplyi ng the theoretical value by
wile oefcient ofdisclarge or meter coefficient "C.
QreuginsDeena
Note: 1f we negect the head lst inal our energy equation, the values we get are Kno
theoretical or Ide! values (theoretic velocy and theoretic al dscharge) . Consider ing head
loss we get the acta values(ecualveloand actualdischarge)

NOZZLE
‘A nozale is a converging tube installed at theet.end of a pipe or hose for the
‘purpose of increasing the velocity ofthe issuing

Consider two pointsin the system,


system, @ al the base ofthe ininlet
let and © atthe throat
‘and waiting the energy equation between these 0 points neglecting head
sleet+P ge ne eg Be

bes)
25 re
where:
He total head at base of nozzle
‘Aq areaat the nozzle tip
‘Theleft side of the equation fs the kinetic energy which shows an increas
valu while the lat sie of the equation s the potential energy which shows
decrease in value, Therefor, neglecting head le ters coefficients for water discharging
~The following tablegives the mean valuesof of40mm
in netic energy ‘through a nozzle having a base diameter and C.= 1.0.
gual he eras in potentienergy. This sateen Known a the Ven
nciple
‘The difference in pressure sure between the inlet and the throat oatisis common y TipDiameterinmm 39 22 25 29
meaoured by means ofdiferentmanometercomecingth letand throat Cu 0.983 0.962 0900 0976 o97s | o9s9
ations and the difference in pressurebetween and @ know
discharge can be solved, e oe
‘The head lost through a nozzle is given by Eq. 6-5.
‘Named after the French physicist and engineer Henri Pitot, Pitottube is a b
| Cahaped or U-shaped) tube with Both ends open and is used to measure
‘locity offluid flow orvelocity ofair low as used in airplanespee dometer,

then established, and the quantity of liquid in the tube remains un


the flow remains steady. Point 2 at the face of the tube facing the stream,
called the stagnation point.

Figure 6 = 6:Pct ube na poe

‘This equation shows thatthe head at 1is transformed into pressure head at 2.
Bb oping a sno
iae ye te recast contd ap
prinnn
fet nt ne nvscalearterets are be bt
‘its coefficient of discharge varies widely. ae

“The following illustrations show the two different flow conditions throu
sluice gate, eee where:
‘C= CC (varies from 0.61 to 0:91)
Any
‘b= width of the flume

‘Tunes
Standard ShortTube
entrance and has a
one with a square-corneredFigure
‘A standard short tube is theinternal
length ofabout 25 times its diameter as shown in 6 -8. Figure 6
TB (a) shows a condition when Taw suctenty ih hende-10
Four 6 7 (0): Fre Flow Figure 6 7 (): Submerged ow pipe. This condition
thatthe jet may not touchthe walls oftheFigure is very much the
same as that ofa sharp-crested orifice. 6 - 8 (b)shows a condition when
Figure 6-7: Flow trough a gate The discharge through this tube is about
the jet touches the walls ofthe tube,
JB Sine 7 wring emery iret a2 meine fonethird greater than that ofthe standard sharp-ecige orifice butthe velocity
‘of flow is lesser.
Bee
22 : pi vea ye
g 7 2g Y
where Pt = dy and 22. = dy
Y Y
2
dt 2 +d +0
2g

gue 5-8 (2)


Figure 6-8: Standard Short tubes
Converging Tubes Re-entrant Tubes
‘Conical converging tubes has the form of a ‘These are tube having their ends projecting inside a reservoir or tank.
frustum of a right circular cone with the
larger end adjacent to the tank or reservoir
‘as shown in Figure 6-9.

Borda's Mouthpiece - This is a special case of a re-entrant tube,


‘consisting of a thin tube projecting into a tank having. length of
‘aboutone diameter. The coefficient of contraction forthis tube,
C= 05 and C= 1.0.
Submerged Tubes
i example of submerged abe
embankments, The discharge through sabmerged tubeis piven by
formula:
0.629 Gait 0947 0.965 0971 0973 O76 O98T| 0:
oats 0.888 0.
a
a
§
§
§
§
3

0.829 0.910 0939 0.938 0924 o911 0.096 0871


Where C isthe coefficient of discharge, is the area of the opening, and H is the

Diverging Tubes
AA diverging tube has the form of a frustum of right circular cone with
‘smallerend adjacent tothe reservoir or tank.
‘The flow through orifice, weirs, or tubesis said to be steady only if the tot
producing flow,H, is constant. Theamountof fuid being discharged fl
te computed usingtheformula

where Q is the discharge, whichis constant orsteady. In some cond When there is no inflow (Qn 0)the formula becomes:
however, the head over an orifice, tube or weir may vary as the fluid flows A, dh
and thuscausing the low tobe unsteady = Qe
Interchanging the limits to change the sign oftheintegrand:

Note: If, is variable, it must be expressed in terms of

the outflow is through and orifices or tube, Qu= CA /2gH1. If the flow is
{through anyother openings, use the corresponding formula for discharge.
tanks with constant cross-
ynal area and the outflow is
ugh an orifice or tube (with no
Mal
Consider the tank shown in the figure to be supplied with a fluid (inflow) ar ow), the time for the head to
fr?
>
simultaneously discharging through an outlet (either an orifice, tube, weir change from H;to His:

‘il
pipe). Obviously,if Qn > Qau the headwilliseand ifQu > Qn the head
fall, Suppose we are required to compute time to lower the level from hy
a (assumingQua > Qu), the amountoffluid whichis lost in the tank will be
AV = (Qu Qu)
ate
Pao
ote firea sD
where dV is the differential volume lost over a differential time dt. If the
over the outlet is , then the level will drop dh,thus dV = A, dlc, where A, is
surface area in thereservoir atany instant and maybe constant or variable,
aes
WEIR
Weirs are overflow structures which are built across an open channel for |
flow of liquids. Weits have
[purpose of measuring or controlling thewater,
permmonly used to measure the flow of but i is now being adopted:
1gud ows though «suber measure the flow of other general,i, applicable toandanyprinciptes
liquids, "The formulas that will
wn, the time forthe head tochange from H; to Hs is: the derivation discussed onthischapter are typeofliquid.
these formulas in Problem6-28) paces pares
Classification of Weirs
‘According to shape, weirs may be rectangular, triangular, usedtrapezoidal, crcl
The most commonly
‘porabolic, or ofany other regular form, shapes. According, to the shapes are the
fectangular,triangular and the trapezoidal form ofthe.
crest, weirs may be sharp-crested or broud-crested.
flow over a weir may ether water surface
be fice oF submerged. Iftheis fie
Drenatream from the weit i Tower than the crest the flow but if the

Definition ofTerms
[Nappe-the overflowing stream in a weir.
Crest ofweir the edge or top surface of a weir with which the flowing liquid
‘comes in contact,
Contracted weir - weirs having sides sharp-edged, soeither that the nappe is
‘contract ed in width or having end contracti ons, one end or two
ends.
‘Suppressed weirorfull-width wer - weits having its length L being equal to the
whereAn andAais thewater surfaceareas inthe tanks at any time, and His ‘width ofthe channel sothatth e nappe suffers no end contractions,
difference in water surfaces in the two tanks atany time. If An and/or Aa Drop-down curve - the downward curvature of the liquid surface before the
vary itmustbeexpressed in termsofH. weir.
IfAn and Aq are constant, ie.the to tanks have Wve uniform uniform cross-sectional
ue Head, - the distance between the liquid surface and the crest of the welt,
H
‘measured before the drop-town curve,
fio ve is,jan
om fae [2e+1)|
b
r= & LOg LL (H+ Hy)(0 HY]
Actual Q=CQ,

es. common practice to combine 3 2 C into a singe coefficient. called


the weir factor, The general formula for a discharge through a rectangular weir
considering velocity of approach then becomes
Figure 6 - 12: Patines of flow over 3 rectangular sharpcrested weir

becomes very.
If the ratio H/P is sufficiently small, the velocity of approach becomes
RECTANGULAR WEIR small and the term hie/? may be neglected. The discharge formula

In situations where the discharge is required considering the velocity of


approach, using Eq,6 - 30 or Eq, 6 - 31 would lead to successive trials to solve
for Q (since the velocity of approach fy is a function of Q). The following,
t simplified equation may be used:
Figure6 - 13: Secon ra of Figure 6-12,
Consider a differential area of length Land height dh to be located hm
below the liquid surface. By orifice theory, the theoretical velocity through
area is:
om 2g
where the total head producing flow H = h+ Iwhere fis the velocity head
spyoach and is equal 12/2. Te daharg through he eemenay sp where d= depth of water upstream,
d=H+P
“dQ. dho,
40-1dk 280)
BAZIN FORMULA
For rectangular weirs of length from 05 m to 20 m under heads from 50 mm
plowing specifications must be applied to a standard rectangular t0.600mm.
tend contractions:
4, The upstream face of the weir plate should be vertical and smooth.
2, The crestedge shall be level, shall have a square upstream comer, and.
shall be narrow thatthe water will not touchit again after passing the
‘upstream corner.
43, The sides of the flume shall be verticaland smooth and shallextend a
‘short distance downstream past the weir crest
4, The pressure under the nappe shall be atmospheric. “The effective length of L ofa contracted wei given by:
5, ‘The approach channel shall be of uniform cross section for a sufficient
distance above the weir, or shall be provided with baffles that a
normal distribution of velocities exists in the flow approaching the
‘weir,and the water surface is Free of waves orsurges. I
where L’= measure length of crest
N= number of end contraction (1 or 2)
H= measured head
‘Standard Welr Factor (C,) Formulas
TOT
Gtobe used in Eq, 6~81 and Bq, 6- 32. Some ofthese are given below.
FRANCIS FORMULA,
‘Based uponexperiments on rectangular weirs from 1.07 m (35 ft)to 5.18
(17ft) long under heads from 180 mm to 490 mm.

weir has a tendency toa adhere to


[At very low heads, the%nappe of a rectangularsuch
its downstream face. A weir operating under condition would give a very
inaccurate result. For very low heads, avertexV-notch weir should be used is
laccuracy of measurement is required. The angle @ of a V-notch weir is
usually between 10° to 90° but rarely larger.
a= Eg (uu ?)-34°7]-0}
o> Fe (4H)

Qf y2g LRA (theoretical Q)


Actual Q= Cx Q,

ig, 6-42 can be used evenifthe side inclinations are unequal


For triangular V-notch weit,
Flgure 6 14: Tanga(Voth)wr
‘The discharge through the differential stripis: L=2H tan (0/2)
dQ=0dA then,
v= gh (neglecting velocity ofapproach) > HC Pa TH tanO72I) =
A= x dh
by similar
ee tangles:
ina
x EG) For standard 90° weir:
L myan
t= EGG
4Q> fish ©-yah
d= Eg WG-1) ae
4Q= Ee (2-10)a
Integrate
Q- $i "Wena
3
=bw?)-ae2t
‘TRAPEZOIDAL SHARP CRESTED WEIR
‘The discharge from a taperidl wee assumed the same
‘rectangu may be noted that, in a rectangular weir, discharge varies with 3/2 power of
lar weir and triangular weir in combination. aes
‘and,in a triangular weir, with 5/2 power of H. There exist a shape for which
ES discharge varies linearly with the head, the proportional flo ox Retiger welt,
o knownas Suttro weir.

CIPOLLETTE WEIR
Gipollett
Gro i weirs arere trapezoidal weirs weirs withwith side side sh slope of 1 horizontal to. ‘SUBMERGED SHARP WEIR
ettical. The additional area at the sides adds approximately enough efectiy discharge over a submerged sharp-crested weir is affected not only by the
‘width ofthe stream to offset the side contractions. hpead on the upstream side Hbut by the head downstream merge H:. The discha rge
for a submerged weir is related to the equati free or unsub d discha rge.
Villemonte expressed this relationship by the on
4sthe exponent of Hin theequation for free discharge forthe shape
‘used. Forrectangular weir, =3/2and n =5/2 fortriangular wei,
‘volumetric tank 1.20 m in diameter and 1.50 m high was filled with o
minutes and 324 seconds. What is theaverage discharge?
Solution
Discharge, Q= Volume
time
Diecharge geTo £02705)
16
Discharge, Q= 0.1025 m/min = 1025 liyemin

‘A weigh tank receives 7.65 kg of liq uid having 5p. grof 0.86 In 149 seconds.
Whatthe lw rte in ters minute?
Solution
Mass flow rate, M= 2
Mass flow rate, M = pQ
|e low is through a suppressed rectangular weir: 0.5134 » (1000 x 0.86)
[Aaa Q=597% 104 m/s
te Q= 0597 L/sec = 35.82 liymin
I, CLS
oeoAewe ts t
(Poec 4 o fat) a
eh dn, M Calculate the discharge in liters per second through a 100-mm diameter orifice
under a head of 5.5 m of water, Assume C, = 0.61 and C, = 0:9.
Solution
ce eae
o-caaa
CAGsG- Asin Ome
Lscrestlength
4cont raruncer revo tak 9xH=55m
055 4 0007OBVES)
Fie finalhead Q= 001877 m/s = 48.77 1/s
Problem 6 - 4
‘An orifice has a coefficient of discharge of 0.62 and a coefficie
0.63, Determine the coefficient of velocity for the orifice. nt ofcontraction Q= cA igh
sion 082
Het 925d 615982
fate z H+ 3.487 m of glycerin,
ee Q= 065% $ 0125) f2@R1)487)
= 0.066 mys
rroblem 6-5
Calculate the discharge trough the 10mm diameter orfce shown. Assume G
[ssa [PobaneaGe;
‘The discharge through a 75-mm diameter orifice at the bottom of a large tank
was measured be 1,734 liters in 1 minute. If the head over the orifice remain.
constant at 5.5 m, compute the coefficient of discharge.

cw 2 Q
Ca
‘Since the head is constant, the flow is steady,thus;
Vol _ 12/1000
Solution T(60)
Q= CA ag (= 0.0289 m/s
1 Hop Hoenn r= A fig
Hoos IS r= $0075) ZOE)
11 = 6568 m Qr= 0.01589 m/s
Q=062x 4 (014) /ZORIESE5) r
= 0:108 mys

‘An open cylindrical tank, 24 m in diameter and 6 m tall has 1 m of glycerin (S; = ‘A calibration test of a 125-mm-diameter circular sharp-edged orifice in a
15), 25 m of water, and 15 m of oil (S, = 082). Determine the discharge vertical side ofa large tank showed a discharge of 590 N of water in 61 seconds
through the 125 mm diameter located at the bottom ofthe tank. Assume C= ata constant head of 4.70 m. Measurement ofthe jet showed that it traveled 2.35
‘m horizontally while dropping 300 mm. Compute the three orifice coefficients,
ws Contraction,
ae Gs
‘Theoretical values:
w= J2gH = (208147) 0.631 = C.x 0.989
1/= 9.608 m/s 0 0.688
i= Av.= § (00125) (2.608) eae
Q,=0.001178 m/s a oan
hea slaes fe ee ye ‘A 50-mm diameter circular sharp-edged orifice atthe side ofa tank discharges
and the
‘Actual discharge: ois Water under a head of 3 m. If the coefficient of contraction C, =of0:63
velocity Ce
Vol head lost is 240 mm, Compute the discharge and the coefficients
2 Fime (eady flow) ‘and discharge
W320. Solution
a Energy eq, between 1 and 2: o
Vol = 0.0601 m*
00601
= 2961 2 Bs
Ej\-HL
es 2 Be
Bh
ay Byrpale rier gator am
Q= 0.000743 m/s
2 =v
‘Retaalveloaiy UFUF SUNS SOFT 50mm oO
x= 235m 2g
on 3
y=03m rem
2 /' > actual or real velocit iy
= xtano- nd-
yoxta 8% v= 7.359 m/s

981(2.35)?
= uno. 29O ‘Theoretical velocity:
-03=235 v= {git = (2080)
=9502m/s 14=7.672m/s. > theoretical orideal velocity
Coefficients: Coefficientof velociy, Go= 2
Velocity, Cy = 2 os 2
Res Coefcent ofvelocity, C= 22% = 0988
vel ‘locity, Co aoa
3603 0.989
Or, using Eq. 6-6:
Discharge, c= 2.
°
2
“3SOO7E o6a1 a le
a. %
Discharge, C= _ 8.000743
‘Contraction, C:
0959 > coefficient of velocity When the orifice is opened,the barge
will sink a volume equal to the
C= Cx G= 063 «0.959 volume of water inside the barge.
C= 0604 > coefficient of discharge Since the cross-sectional area of the
barge is constant ancl its thickness is
repligible, the barge will sink to a
depth equal to the depth of water
C= 06calis
withcylindri that goes in. Thus the hend over the Soxtom
located mun ofsquare,
‘An orificeon ofone80 side a closed
Pe =-03 mt9 orifice, being submerged, is kept
tank as shown. An open. mercury constant at 1.5 m.
‘manometer indicates a pressure head of 300 ‘The barge will sink to its top
‘mm Hg in 4them airof atthethetanktopisofoilthe6p.tank.gr. =If
the upper when x= 05 m.
(080) and the remainder is water, determine 4
the discharge through the orifice. Volume = Qt
Volume = 5(10)(05) *
solution Vounes3ay
Q= cA \2gH peel
H=4+408)+(03)036)
nuaitmawaer 0-06 § 0197 268K)
Q=06x 005y /2981)8:12) (Q 0.08288 m/s
Q= 0.01173 ms 0.08283 ¢

{A steel barge, rectangular in plan, floats witha draft of 15 m. If the barge is eter
rm long, 5 m wide, and 2m deep, compute the time necessary “Calculate the discharge through a 90-mm-diam
Assume C=
‘edge after opening a standard orifice, 180 mm in diameter, in its sharpedge orifice in the igure shown.
065
Solution
Q= CA fght
H=3+ es
a1x090
H=5718m
= 0.65 x $ (0.090)? /298I(E718)
= 0.0838 m'/s = 43.8 1/s
Problem 6 - 13 (CE May 2001)
Water flows through an orifice at the vertical side of a large tank H=145=1.968
‘constant head of 24 m. How far horizontally from the vena contracta will $= 0.968
jet strikes the ground 1.5 m below the orifice?
Solution
Fs ‘large closed cylindrical steel tank 4 m high with is bottom on afevel ground
rian» $e
uch conan ole of gl The oto lyr wae 2 meters dap The ap
yrs layer is occupied by a liquid whose speifc gravity isnot known, toa depth o
un Ball meter, The alr space atthe topis pressurized to 16 kPa above atmosphere, A.50-
a= JOENEA rmdiameterotfice with a coefficient of velocity of 0.98 i situated one 3.3mater
dae. from the bottom of the tank. The jet from the orifice hits the ground m
areerele fortonaly evry fom vena contact: Determine the pec Pavofthe :
2 liquid atthe top layer.
AS=xtno- —S51e
216862)? cos? Solution
19379m From the trajectory:
yextang-—& ee
Problem 6-14 (CE May 1996) aoe
A large cylindrical steel tank 4 m high with its bottom on a level grou ono a ze
contains two layers of liquid. The bottom layer is water 2 meters dep, The joo. 281052 is
Ieper W sabiby eid veci bavly nceiaan esele ear
‘meter. A 50 mm diameter orifice witha coefficient ofvelocity of 0.96 is sit pete hs
fone meter from the bot ofthe tank. The et from the ore hits the groun 2m
275 m horizontally away from the vena contacts. Determine the
sravity ofthe liquid tthe toplayer.
Solution 7.75 = 098 {2981)H
H=319m
we Cy gH Ha 141()+ oer »: 7319
+16) + Jo
H=1+1()
Hates = 056
Prom y= x tan 0-
Feycaso
= 275 tan 0" 981275)? tl
20cos
4" 6.09 m/s
609-098 (2980
Problem 6 - 16 (CE Board) (0Velocity ofthe jet as istrikes the ground:
Ajetis issued from the side ofa tankunderconstant head of3 m. The side Work-energy equationbetween O and 2
the tank hasaninclination of 1H to 1V. The total-depth of water inthe tank KEo+ Wy.= KE
2 m. Neglecting air resistance and assuming C, = 1.0, determine

(@)lowing
wWor+wree Woe
the maximum height to which thejet wil rise, z z
(thepinsk Reza plane 1.2m bow he tom of te 7.672? cd
(© the velocity ofthejet ast strike the ground zea"4°" 3g
n= 1265 m6
Solution
c 6-07. e
Determine the diameter of an orifice that permits a tank of horizontal cross-
section 1.5 m to have its liquid surface draw downatthe rate of 160 mm/s fora
3.35-m head on the orifice. Use C=0.63.
Solution
‘The discharge through the orifice is equal to the tank's cross-sectional
42) lies draw. dou
Q= Ausnk * Daeew dome
‘ 7 Q=15%016
Q=024m/s
1 Cu 2gH
24 (1) 2@8N@) =7.672:m/s [Q=CA.(29H)
(@) Maximum height(at point1, = 0) 0.24 = 0.63 $2 (25835)
From physics, D=0245m=245 mm
Wh Woy - 2gy
0= (7.672sin 45%? - 20.81) y
yi=15m > maximum height above the orifice. Problem 6-18,
‘A 75-mm-diameter orifice discharges 1.812 m? of liquid (sp. gr. = 1.07) 82.2
(©Points strike the ground (atpoint 2,yz=-49m) seconds under a 275 m head. The velocity atthe vena contracta is determined.
rom physics: by Pitot static tube with a coefficient of 1.0. The manometer liquid is acetylene
2 tetrabromide having a sp. gr. of 296 and the gage difference is 1,02, Determine
y=rtand- ax the three orifice coefficients.
Zo,cost
49 x tan 45° (981)x3" ‘Solution
2(7.672)? cos? 45° “The actual velocity of flow using Pitot static tube is given by:
0167x#?-m-49=0
42948 m > horizontal distance from theorifice Fae)
ing
‘= sp. of the gage liquid
ofthe liquid flowing

foe) ‘A closed cylindrical tank 5 m high contains 25 m of water. A 100-mm circular


orifice is situated 0.5 m from its bottom. Whatair pressure must be maintained

ae
v= 10 desc. oa(238-1) {nthe airspace in order to discharge water at 10 hp.
Solution
Power=QyE
“Theoretical velocity through the orifice:
we gH a P2ORN275)
Pomer= (go
0) 73454 m/s ie
10746) = 401) C810755
Coefficient of velocity, C= = p=1238m/s
Coefficient of velocity, Cy = 2855 v= 2H = 1238
358 H=782m
Coefficient ofvelocity, C, = 0.809
‘Theoretical discharge:
Qin As (2g = § 0.075) Y2(981)(2.75)
Qi = 0.03245 m?/s.
‘Actual discharge: concrete culvert 1.2 m in diameter and 5 m long conveys flood water. Both,
= Yolume 1812 ds ofthe culvert are submerged and the difference in water level upstream
‘downstream s 240-m, Calculate thedischargeassuming C = 0.6.

= ca f3git
Q=061x § (1.2)? Y2O8NEA)
Q=4794 me
Is desired to divert5.1 m?/s waterfrom a pool whose water surface
Ho 40mtanaajcentpon whose water surface levation i 2m by meas
culvert 8 m long and with both ends submerged. i
ulvert is needed assuming C= 0.58? hi
Solution

‘A standard short tube 100 in diameter of 495 m.


discharges water under headentrance,
2h small hole, tappedupperinthe side of the tube 50 mm from the i
wet with the end ofthe piezomete r tube the lowerend of which
He 45-42" 3m energd in a pan of mercury. Neglectin g vapor pressure, to whatheigh twill
5. = 058 x 4 (DY (208) mercury rise inthe tubs? Also determinethe absolute pressure upperat the
D=1.21m ofthe piezometer tube
lution
abl Note: For standard short tubes, the
essure head at point near the entrance is -
‘A 7nmdlameter dlacarges 2341 ers second of quid und
aad of 285 m Thooncedlaneter ofthe tat teve contcta found
calliperingto be 66.25 mm, Calculate the three orifice coefficients. q

‘Actual discharge, Q=2341L/s


| ‘Theoretical discharge, Qr= A J2gH
‘Theoretical discharge, Qr= $ (0.075)?/2@81)(285)
Borda's mouthpiece 150 mm in diameterdischarges waterundera
‘i, Determine the discharge in m/s and the diameter of the jet at the ¥
contract.
Solution
Underideal conditions,the coefficients of a Borda’s mouthpiece are
and Cy= 1.0. Seepage 293.
‘Diameter at vena contracta:
a=GA
Jat = 050% $ (507
d= 106.1 mm Energy equation between 1 and 2 neglecting head lost (theoretical:
Discharge Es B
QCA 2g ae ae,
C#GKG
C= 05x10050
Q= 05x F015 208A)
= 0.0678 m/s 2woiQr= Bt > a.)
‘Sumup pressure head from 31 in meters ofoi:
Oil discharges from a pipe #8Hs sore 018i -075= 2"
through a sharpcrested round
orifice as shown in the figure.
‘The coefficients of contraction PL = 213 mofoil
and velocity are 0.62 and 098, 7
respectively. Calculate the
discharge through the orifice Ing):
‘and the diameter and actual 2601Q7 = 213
velocity in the jet. Qr= 0.0286 > Theoretical discharge (since HL is neglected)
‘Actual discharge, Q= CQ: = C: C-Qr
‘Actual discharge, Q = 0.62% 0.98 x 0.0286
‘Actual discharge, Q= 0.0174 m'/s=174 Ifs
(Oil lows through a pipe as shown in the figure. Determine the disch
in the pipe assuming C= 0.63.

‘Theoretical velocity, 27 = oe
A
Theoretical velocity, or = 0.0286 =6474 m/s
(0.075)?
‘Actual velocity, » = Cor Solution
‘Actual velocity, 0 = 0.98(6.474) Qi=O-O
‘Actual velocity, 0 = 6.344 m/s
[Energy equation between 1 and 2 neglecting head lost(theoretical)
‘Another Solution: aE:
The Discharge through this type oforifice is given By: 24h

where C= coefficientof discharge ‘Sum-up pressure head from 1 to 2 in meters ofoil:


eee
‘7 pressure head at1in meters or fet of the uid flowing
Pesee 7+035-0356i¢-2= 2Y
= dlamete oforifice PL. Fan 4381 mofo
= dlameterof pipe oY
C=C xc=082x098 ee
C= 0.6076 Qn OOTPm/s
FORA) :
a = 0.6076 x BOY
(0.075),
oF ‘Actual discharge,
peareena maety =C
Qr=0.63(0.07787)
Q=0.0174 m/s =174 1s
C= coefficient of discharge
‘A. area of the orifice
2. pressre ea atin meer et of theowing
| 22-= pressure head at2in meters or fet ofthe fluid flowing
Ay= area of pipe 2x55)?
t= aay (a5-vi0)
A/Ay= (Oo)DF
1=984 seconds
2: 1
| Problem 6- 28
ose) ‘A 100-mm-diameter orifice onthe side ofa tank 1.89 m in diameter, draws the
Ross tO1e ‘surface down from 2.44 m to 1.22 m above the orifice in 83.7seconds. Calculate
1a the discharge coefficient?
= 005911/6m/s
Qns91 Solution
Since the head vary,the flow is unsteady,
= oy Min]
‘A.1.5.m-diameter vertical cylindrical tank 3 m high contains 2.5 m of water
100-mm-diameter circular sharp-edged orifice is located at its bottom. = a
7 249OE (aa 4 via]
C=0.60.
(@) How long will it take to lower the water level to 1 m deep after ope C= 0.8265,
the orifice?
(#) How long will it take to empty the tank?

‘and 4 m ofol 6p. gr. = 08). Find the time to empty the tank through a 100-mm_
diameter orifice atthe bottom. Assume C* 09 and C, = 0.98.
f=? from H;=10 to Hs=10-6=4
2A,
Dan) t=3.162|Y10 - Ja]
Ba ie a 1478 minutes
CAJZ~ (09%098)3 002° 2081) He)
alts
aay = 819.1
, Theal hand on ola) and when the Now wee erateeam
hhead was measured at4 m, Under whatconstant head H would the same orifice
Foneaoe
Tere
discharge the same volume of waterin the same interval of time?
Solution
eeasz_ (32) Under variable head:
Hy= 408) = 32m

fy" 10193 80 rts, 100mm


Time to empty theoi
thas
i=0
t= 8191 (V4 -J0)
f= 1698200
Total time to empty = h + ts = 2657-5 sec
Total time to empty = 44.3 min

obiem 6= 30
tank circular in cross-section is 10 m high. It takes 10 minutes to empty
‘hole atthe bottom when the tank is full of water atthe start,
Tong will ittaketodrop the upper6 ofwater.

ore i vertical cylindrical tank has an orifice fr its outlet. When the water surface in
the tank is 5 m above the orifice, the surface can be lowered 4 m in 20 minutes,
‘What uniform air pressure must be applied at the surface ifthe same volume of
t=KWA,A
t= 10min from Hi = 10m to Hs = 0m
ater isto be discharged in 10 minutes?
w=x{f0-vo]
K=3162
33 (CE 1992)
‘A composite non-prismatic Sm-high
= of | cylindrical tank shown has a frustum of a
‘one atthe bottom with upper base diameter
letk=ce 25 m,1.25-m-diameter at the bottom, and 2
ca tm high, The bottom contains 100-mm-
diameter sharp edged orifice withcoefficient
t= K|fi - a] of discharge of 0.60. If fully filled tothe top,
determine the time to empty the tank in
| In Figure(a) minutes.
y=5m
Hem Soluticon
‘The cross sectional area from
Hie im
t= Kit,
MJ] - Jip level to level
ee 2is constant
20=K\5-vi] spe ny - Jit}
K= 1618 _
Asm £57 = 491 me
aLO CAPg = 06% 404) 25
ie Ane = 02087
ies Hy=9m
Pa a54c
Hhn5+ Pe ata
He4m

thers Be: atse ares


= goer000 0-4)
fy a705
t=KVA,- Ji] oor mn From level 2to level 3:
10= 161850- Viz] ee
Bre =0618+ Vive apa
Squaring both sides:
5+0=03819+1236Ive +1+¢
ive =29273
Squaring bothsides:
Ten 8569
c= 7569m= Pe.
Peiy Ave aS +P *
Pe=75000. eee =) : |
Pa™ 74.25 kPa
pa 20762106+ 8h +H?)
A, Path
2 ry Etch-1oan8/2)
= [SPSTBEN
0.02067Th Z [pees é
062A,(2081
1 3.673{16(2)0'/?+ 8h"? 6 gh? ih,
= sosf2e4)'/2436(4)°7?+44)°7]
= 439004 sec sustna~ [st# Many, 2y872]
Total time to-empty, t= +f
Total time to empty, t= 4705 + 439.04 ost 2a,-[2505)"7 +Bos?s 2asrr]-0
‘Total time to empty, t= 909.54 sec = 15.16 minutes
= 0.009448 m? = 4 D*
D=0.1097 m= 109.7 mm
a <3
tank in the form of a frustum ofaright circular cone 150 m in diameter
edged orifice with C = 0.62 is located atthe bottom of tank, What diam sharp-edged orifice 100-mm in diameter, in the side of a tank having a
‘oforfce is needed to empty the tank in eightminutes? ontal cross-section 2 m square,discharges water under a constant head.
The rate of inflow over which the head was kept constant is suddenly changed
m 20 it/sec to 30 it/s. How long will it take,after this change occurs, for the
to become 2 meters? The coefficient of discharge may be considered
and equal 0 0.60.

© Qou™ CA JR
= 0.60 4 0107 Y208i)h
Qoa™ 0.02087 hi”?
for hi:
"Since thehead was keptconstant
when Qu = 20 L/s, therefore
Qui = 20 L/5 = 0.02m/5
Qe 0.020871/2 = 0.02
i=0918m
An n193 05+ HF
‘Ay = 0.144(12.25 + 7h + i?)
(OEP iowneueaeat
‘When the inflow was suddenly
changed to 30 L/s:
MA,
I, Gin Qo
Qn= 003 m/s
(Qu = 0.02087 hv?
A= 2x 2nd
Hy hy=0918m
H=hy= 2m
[9 atte
0.03 - 0.020877
Lae
os (0.02087)(1.437 — 7?)

=x
‘dh = 2(1.437 ~ x)¢ds) = -2(1.437 = x)de
When f= 0.918; x= 0.479
When 2; x= 0.0229
Z
seins [247
te
-snn [a -a) 1 [saa
sane[uirins-s Ir Mi, Qe
Q= CA, /2gH
= 388321.497fIn0.0229-1n0.479)-(00229-0479) dH = dy + diy
1,500 sec=25minutes [navy
_29te
hp TCAs a Vin Ve
40y=")
Aadays formul 92.25 m
Het yeas
When An and Ae are constant =5-1-225
W175 m
is a fr
Anta
t= Anda 1 1,
Par
Anha
aon" )h, Se cay,
iy=5m
a+ Ba CAL An= $QF= ame
haw QF =225em?
Hy™ initial head = 3 m
Ha= final head = 0m Ang = 0.60. § (0:20) 28381)
‘Aq cross-sectional area of tank 1=3 x2 = 6:m@ CA, 2g = 0.0835
‘Aqo™ cross-sectional area of tank 2=7.5 x2 15 mt
a 1. 12259)
"as ae Fe eee
1-089 seconds j 14757conde

. “ nd vertical rectangular water tank is divided into two chambers whose


jeesoiayoc coment peeesaceerfisfans a nial sections aremm3x mi?100and 5 mi repectively. Them dividing wall is
having C= 060. When the tbe is closed, the water surface in tank
tndirae ided with a 100 SyUe mm square hole located 0.5ees
rasieee es
from the bottom
meters above tank 2. How long wilt take after opening the te, forthe aes 9 etea ee deeake
piraee i Wak eee eet cen the water level in the two chambers after?minutes?
Anda \-ve]
a +a aa
CA,
~ Vi
Fag ae

FaVln oe ca
” 345 Daoinonyag =a
Aye al
(2.25%) e
amcor om
In the figure shown, how
long does it take to raise the
Water surface in the tank by
large reservoir other enlisted witha drain pipe 200 mm in diameter atthe lowest part
of constant water surface Compute the time required to drain the full content of the pool
‘elevation.
41 (CE May 2003)
CA, 2g = 0.111 te flows into a
A,=10%15= 150m? as shown in.
Hy=3m
Hhe14m
9 3 [o-vaa]
y= 14833seconds
‘Timefrom level to level 3:
he Ac dh
, CA, Page
= 10x
5
Lae
sa1071th ihs= 085% 1 "085m
| ioqto710h =17100
‘Au (Pgh 0.80: =
ergy equation between
© and © neglecting losses: conegt
f2gh =o.11h2 (Hi Aa)

|} 22
vo? 602
2g oO 05
ye Ld oe
ao
pe tztanan
ee nian
3%
ass
4 vf-o7= 10,08 > Ea.)
= 965.225 |i/2an
[Qi= Qa
(6% 101 = (085. Tor
=
ea 2,92]
= 965.2053 (1472-0)
= 014170
An Bq, (1):
2 (0.141709)! = 101.053
y= 1066 seconds (0.97993 v7 = 101.083; 22 = 10.154 m/s
‘Total time toempty, t= f+ fs ‘Actual velocity at ©, v4C.02 =095(10.158) =9.6467 m/s
= 14853 + 1066 Discharge = Ar(185 x 1)0.6467) = 8.2 mis permeter
Total time to empty, t= 2549.3seconds = 42.49 minutes
6-42 (CE Board)
A horizontal 150 mm diameter pipe graduallyreduces its section
diameter, subsequently enlarging into 150 mm section. The pressure
‘mm pipe a pointjust before entering the reducing section is 140,
head is lost between the points where the pressures are known, com
rate of flow of water through the pipe.

Q=2=Q
Energy equation between 1 and 2:
E,-HL=E,

981 ‘Sum-up pressure head from 2to in meters of water:


#2 +y 03760136) 0375-y= Ft
7
PB 475m

‘water main, The deflection of mercury in the differential manometer con Ina.)
161.20=4725
from the inlet to the throatis 375 mm, (a)Determine the discharge neg
head lost. (6) Compute the discharge if the head lost from the inletto the
Q= 01712 mys (theoretical discharge)
4s 300 mm ofwater, and (c) what is the metercoefficient? Energy eq, between 1 and 2 considering headlost
E\-HL=E:
mA
+Begene oe t+ Be Ben
7 a
ee he ee Th7 sors+y 0x26 -036-y= BLY
1612 @=4725-030 PA~P2 5.286 m of water
= 01657mys (etual charge)

G=0172208/2 > (eoretical discharge)


153 Q? 5.286 0.75
ee ee
| Seg
_ ons
oan?
co 086s
x Jost: (use the actual discharge)
vertical Venturi meter, 150 mm in diameter is connected to a Tnergy equation between 1 and considering head lost:
diameterpipe. The vertical distance from the inletto the throatbeing HL=B
If the deflection of mercury in the differential manometer connected.

HOATIYE , Pr 4 gp = SOL D, Pe + 075


‘Solution Meosyos esos y
Hus 2098-075
Q-2=2 7 Pa
cry Ea beeen and 2ephting hadnt Pe= 5.286 ~ 2058-07
OE
oe
Beye Ben
ve? Pa
ag ag
2
2? 80 2 =
Fosyose °°” Faso
Go C= 068
+2 6075 Q= 01171 m/s
a 2
A: 015? 80.1171)"
u-()f-(S5) [poe
=
Se ee Hu=2499m
Sum-up pressure head from to 1
in meters of water:
eee
Neglecting losses, ¢
‘ealculate the ‘Also, from the figure:
through the Venturi Btwn 025+y
‘meter shown, yi-y= 025-2
Fi Pe w025-2
toot
In Ba.)
153.01Q'= 025-242
Q= 0.0404 m/s = 404

Soluti
i ‘ng oil having specific gravity of 0.852. If the recorded flow in the
jor vas 1.5 liters per second, what could have been the deflection of water in
rl inlet and the throat?

Qi Q=Q
Brergye between and 2 neglecting head lost
‘Actual discharge, Q=1.5L/s= 00015m'/s
ao? 04 Since the head lost is not known, thetheoretical discharge will be used
}
Hoa 7 gorse 4 Q=CO;
ae Or 0.0015 = 0.957 Qr
assoige= FL fer 3m) Q;= 0.001567 mi/s
‘Sun-up pressure head from 1 to 2in meters of water: Qi= Qs= 0.001567
Note: Neglecting densty ofa, the pressure in at ay po in the
fae
Fu iytye
pe ey,
Tinergy equation between 1 & 2 neglecting headlost: (usingQ)
Boks
Me tt Ee Bay
BF of
0.001567)", = 80001567)? ps, 4
7 osomy O° se(00375)° * -
BL Pa 0.09618 m of oil
rain
‘Sum-up pressure head from 2 to 1 in meters ofol: Qr= Q)= 0.0085 m/s
Be
Pa eer
h
79" oes ho¥ 12" 0 52 a 95400085
840.0085)")
Be ee 900.025)" i
oarari PL. Pe Hsa=0764m .
ey na 02222 020-5(0.0085)
0.17374 = 0.0618 Hiss 02952 920-2 (0.025)
(0005)
= 0554 m=354 mm Has =3.057m
Energy equation between 1 and 2:
Oil Gp. gr. = 08) flows at the rate of 85 liters per second through a 25 Ey-HL= Ey
diameter horizontal Venturi meter, which is attached to a 375-mm-di oe ea Ha +
BP oy ey Pa
pipe as shown in the figure. A differential manometer con mY 28 a
attached from the base of the inlet to the throat and to the base of the 890.0085? Pi, = (0.0085)? Pay,
‘Calculate the deflection of mercury in each tube ifthe head lost from the Figmasrye 7 07" recone yO
the throatis 5%of the velocity in the throat and from the throatto the 0} 8
20%ofthe velocity head inthe throat, FL. P w1303.mofoil
toe
names vine Sump pressure head from 2 1 in meters ofol:
Pa7 ey-mny+nfig y= BE7
ton BE. P2 a1g05
Y 7
jy= 0814 m= 814mm
Energy equation between 1 and
Fi-Hlsa-Hlas= Bs
2
+ Ph + ay-Hlsa-Hlay= + Poy abe vi ees opeA bk sade, 1 Sri
Y ary, ‘of oil (6p. gr: 0.90) 50 that one ofthe openingis directed upstream
+P 10-0764 3057 = 0
PL. PS «3.821 mofoil
7
a Pressure head from 3 to 1 in metersof ol = (EGR (borevel)
BE sy-hy+ nig ye = (2005)
= 0.99 mys
sain B 3.821

‘spward. One hundred twenty-six (126) litre per second of all (sp.
flows through the pipe. The throat section is 15 cm abovethe upstream.
fC, = 0.57, whatis the difference in pressure between the inlet and the t
Solution
‘The discharge through a vertical Venturi
-meter is given by the formula:
ae
+ Q=CAr 2g Jt
}1
1 “0
=(57
0,0;
/0,"
AP o
ane
0126 «0957 § (0159/33 1 (0115/030)*
bid Flow Measurement CHAPTER sux
Fluid Flow Measurement
Hay equation between and 2negeting hes ot:
IE twopoints 1 and 2 as
Baye
vy?
BsBen
2 in the figure. Point 1is “ome
2 y ay
2
BP yomny
2 Y Y gy equation between 1 and 2
Pa
PBB tng head lost
| 28 moter > 0) fn
neae e oe sue Ben
vy ors10 -0075-yx Pe
0+v ron Boo
23 7
ee asad
Rireys
Int,
nose or-apprewarbed fronton malar fir
=356m/s > theoretical velocity TyOeSRE y= PE
Actual velocity, = Cy 22 = 54.006 m ofa
‘Actual velocity, v= 185.6) = 35.6ny Y
Be oe
Air (w= 12 N/m) is flowing 22 = 54.006
through a system shown. If ol 2s
(sp. gr. = 0827) shows a m= 3255 m/s
deflection of 80 mm, calculate — Flow rate, Q= Arma
the flow rate neglecting head
lost. Taree:
Flow rate, Q= 0.06391 m*/s = 63.91 1/s
‘A Pitot tube in the pipe in which air is lowing is connected to a
‘containing water as shown in the figure. If the difference in water
manometer i 875 what is the veloc Poin
of low eee = ;
/=37A4 m/s. (theoretical veloity)
CoeficentC= 099)”
‘Actual velocity:
peeve
= 099 «37.44

nozzle is a device inserted into a pipe to measure the flow as shown in


figure. If Azis theexit area,showthatforincompressible flow,
ovo A eZ

| is and isdetermined experimentally.

Energy equation between 1 (stagnation point) and 2 neglecting head


EinB
Biot se ae
25 ecea ag: IY ae
0+ Brome Bag
Y a
ils Bien Pa i
3% oY Y pa (0)
Sum-up pressure head from 12in meters of water:
PL y0108752810 «, tyne
ead 2 Sorasy ¥
Bs 2 a 71s4mofair
Ta
6-54
Bray equation between and 2 malighen ost: tube being used to determine the velocity of flow of water in a closed
indicates a difference between water levels in the Pitot tube and in the
‘of 60 mm. What is the velocity offlow?
| SC Pil hm ee
2g a oak
Beh
2
oe A
4 spate
vy? Bee
tee
Pi
a be Yaak
BY
(ReBIVO.0)
a ot es
2s” peaiveeatier St
[Q1= Qi)
‘Anti = Avs figure shown, pressure gauge A reads 75 kPa, while pressure gauge B
n= (&e/Anon '82 kPa. Find the velocity of air assuming its unit weight to be 20 N/m,
G.= 0.92 and neglect compressibility effect.
In Eq, (1):
a
a (Ay
[1 (Ay/Aypy 22 = Bt. Pa
14 fg
- 1 Pi-P2
= faa Ao)
Q= Cox Ams

= $2000 _ TEAR~ 350m of ar


20
v= 092 /21981)250)
‘Note: This formula can also be used for Horizontal Venturi Meters, = 76.24 mys
358 riusd Flow Measurement
| Problem 6 - 56
Carbon tetrachloride having specific gravity of of 1.6 is is f flowing ar, sha up-crested ‘weirwel 15m long withil end contractionssu
The dlferental gage atcha othe Pesta shows 100mm Tettnine hedchage whentoe hendis 30cn Se 2
‘of mercury. Assuming C, 1.00,find the velocity offlow.
Solution ce the height of weir is large compared to the head H, the velocity head
approach can be neglected.
Using Francis Formula:
(Qe 188 Lip?
= 1.84 (15/03)
(= 4595 m/s

38
sngular, sharp-crested weir with end contractions is 1.4 m long. How
Pehould it be placed in a channel to maintain an upstream depth of 2.35 m
Mow of 40 literssecond?

p= PCPA
7
~ PSpenia-Sra)
Saas
= 9.08(13.6-1.6)
16
h=06m
paix JAOBO8)
v= 343.mys5 L=14-02H
0.40 = 1.84(1.4-O2H)EP?
“Solve for H by trial and error:
‘Try H=03
1.84[1.4- 0.2(03)](03)°2 = 0.405 = 04 (OK)
From the figure shownabove:
Pad-H
P=235-03=205m
During, a test on a 24m suppressed weir 900
"maintained constant 1t 200factor
mm.Ca?In 38 seconds, 28
collected. Whats the weir
Solution

Q= CoLlH + hv? he]


Q , 10125
4
Q= Col+h- her}
= Volume “(since the flowis steady)
~ 800-7579 1/s 10.125 » 1.88(75)(H + 0.028792 - (0.0287)"3]
ef m/s
Q= 0.7579 H=077m
Height of weir, P=1.8~H
Velocity of approach, v,= 2 = 97579. 18-077
Height of weir, P= 1.023 m
ee
Velocity of approach, 21=0.25316240.2)
m/s
_ (026316)?
are
Aree ce the flow over a suppressed weir 3 m long and 1.2 m high under @
of 900 mm, The weir factor C,= 1.91. Consider velocity of approach.
Q= Col2.4)(03 + 0.00353)" - (0.00353)"2] = 0.7579
C= 181 is
_—
a
A appne es 7 mgtotere10:0m/e yaOl
channel. The weir factor C. = 188. To what height P may the weir be bul
Hf e-tin
thewaebein thwelmhaocxcsodD0mcap? 12m

L=am
fi the width, in meters, ofthe channel at the back ofa suppressed weir using
following data:
Head, H1=285 cm
Depth of water, d= 2485 m
It can be seen thatthe discharge Q varies with fe which in ee ee
Q. Using this formula directly would lead to trial-anverror
First, we solve the approximate velocity of approach bysolving a hi oe
discharge using the formula Be Ae
Q= Glin e terete
Coane 5 the formula:
SolvingforLandyusing
Q= sain
he = 0.001288(4.892)° 0.84 1.8410.285)¥2
1h, 0.0307 m
New Q=1.91(3)[(0.9 + 0.0307)- (0,0307)%]
New Q=5.14m'/s

|
New Q= 1.91(3){(0.9 + 0.03358)"- (0.03358)
New Q 5.133 m’/s
| ‘The discharge converges at 8.133 m/s
UsingEq.6-33:
wet
Q coun! |s of) free)
G 3C,? 3a
ost tsxjeay|100 3) !
I 2g 2 2 L=3m
= 02789
Q= 191809") re027eq are 22)
‘09?
=)
discharge from a 150-mm-diameter orifice under a head of 3.05 m and
Q=5143 mys approximately ient of discharge C = 0.0 flows into a channel and over
angular suppressed weir. The channel is 1.83 m wide and the weir
ght P = 1.50 mand length L = 031 m. Determine the depth of water inthe
a tion in depth = Hi ~= 60 mm.
‘The discharge through the orifice equals the discharge throughthe 0574 yee
as 0481 OS
For the orifice: oe
F Q= CA.fight SOF
Nee =0.06
Q= 0.60% § (015)(APB) Ne
= oosar m/s
siya eae 55 (CE Navara F096) 1315 m/s. Thecet length s2 mand
octane
Soman = 1 s403ny02
Mhcharge
ins ar ince 9 79°37 49weiwith the Hootie. nd the Bed on he
vera rpevoial
H=0zAm ies
DapTuer
ar pee eee sie init
He nas angle gven ithfr «Cpa wk
tomss1so Lin
tou Tats 1859@)02

se ee eeeeeae
‘The flow in a rectangular channel varies from 225 liters per secondto
ee = eee
ion 75m If water can be dravvn from elevation 765 toelevation m
degree V-notch weir atthe end. How many weirs are needed to Ue minutes, find the length ofthe spillway in meters. Use Francs formula
variations indepth to 60men? lecting velocity ofapproach
Solution
For standard 90° V-notch weir, Cy = 14
Qe tars?
Letbe the required number ofweirs
Total flow, Qr =Nx Q= L4NHB?
When thedischargeis0225m’/s
0.205 = 14NH?
Hix O37y (head when the discharge is 225 L/3)
‘When the dischargeis 0.35m’/s
035 = L4NHS??
Hy= SOT (head wien the dischargeis 350 L/s)
ig. 6-53: ‘Length of weir, L= 1m.
p= 24 [ 1 ‘ Initial head, Hy =m
etlge-ge|
‘Av Abectaes = 46000m2
‘The dropof waterlevelafter discharging 72 mis
ere
{42 minutes = 2520 seconds Pe roa,
Hi=15m Finalhead, Hi=1~ 018 = 082 m
Hy" 050m Weirfactor (Francis), C= 1.84
14 randis Formula)
200| 1 i
oa [as-7]
250= 2146000) 14
L=1186m
i
ne
tm HAO 1A = 45.38 seconds

V-notch weir is located or cut at one endofa tank having a horizontal square
on 10 m by 10 m. Iftheintial head on the weir is 120 m and it takes 375
AA rectangular
tal rectangularsuppressed weirhorizontal
tank havinga oflength 1 m is constricted or cutatthet ds to discharge 100 my of water, what could have been the vertex angle of
section20 m by 20m, Ifthe wee, Use = 060,
overthe weir is 1m, compute the time required to discharge 72cu.m.
Solution

‘When 100 m? is discharged from the tank, the waterlevel drops by y meters,
100xy= 100
yim
ee ay ‘Thus, the flow is unsteady with inital head Hi = 1.20 m and final head
Ctl yiVm Hy= 02m.
‘Water surface area at any time, A, = 20(20) = 400 m?;
ye [Aeait
Ii, Ome Qs dA figh
‘Av 10% 10= 100 me dh 2d
Qua = i C25 tan} Hi? Express x in terms of h by squared property of parabola:
= $5 (060) J25 tan17/2 Sa
Quer ™ 1.417 tan HPI? 15-k 2
a # x=0707/15-h
a aA = 2(0.707 JI5—h )ah
=: 100d
Taian= = 2(0707JI5=h jak [Fgh
220 dQ =6.263V15—h Vh dh
s21s7tang = fir?
o-sam [as-is
5.3137 tan$ -[ 2r] a By trigonometric substitution:
om Let h=15sin9
Pater a =O T= TIAA
tan = 130726 3 sin cos 09
6 =10517 when h= 0,0 0°
whenl= 15,8 =90° = x/2
® lows trough a arto weithat is 2m dep and 2m wide att
6-6 2
Wate 2-620 [ 151 Ssin? 0(1.2247sinXBsin0cose)
under a constant ead of 50 m, Assuming C= O48, determine dsc
through the weir. = te /2
Q=28182 |sin* @cos? ado
‘Solution
ie
oval33+4]
abies) eg
ely Sceeaaia
‘Actual discharge = CQ = 0.65(55836)
‘Actual discharge = 3597 m/s
ER SIX
Fluid Flow Measurement
"A apesoidalweir having sie slope of 1H to 2V discharges 50 m/s rp-<estd suppresed rectangular weir mongand a standard 04d
‘stantheadof2m. Find the length ofthe weir assuming C= 0.60, h weie are placed in the same weir box with the verex ofthe V
150mm Below the rest ofthe rectangular weir. Determine the eadonthe
lr weiwhen herdischarges are equal. Use Francis formul
Let H be the head on the rectangular weir:
For the rectangular weir: (Hy = H)
Qe= LBALHip?

For the Vanotchweir (Hy = H+0.15)


Considerthe horizontalstrip shown(treated as anorifice under head) Ore 14th
dQ= CHA 2h LACH + 0:15)
dA=xdh [Q= Or
z= 2-H) 1727 = (H1+0159
rate -W By ial and error,
| seh eave a? ‘H>0051 m
dA = (L+2~hydh

AQ» CBg (L+2—Myh2a


0-64 [fsa
= ag [4un°?? + $427? —308/2 t
5006, [31097+40"? -307"?]
L=918m
Using the combined rectangular and tiangulatweir formulas:
O= $C2gLPs & Cgtangie
From the figure, tan $= 3
50 = $10.6) 2gLOW? + # (0.6) J2g ( $)@2)"2
L=918m
4 Collet veer should be constructed inorder that the head of
im when the flow rates 3761/5? *
Water is being discharged through a 150-mm-diameter pipe dl Ans: 215m
container that has a volume of6 m’. Find the velocity of flow th
ifthe timerequire toil thecontainers 3 minutes and 28,7 second ”
upward through a Venturi meter as shown. Assuming discharge
1.984, calculate the flow ofol.
Problem 6 - 73 ‘Ans: 158 L/s
Calculate the discharge through the submerged orifice shown in,

| 50106 peach 2 15199



Te ie
i

Problem 6-74
‘The truncated cone shown has 8 = 60°. How longdoes it take to dray
surface down from h=5mtoh= 2m?

having a throat diameter of 150 mm is installed in a horizontal


water main, as shown, The coefficient of discharge is 0.982.
liference in level of the mercury columns of the diferental
ed tothe Venturi meter if the discharge142 L/s
‘Ans: h=255 mm
Problem 6 - 78 are closed conduits through which fluids or gases flows. Conduits may
full or partially full. Pipes are referred to as conduits (usually circular)
Determine the head on a 45° V-notch weir for a discharge of 200 L/s. th flow full. Conduitsflowing partiallyfull ae called open channels, which
‘O87. be discussed in Chapter 8.
Ans:6
id flow in pipes may be steady or unsteady. In steady flow. there are two
eis cof flow thatexist; they are called laminerflow and turbulentflow.
For the sluice gate shown, if C= 0.98, whatis the flow rate? If C. = 0.62, w
Ans: Q= 9.7 m/s; y= 1 low is said to be laminar when the path ofindividual fluid particles do not
for intersect. The flowis always laminar when the Reynolds number Reis
than 2,000.

lent Flow
flow is said to be turbulent when the path of individual particles are
gular and continuously cross each other. Turbulent flow normally occurs
bn the Reynolds numberexceed 2,000,
flowin circular pipes can be maintained up to values of R, as high as
4000. However,in such casesthis type of flow is inherently unstable, and the
disturbance will transformit instantly into turbulent flow. On the other
dit is practically impossible for turbulent flow ina straight pipe to persist at
‘of R, much below 2000, because any turbulence that is set up will be
‘outby viscousfriction.
Velocity,
critical velocity in pipes is the velocity below which all turbulence are
‘out by the viscosity of the fluid, This is represented by a Reynolds
oF 2000,
(CHAPTER SEVEN
‘GHAPTER SEVEN Hluid Flow in Pipes
Fluid How in Pipes
“REYNOLDS NUMBER DISTRIBUTIONIN PIPES
Reynolds number, which is dimensionless, is the ratio of the i Flow
viscous force. locity distribution for
flow, at a cross section,
For pipes flowing full parabolic law of
with zero velocity at
TR ee lls In circularpipes, the
varies a¢ the ordinates
paraboloid of revolution _
average velocity equal to
of its maximum
where: <
‘= mean velocity in m/s Figure 7 - 4: Laminar flow velocty éstibuton
D= pipe diameter in meter
‘v= kinematic viscosity ofthe fluid m"/s juationforthe velocityprofile for laminar flow is given by:
14 absolute or dynamic viscosity in Pas
i
For non-circular pipes, use D = 48 thenthe formula becomes;
Ay
apoo Vie
\elocty at any distance r from the center ofthe pipe may also becomputed
the squared property ofparabola:
Se
ont ‘

Table 7 = 4: vcs and Densty of Wate at 1 atm


Temp, °C peal Pes mils
10 Tei07] Ta 0
ra© 000. 1.3071 1.307 10
20 1.0031=07]107 1.005<0
025- 10 where fc head Tost inthe pipe
50 799 L= pipe length
0.657 10" —|~0.652 10
30<0 Osae 10 Os88. 18 rom1 pipe radius
Centerline or maxinwum vel ee
eo 467 107} aro ci
2
70. 0.405 « 10°
(035510
O.414 x 1
0.365<1 i absolu viscosity ofthe liquid
ee eel te
0316.10" 032710 init weight ofthe fluid
30 10.283 «10 0.285 10 w= velocity atdistance r from pipe center
100 v= average velocity
"Turbulent Fiow
‘The velocity distribution for turbulent flows varies with Reynolds: {f= friction factor
4et0 velocity atthe wall and increases more rapidly fora short dist ‘v= mean velocity
‘walls as compared to laminar flow.


Figure 7 2: Turbulent Now velocty stbuton

‘The velocity,wat any pointrin a pipe of radius r, and center velocity

der a mass of fluid of length L and radius r to move to the right as shown
figure. Due to head lost hi, the pressure pz becomes less thanpr.
shearing stress, atthe surface of the fluid can be found as follows:
[h=0)
A-B-
eB
Am pAi-pAr
wx Bee Lm pr net paren?
D LOSSES IN PIPE FLOW
fosses in pipes may be classified into two; the major head los, whieh
< by pipe friction along straight sections of pipe of uniform diameter
roughness, and Minor head loss, which are caused by changes in
ons of flow,and are commonly expressed in terms of kinetic energy.

JOR HEAD LOSS, hy


Darcy-Weisbach Formula (pipe-friction equation)

“f= friction factor


L= length of pipe in meters orfeet
ipe diameter in meter or feet

‘non-circular pipes, use D = 4R, where R is the hydraulic radius defined in


7-4
circular pipes, the head loss may be expressed as:

Where Qs thedischarge,
‘Manning Formula
tanning formula foneofthe best-known open-channetls and I
only utedinpi pes. The Formulas given.
ow
97 TABS2/502 ngtoh un)
Where n= roughness coefficient
R= hydraulic radius
$= slope ofthe energy grade line = l/l
hy and R= D/4 to Eq.7-31
Substituting $= am and solving for Mt
y" 12

Fornon-circular pipes,use D = 4R
circular pipes:
Q

wae Were wae


ey urbeR
Figure 7 ~3: Moody Fiction Factor Chart. This chart i idemtcal
to Ea. 7-29 or turtulen fow, ‘The value of mis given in Table7-4.
Hazen Wiliams Formula
‘The Hazen Williams formula is widely used in waterworks industry.
formula is applicable onlytothe low of waterin pipes large than 50 mun (2
‘and velocities less than 3 m/s. This formula was designed for flow in
pipes and openchannels but is more commonly used in pipes

=02785G Dm SH
1o67L.Qus
oot
where: C,= Hazen Williams coefficient
D=pipe diameter in
R= hydraulic radius
‘S= slope of the EGL = hy/L.
‘Table 7-3: Recommended Value for C;for Hazen Wisams Formula IINOR HEAD LOSS,
Description ofPipe Value ofG nor losses are caused by the changes in direction or velocity of flow. These
Esremay enooth and svat pipe. 10 iges may be due to sudden contraction, sudden enlargement, valves, bends, ancl
ew, smooth east ron pipes 12 y other pipe fittings. These losses can usually be neglected ifthe length of the
Averageiron pipes ii ipeline is greater than 1500 times the pipe’s diameter. However, in short
‘sified sewer pipes 110 ‘because these losses may exceed the friction losses, minor losses must
‘Cast on pipes, some yes soca 00- considered.
‘Cast ion pipes, n bed condition 0
New rveied ste 110
‘Soath wooden oe wood Save 120.
388 HAPTER sevEN
$luid Flow in Pipes
‘A. Sudden Enlargement
‘The headloss, m, across a sudden enlargement of pipe diameter is:

where: v1 = velocity before enlargement,m/s


= velocity afterenlargement, m/s Head Loss = Kv = v)'/29
Another equation for the head loss caused
determined experimentally by Archer, and givenbyas: sudden enlargemé

A special application ofEq, 7 -40 and Eq.7- 41 is the


‘reservoir. The water in the reservoir has no velocity,discharg
so a
e froma pip
full velocity i
Sa IO AF OT 0 9 TTT LO TAP TT
oie between vere Hes Doe
lost,
laure 7 coeficent ora cuacig
B, Gradual Enlargement
‘The headloss, m, across a gradual conical enlargement of pipe diameter i ‘Table 7-5:Loss coeients for sxden contraction
yo, oo 01] 02] 03] oa] 05] 06] 07] 08] 09] 10
K__[os0[ 045] 042 039] 036] 033] 028] o22| 015] 006] 0.00
‘The approximate values of K are shownin Figure 7-4

SuddenContraction Flush connection.


‘The headloss, m, across a sudden contraction of a pipe is: Projectingconnection.
Rounded connection.
Pipe projecting into reservoir.
Slightly rounded entrance.
Sharp-cornered entrance.
where: _K,~ the coefficient ofsudden contraction, See Table 7-5
‘v= velocity in smaller pipe
890 Hudrowpcs
©. Bends and Standard Fittings
‘The head loss that occurs in pipe fitings, such as valves and elbows, and
is given by:

‘The approximate values of K are given in Table7-6. K values vary not 6


differentsizes of fittings but with different manufacturers. For these
‘manufacturer's data are the best source for loss coefficients. [vne
‘The head loss due to pipe fittings may also be found by increasing the
length using the values of L/D in Table 7-6. For very smooth pipes,itis
to use the K values when determiningthe loss through fittings. See Pro pipe with constant diameter, the difference between the water levels in the
M4 jmetertubes, If the pipi is horizontal and with uniform size, the difference
pressure head measures the head lost between the two points
‘able 7-6: Loss factors for pipe tings
Fiting eps the pipe is very lange such that the velocity head is very smal, the total head
HI canbe taken as equal tH.
[meen
‘Geer yorebend
ot
groan od CONNECTING TWO RESERVOIRS
eunradusebm When one or more pipes connects two reservoirs asin the figure shown, the
oe head lost in all the pipes i equal othedifference in elevation oftheliquid
45" cow faces ofthe reservoir.
oteve, wide ope
Gatevate Ra open 4

PIPE DISCHARGING FROM A RESERVOIR


‘The figure shown below shows the conditions of flow in a pipe of
diameter discharging from a reservoir into open air. The velocity head
pressure head in the liquid surface ofthe reservoir are zero. If there
head lost, the velocity head could have been equal to H, which is the
between the water surface in the tank and the exit end of the pipe
velocity of flow could have been v= /2gHf, but such is not the ci
Tosses.
Garren seven
392 Fie HowPipes
PIPES CONNECTEDIN SERIES
For pipes of different diameters connected series as shown in t
‘below, the discharge in all pipes are all equal and the total head lost is
pee oemven ie ae the pipe system shown, pipe 1 draws water from reservoir A and les t0
jon C which divides the flow to pipes 2 and 3, which join again in jusction
land flows through pipe 4. The sum of the flow in pipes 2 and 3 equals the
in pipes 1 and 4.Sincethe drop in the energy grade line between Cand D.
qual to the difference in thelevels of piezometers a and b, then the hei lost
pipe 2s therefore equal to the head lostin pipe’.

TUE PIPE TAU APO


from neglecting minor losses will ordinarily not exceed 5%, and if the:
length is 1000 diameters oF more the effect of minor losses can ws necessary equations forthesystem are
considered negligible. Neglecting minorlosses, the head lst becomes: Oa Ea
Q=O:+Q 34.0)
ha=he 2.0)
HLa=Withathe > Ea. (8)
If, however,itis desired to include minor losses,a solution may be made! 5
neglecting them and then correcting the results to correct them. ‘The number of equations needed to solve the problem must be equal to
Ee ; pices ‘number of pipes.

PIPES CONNECTED IN PARALLEL


‘equivalent pipe must have the same discharge and head loss asthe origin
system.

‘Type 1: Given the discharge in one of the pipes,or given the pressure at the
Junction P, and the required is the elevation one of the reservoirs or the
RESERVOIR PROBLEMS: diameter or length of the one ofthe pipes, ane
In the figure shown below, the three pipes 1, 2, and 3 connects the &
reservoirs A, B, and C respectively and with all pipes meeting at @ com ‘Type 2: Givenall the pipe and elevation of all reservoirs, find the
junction D. flow ineach pipe, whichcanbe solved by trial and error,
any of these types, the main objective isto locate the position (elevation) of
he energy at the junction P. This position represents the water surface of an
pinary reservoir at P. The difference in elevation between this surface and
he surface of another reservoir is the head lost in the pipe leading to that
ervoir (Seefigure abooe),

re In Solving Reservoir Problems:


Q=Q+Os
1. With known flow in one pipe leading to or flowing out from a reservoir
‘of knownelevation solveforits head lostic
2 Determine the elevation of the energy grade line atthe junction of the
pipes (P) by adding or subtracting (depending on the direction offlow)
the head lost in the pipe from the elevation of the water surface in the
reservoir.
"4, Ifthe known value isthe pressure at P,the elevation of Ps the 3. AMAftertheheat
cee the diectin
in teeth of Os (myeofp
oesye vars. svat
of P+ pri. Lege
4, Draw a line from P’to the surface of the other reservoir. These
represent the EGL’sof each pipe. The difference in elevation
and the surface ofthe reservoir is the head lost in the pipe.
5, Solvefor the discharge.
‘Type2:_ (See Problem? -65)
1. Given all elevation and pipe properties, determine the direction of flo
‘each pipe. Of course, the highest reservoir always have an outflow
the lowest always have an inflow, but the middie reservoir (B) may’
aninflow or outflow.
2, To find out the direction offlow in pipe 2, assume that Qo = such
is aelevation B, then the values of fn and la can be solved. (In the fig
shown, hy = 20m and hp = 30m). With hg and hp known, solve
| and Q). If Q1> Qs then Qs is towards B and P’is above reservoir B. If
<Q, then Qs is away from B and P”is below reservoir B.
‘expressed in terms of x (usuallyinthe form a V+).
SampleDarcyWeisbach or Manning formal i wed, Hy vas with
W=KQy
hm x= Ki Q?
Qiekive > Faq)
n= 20-* Kx Q#
Qr=Kk/2=x > Eg.)
Iip=50-x= KO
Oees 75.0)
[Q=Q+Q)
KiVk = K1y20-3 + Ks50-2
Simply the equation and solve for x We may aso use wi-and-eror
lution
“4. Once xis determined,ees
recaer substitute it value
(oe Seaties CaCl
loop must be zero and,
2. The flow entering a junction must be equal to the flow leaving it Inapplyingthe above equation.
EKQ#* algebraic sum of the head loss in the circuit (clockwise postive,
‘The first condition states that there can be no discontinuity in pressure | ‘counterclockwise negative)
pressure drop through any route between two junctions mustbe the sam EKQ, = absolute sum without regard to direction of flow (clockwise
‘second conditioni a statement ofthe law of continuity . positive, counterclockwise positive)
Pipe network problems are usually solved by numerical methods The correction a is added or subtracted from the assumed flow in order to get
‘computer since any analytical solution requires the use of many simul the true or corrected flow, It is added if the direction of flow is clockwise and
‘equations, someof which are nonlinear, btracted if counterclockwise,
ay The general formula in computing the correction a can be expressed as:
‘The procedure suggested by Hardy Cross requires thatthe flow in each p
assumed so that the principle of continuity is satisfied at each ju
correction to the assumed flow is computed successively for each pipe
cen
= 2 for Darcy-Weisbach and Manning formulas and n = 1.85 for Hazen-
Let Q,= assumed flow iams formula, The value of K are as follows:
Qe true flow
‘a= correction then:
Q-Q+a
Using Darey-Weisbach formula:

TKQt=0
IK(Q, +a)? =0
IK Q? + 22KaQ, + 2Ka'Q,=0
Ifcxis small, the term containing o? may be neglected.
ing specific gravity of 0869 and dynamic viscosity of 00814 Pas flo
‘casiron pipe ata velocity of 1 m/s, The pipe is 50 m long and 150
indiameter. (a) Find th head lost duet friction, and (theshearing strat
Water having kinematic viscosity v = 13 x 10% m?/s flows in a 1004 eof ce pipe.
diameter pipe aa velocity of45 m/s, lathe low laminar or turbulent?
Rem We
#
ew (1N0:5)1000%0868)
0st
R=1,601-<2,000(aminar)
Oil of specific gravity 0.80 flows in a 200 mm diameter pipe. Find the ef
velocity. Use x= 814 x 102Pa-s,

Atcritical velocity R.=2000,

| ogo = 22(02)(1000%080)
8.14% 10"
= 10175 ays
73 = (98100.869)(0.68)(0.15)
For laminar flow conditions, what sizeofpipe will deliver 6 titers per 460)
oil having kinematic viscosity of 6.1 x 10¢ m/s? wo da,
Solution
For laminar flow conditions, Rs 200, Ble
Qp ermine the (a) shear stress at the walls of a 300-mm-diameter pipe when
p-2eeevv be wing canoes a asl Lah of esl Mn a lenge Gh tee thee,
velocityand the () shearsires at50 mum ftom theentein ofthepipe.
0.006
oe
nao
D= 0.625 m=626 mm
= 0127304)
"(@) Shear stress at walls: BP ®t
yD. R=118 (laminar)
aL
= 98106)03) = 0127304)
Re Tsao?
Re 843 (laminar)
() Shear velocity
F p= 0227302)
£06510
ae Fi R.=31,361 (turbulent)

jae = 01273(0.)
‘000 Ke" Yo2e10
| 0-02 my Re= 10483 (turbulent)
| (€) Shear stress 50 mm from pipe center jew 0127400)
| woth, 115%107
= 38100)69) ; i
2160) ge 0127401)
| re16Pa 118610"

| A fluid flows at 0.001 m’/s through a 100-mm-diameter pipe.


whether the flow is laminar or turbulent the flu (a) hydrogen (7 flow atthe rate of 200 lit/sec through 120-m horizontal pipe having &
104 m/s), (2) air (v = 1.51 x 105 m/s), () gasoline (v = 4.06 x 107 me ‘of 300 mm. If the pressure difference between the end points is 280
| water (v= 1.02106 m/s),mercury (y= 1.15 + 107 m/s), oF) gh g, determine the friction factor.
| 1.18 x 10° m?/s)

| . a)
3.808 = 0.0826(1200.2)? 75 = £2.0826(00366)(150}0"
03)" (02)?
2 #000111 m/s
Q= 441 Lise.
A fluid having v= 4 x 10% m/s flows in a750 m long pipe havinga dian
20mm. Determine the headlost required to maintain a velocity of 3 m/s. The head lostin 50 m of 12-cm-diameterpipe is known to be 6 m when:liquid
Solution ‘of sp. gr 0.9 flows at 0.06 m/s, Find the shear stress atthe wallsofthe po
R= D
a
= (9810«09)(6)(012)

Ti (CE Board)
t commercial size of new cast iron pipe shall be used to carry 40 gpm
tha lost ofhead of1056 feetper mile? Assumef= 0.019.

= 0.08260"
oe
ng £24, 223i ein
Fluid ows through a 20-mm-diameter pipe, 190 m long at a Reynolds Q= 440 min gal * sec 60
‘of 1,750, Calculate the discharge ifthe head lost is 175 mi (= 284 lit/sec = 0.284 m/s
Solution 5280 ft 1m.
tat mites tes “388%
(0.0826fLQ* L= 1609.76 m
WS epaae 1
Since R, = 1,750 <2,000, the flow is laminar. jenssnedee
osm
3.22. 0,0826(0.019)(1609.76)(0.288)*
DF
D=0576m=576mm
‘heres a leak in a horizontal 300-mam-diameter pipeline. Upstream Jater lows from a tank through 160 feet of 4 inches diameter pipe andth
Jikto gages 600 m apart showed a difference of 140 kPa. Downst charges into airas shown in Figure 20, The flow of water inthe pipe is 13
ite leak two gages 600 m apart showed a difference of 126 KPé ‘n= 0.013 and neglect minor losses. Determine the following:
{0005how much water is being lost from the pipe @) The velocity of flow in the pipein fps,
() The total head los in the pipe in feet,
(0) The pressure atthe top ofthe tank in pi
Solution

o =a oe Ke ao
k}-—u= 600m—4 Naxos
Les
a
ya2 AHHon
Iy= 509.876Q"
Sve the pipe is uniform and horizontal, the head lost between any to poi
equal tothe pressure headdifference.
ie Pip
Ine Pa OOP)
dpespacts
Q1=0.167 m/s Q=128/s= 03401 m/s
D4" = 0333 f= 101.6mm
n= 0.013,
lias
Wane 222a
Ps L= 160 feet = 4878 m
126
50987602 = 5-5
Q:= 0.159 m/s
[=a Ql
(Q.= 0.167 - 0.159 = 0,008 m'/s
Q= 8 Livecc
em 7-14
600-mm diameter pipe connects two reservoir whose difference in wal
face elevation is 48 m. The pipe is 3500 mlong and has the following pipe
tings: 2 globe valves, 4 shor radius elbows, 2 long radius elbows, and one
‘alvehalfopen. The values of loss factors forpipe fittings ingiveninTable
(0) Head lostin the pipe:
p= aa 10290LQ?
= 1024.13)(4878034007
@x016)"” The total head lost in the system is equalto the difference in elevation of the
HL = 194233 m= 637036 feet surfaces = 48 m
(0) Pressure in the tank: Hee oeeenaea
uation between A and C: 1. = 3500 + 2[850(0.6)+ 4[82(0.6)] + 22006)+ 117206)= 4064 m
Ech e
Bal 4 Ph 4 Z,-Ht= 2 + BE 426 tain RORMOIEY CD 2
at ae ae
OF2 +10 7a = SAO OH Q=0861m¥/s= 8611s
| 2 mersestetot ater
paasas thesyring:oftheigre shown thdrug had p=900kg/m?and n=0002Pe
‘The flow through the needle fs O4 m/s, Neglect head loss in the larger
Using the English unit for Manning’ Formula: hinder
| 107%

S=HL/L=HL/160
sas 8eomayensraye —
m-<sates
(@Determine the velocity at pointBin m/s.
(®) What the Reynolds number forthe flow in the needle.
(©) Determine the steady force F required to produce the given flow.
SEVEN
low in Pipes
" (@) Velocity at point B: ‘Colebrook Formula:

a rote ad
” Q_ 04x10%
mn A” soe /D, 2
(Reynolds Number: Fe -2og O08, 231
Tr ae Sof,
0.015, styl
251
002
ee inf S08"5000/7
R,= 91673 (laminar flow)
Solve forfby trial and error:
(0) Force F: f= 00515
Equation between A and B:
EqHL= Es Using the Moody Diagram,f= 0.05
2
Peg PA oe ta yh
2 ay” oy
| HL lyin the needle
‘Since the flow is laminar
J 64/Re=64/91673
f= 0.0658
HL= 0.0826(0.0698)(.02)(04 x10)?
(@.00025)*
HL=1889m
+ PA + 0-189
OO = BIE!
ee 5, Te
: pam 196681 Pa
| Force, F= py x Area of piston
= 196681 5 (0.01).
Force, F= 1545

7-36
Determine the friction factor for flow having a Reynolds number of5,
relative roughness (¢/@) of0.015 (transition zone)using Colebrook formula,

Sa
485m/s atx = 100 mm. Determinedischarge iff=.
Solution
‘The velocity at any pointis given by Eq. 7-8:
wont
: fe Discharge, Q= Av
= 707099)
Discharge, Q= 3:13 m/s
Pipe radius, 15 = 500min
Centerline velocity, = 5m/s
Velocity at r= 100 mm, = 4.85 m/s. oblem 7-18
Dil or sp. gr. 0.9 and dynamic viscosity 4 0.04 Pa-s flowsa the rate of 60 liters
ond through 50 m of 120-mm-diameter pipe. If the head lost is 6m,
line (a) the mean velocity of flow, (b) the type of flow, (c) the friction
or f, (the velocity atthe centerline of the pipe, (2) the shearstress atthe
allothepipe,and (the velocity 50 mm from the centerline ofthe pipe,
(0) Mean veloaty
2.
A” Fore
006
= 5.31 mys
(b) Type of flow
ren HBP. $340:121100009))
# 0.08
R= 14337> 2000 (turbulent flow)
(©) Friction factor
4 0.0826/1.Q?
= 20826(60)(0.06)?
(0.12)°
‘fe 001008
(@ Centerline velocity
vemn(l + 133972)
= 531[1 +1.33(0.01008)"]
=6.02 mys
= 46 -3.75(0.2562)
0=363m/s
Discharge, Q= Av
= $(0.757(0.63)
0010046531)? Discharge, Q » 1.6 mya
100009 8
"31.85 Pa
Using Eq.7-16: Pienaar
ee TD What the hydrate ads of rectangular ait duct 200 mm by 350 mn?
Solution
_9210«09)6y012) Hydraulic radu, R= A/P
wane40) Ro 20380
205235052
Velocity at SO mam From the centerline
un0-575 [2
ae ble 7= 2
ir at 1450 kPa abs and 100 °C flows in a 20-mm-diameter tube. what is the
= 602-575, SP | Ss m laminar flow rate? Use R = 287)/kg-*K, =217108Pas,
10000.9 ee
u=5.178 mys

Problem 7-19
‘The velocities in a 750-mm-diameter pipe are measured as 4.6 m/s and 4s
atr=0 and r= 100 mm, respectively. For turbulent flow, determine t
rate
Solution = —1450(1000)
wensff og 287(100+273)
p= 13.54 kg/m’
g 98
dente ee met
946-575, = 10.02)13.54)
Boao
‘v= 01608m/s
ff° soe
FadFowene 417
= § (00201603
= 0.000050 m*/s line = 0.0158). (2) Determine the head los per kilometer of pipe and ()
(= 0.0508 liysec. ie wallshearstress.

Problem 7=22 (@) Head loss pe Kilometer (L = 1000m)


Glycerin (ep. ge. = 1.26 and y= 1.49 Pa-s) flows through a rectangular -Av
300 mm by 450 mm atthe rate of160 lit/sec s D 2g
(@) Is the flow laminar or turbulent?
(6) Determine the head lost per kilometer length ofpipe. = 9.0158(1000) (3.2)?
on 29.81)
Solution y= 825 m
(@) For non-circular conduits;
pew BORD (b) Wall stress:

te Gia
A” seGde
030«0a8
ped ‘= 0302045
2(03+ 045)
R=009m
p= $(:185)(0.09)1000% 1.26)
149
R,» 960.75 <2000 (laminar) = (9810%0788)x8250.1
41000
(For aminar ow:
funnRos
soar oblem 7= 24
with sp. gr.0.95 flows at 200 lit/sec through a 500:m of 200-mm-diameter
it aE {f= 00225). Determine (a) the head loss and () the pressure dropifthe
ipe slopes down at10° in the direction offlow.
Dasa) e036
D=4R (for non-circular pipes)

y= QA779C1000) (1.185)*
036-2981)
y=3527m
ae
O+0th-ha BE +0+0

ney > Eq. (1)


fe
D 23
Energy equation between 1 and 2 datum at 2) D=aR=44APe
iH Bs ‘i
ga We ee A= FO1P- } 05
ee 23 A=0.005026m?
23 cancels cancels out P=2D.+ sD,
Since m= =o, oy 2
GE and and 22 = =(0.1)+ (0.06)
P= 0.50255 m,
P+ 9682-1162" 22 +0, 0.005026 00m
Dhee
Y 7
Pi 2 = 29.38 mof oll eee ae0
005
pi= 29.38(981 x 0.95) 01.9896 m/s =m)
pi pr 27381 kPa y= 90232050) 1.98962
004 2981)
we y=585m
Water flows through commercial steel annulus 30 m long as shown in Ing. @):
figure. Neglecting minor losses, estimate the reservoir level h jw 19898 5.85
‘maintain a flow of 10lit/sec. Assumef= 0.0232 2981)
h=6052m
lem 7-26
ind the approximate flow rate atwhich water will flow in a conduit shaped in
form of an equilateral triangleifthe head lost is 5 m per kilometer length.
cross-sectional area ofthe duct is 0.075 m?, Assumef= 0.0155.
Solution:
Energy equation between 1 and 2 (datum at2)
Ey-hy= Ex f
Hee ee yo
ape, aE Hoy
D=4R p= 3878(025) = 682<2000 laminar, OK
Reale Ae ae 0.000853
Ansasin =
x=0416m sau
R=0.075/130416)] Q= §(0:157(1.878)
R=0.06m Q=00392 mye
D= 40.06)
D=024m 2002)
5 = 20155(1000)
a _v? ie shown,
Malige
In the figure the
ee mass density f 920 kg/m?50-mandpipe is 60 mm in diameter. The Aud flowing has
dynamic viscosity of029 Pas. The pressure In
the enclosed tank is 200 kPa gage, Determine
ag
a ( Theaountand diction offow? the following
cee velocity offlow in the pipe?
0,075(1.232)
(©) The Reynolds Number oftheee flow?

Heavy fueloll flows fom A to B through a 1000-m horizontal 150-ma


steel pipe. The pressure at A is 1,050 kPa and at B is 95 kPa. The
viscosity is 0.000413 m/s and the specific gravity is 0.92. What isthe flow r
Solution
For uniform horizontal pipe, y=
yx 10235,
981x092
y= 11246 m.
Assuming laminar flow:
ya le
0?
te __ 32(0.000413)(1000)0
o=1878m
“Taking level 2 as the datum: 2%.
Energy Ei= 12m.
Energy,= 0+ 7 = 2 DOM 16m Try f= 003:
7g” Sanaa 7276 InEg.( 0030°=0471; 0=3.962 m/s
Since E> By, the flow is from 2 to 1,
3962(03)
39 0.00002, ees,
Energy equation between 2 and 1:
Ea- y= Es
22.16 — hy 12
y= 3016 mi

ya ag
‘Assuming laminar flow (Re-< 2000)
ta Newf=(00a
00006
|
ra020)6010
11s» 28029800
OD\GEIVOOST infg er v=
«471;ae
| Q* 0.00201 m/s Rem Soong 77700

W 8=
oF How, 1
eloalty iy oTTo
Reynolds Number, Re nm a f= 00199 (OK)
Reynolds Number,R= = 135.4 < 2000 (laminar flow, OK) In Eq. (1): 0.01992 = 0.471
v= 4.865 m/s

SerGiAea= O71rcomrte
Since our assumption is correct, then

Velocity of flow,
ore
= 403914865)
| Hee cise = 034 mys

| Problem 7-29
through a 300 aks fa slvent
7-30

Ofbecommercial sl (= ONDOII8 m/s¢ =y=8 N/m?ar comet 350he


|
|
Olt, with p = 950 kg/m? and v = 0.00002 m?/s, flows
diameter pipe that {s 100 m long with a head loss of 8 m. ¢/D = 0 pp (wagner BOODAE mn).
to convey 60 L/s, ifthe surface of one tank if 5 m
Wat soe mt
higher than the other,
| Galette the ow rate
| Solution
(iene
£(200) _v?
03 2(981)
= 0.0826/500.067 D = 0.461(0.057)15 = 0.24 m
De Re 5192
«/D= 000019
= 0.61 f°! f= 0.0874
‘The procedure has converged to the correct diameter of 240 mm.
An approximate formula for D is given as follows:
7s
D=066le1% 2)
ay
7
350
- sufS905") ++613%10°*0059'(
9816),
In Eq (1): D = 0461003)" = 02286
Inkq.Q): R= # = 5150
ine = = o0b0amT
sent2
Fede)
aaa 2,
f=0.03682 > News
InFg. (1): D = 0461203682)"= 0299
InFa.Q) R= 22 246 «529
mor 5S os
Solve for newfrom Eq.7-30:
fee ‘Tryf= 0.03,
(D=1,528(0.03)"5 = 07578
poss
fH 00874 Lets usethisf
Re=776366/(07578) = 1024500
£/D = 0,00026/0.7578 = 0.00343,
6 «= M8025)
Hote tel <a
fnoms westsore
Newf= 0.016 .
Oo 1.528(0.016)"9
= esa/oceans = 0.668.
(Oil with p 900 kg/m? and v = 0.00001 m?/s m flows at0.2 m/s through 200-
/D = 0.00026/0568 = 0000389 rum-diameter ast ironpip 600m long. Determine the head Ios.
1 69,
tas (0.000389 "|
0.000389 Solution
ra 162,225, 37 ) From Table 7-1, ¢= 0.26 mm
fx0018 (OK) Het eae
#/D=0.0013,
D= 152800165)=0671 m
Do6rimm
| Using the approximate formula: <2 7]008 = 637 m/s
| Fei) eel |
ese ACAI |
637(0.2) = 127,400
0.00001
From the moody diagram:
‘f0.0225
plem 7-32 By Haaland Formula:
| ‘Water is flowing through a 915 mm x 1220 mm rectangular conduitwéof
|
91.5 mand a head loss of 4.6 m. What isthe shearstress betweenthe
the pipe wall? undaeen'(ar) |
69 ‘0.0013 )'""

| ‘Solution
ita al
the
Fahydrate rds
A
oats
“Pp
p=Seo casi
From the Moody diagram, /= 0.02
Using Bq. 7-30
satel? -(*22)"]
wey")
vaseStat
panos
Uy he
oa
p= 102(80) 1.837
O15 2(9.81)

Wane wank wane ie Pressure drop forhorizontal pipe, Ap=yhy=p g hy


Pressure drop for horizontal pipe, Ap =17,818.5 Pa

om7=35
at sizeof pipe is requiredto carry 450 liters per second of water
‘of 3.4'm for 5000 m length? Assume friction factorf= 0.024. with a head

Compute the head loss and pressure drop in 80 m of horizontal 150


diameter asphalt castton pipe caryng war a 20% with mean
of 1.83 m/s. 3.5 = 110826(0.024(8000)(0.45)?
D’
Solution (D = 0895 m = 895 mm
From Table 7- 1, v= 1.005 « 104 Pa-s
From Table7- 2,¢= 0.12 mm
1.83(0:15) fater flows in a 300 mm x 400 mun rectangular conduit atthe rate of150 lit/sec.
11.005%10"° = 0.025, find the head loss per km length.
Re=273134
£2 222 ~ 0008
D” 150
ee a ee
the rate of 85:m*/s. What must be the difference in water-surface elevations
the two reservoirs? Neglect minor losses and assumef= 0.018,
0200
D=4703+04) ~ 043m For two reservoirs,the difference in elevation between the surfaces is equal
to the total head.
ny = OO
1082640.018)(2500)(85)*
sy
HL= 2.75 m (difference in elevation)
oblem 7-39
ner at 20° fo be pumped trough 9 kn of 20.diame wrought on
the pipe, Neglect minor losses and assumef= 0.12
Solution

Energy equationbetween and


Eiae
Bh Bisa y= Be Fe Be
Big sy ieee
ovons, SORMBIBIEOR"
{0.12)(30}Q’ - 9 949
(002° Fan
Q= 0.000179 m*/s
Q=0.1791/s
A pumpdraws 20 lit/sec of water from reservoir A to reservoir B as shown.
‘Assumingf= 0,02 forall pipes, compute the horsepower delivered by the pump.
land the pressures at points 1 and 2.

Wane wae We a

$s 582 (2982)"| ua
fe RRNDa
Qi = Qi 0.02 m/s

jpg 08280021200)0.02)? = 10442m


015
Energy equation between A and B:
oat y PA
PAE Pa tags + HA-hgm ee + PB a zy
wer required,quired, P= P== 0.06(9.81)(46)
Pox Qy Qy HL 0+0410=1033
- + iHA-10442=0+0+
# 60 i
Power required, P=27:1 Kilowatts HA=61475m

Power delivered by the pump (output power)


P=QyHA
= 0.02(9810)(61.475)
"= 12,061 Watts « (1 hp/746 Watts)
P1617 Hp.
Energy equation between A and @:
Eq- hm Ei

pi" 87.76 kPa


Pressure at @:
Energy equation between @ and B:
Ex- ham Ex
arg
Si ee
BL t-te os, Be cs Frictional head lost, = 226720"
Pa a eae
002)". P2 49-1042=0+0+60 Frito end ost, y= 2067C000)035
wg015)" “Gay
025)
Frictional head lost, ly= 3.89 m
[Energy equation between ® and @ (Datum atEl0)
a a
E\-HL-HE=E,
pe
Npaei gi
etepo repr a
em aser eniee nam
0+0+197-389-HE=0+0+50
HE= 143.11 m,
Se a ee e
ae Power, P=ee s (input power)
1ho meeweoeeae
water surface elevations ofreservoirs 1 and 2 are 197 m and 50 m, resp 210.59 kW
Determine the power generated by the turbine if it is 85% efficient? Power generated (output power)
minor losses. P=21059% 85%

Pump shown in the Figure draws water a reservoir and discharges It


toa nozzle at D. The length of pipe from thefromreservoir tothe pump is 150 m
id from thepump tothenozzle is 1500m. The pipe diameters before and after
‘Pump are 450mmand 600mm, respectively.
atmospheric pressure i 99 kPa absolute the vapor pressure Is 35 KPa.
f= 002 for both pipes. 21 = 4m. The and
pump is to operate such that the
ge willbe themaximumpossible.
Maximum discharge, Q= Av
= § (045) 6.692)
Maximum discharge, Q = 0.59 mys

743
‘Assume that57 liters per second ofol (p= 860 kg/m?) is pumped through a 300
fmm diameter pipeline of castiron. If each pump produces 685 kPa, how far
past can theybe placed? (Assume f= 0.031)
lution
Each pump must be spaced such that the head lost between any two
[pumps is equal tothe pressure head produced by each.

“ySolel
860981) rie
0.0826/L.Q?
y= Dezel"
D'
‘ang = 20826(0031)L(0.057) 2
(03)?
L=23,718 m=23718 km
Since the pump is above the water surface ofthe source tank, the p
atthe inet at) is always negative (vacuum), a
Asthe discharee
‘Gest rge increases,
ot Bont thenal
pressureBelow
a Bthdrops.venTovepor
avoidprone
cava 4300 mm diameter concrete pipe 3,600 m long, find the diameter ofa 300-m
ie equivalent pipe.Assume thefriction factorfbe the same forboth pipes.
q lution
Energy equation between A and B: Foran equioaent pipe system, the head loss andflaorate must be the same asthe
{sing absolutepressureand datum at A) ovginal pipe system.
=a > Eq.)
Tom hy > Eg. 2)
Using Manning's Formula for circular pipes;
-= 1029n? LQ?
>
big 1029n?Q?
TL” pee
For pipes in series, Q: = Q:= Q
For pipet:
Two ppcach 00 log areconnced nse, The oofwate i a2
y= Ramo?
the pipes is 150 lit/sec with a total frictional loss of 15m, If one pipe! Die?
Liao
diaeter of 200 min wat thdiameter
jaluncnepsoonpeboan eet of he abepipe. Negi eae
Solution 5)" 414.87 np @ > Ea, (1)
Forpipe 2:
= SE
2
Qi=Qr=015m%/s = 1029(2n,)7Q?
HL» int hp
De
0.0826(0.02)(300)0.15) | 0.0826(0.02)(300)(0.15)* 5 13603
Se 284.2)
be 03° D
D=0255m
D255 mm.

oblem 7-46
‘Two pipes 1 and 2 are in series. If the roughness coefficients 1=
diameter D; = 500 mm, find the diameter Ds ifthe slope of their
lines are to be the same.
we a (0.01 1.549 m

Be
= 2082s
Be = MORO}? 1495.64 Qe
(mreioe = 208240102)200)05)2 = 1652002
Q:= 053 m'/s 025
Q: 530 lit/sec
20526(0.02)2800N0.01)? «5.9459 m
Problem 7- 48 (CE May 2003)
pipe network consists of pipeline 1 from A to B, then at B itis connects
pipelines 2 and 3, where it merges againat Joint to form single pipeli
to point D. Pipelines 1,2 and 4 arein series connection whereas pip
3 are parallel to each other. If the rate of flow from A to B is 10 liters/s
‘assuming f = 0.02 forall pipes, Determine the flow in each pipe and the
headlost from A to D. Q)= 000231 m/s
Qs=231 ys
Ppelnes Lengih (my Diameter (men aeeAENs
2 (Q:=33230200231)
0.007687 m?/s
3
4
4
== 7.587 Ys
THL= n+ ha + ha)
HL = 1.549 + 1495.64(0.007687) + 0.0452
HL=1683m

to 640 m of 300 mm diameter pipe (pipe 2) and 640 m of 450 mm diameter


ipe (pipe 3) in parallel, which join again to a single 600 mm diameter line 1300
1032 for all pipes, determine the flow rate in
Q= Qe 10L/s
i= Qe 0.01 m/s
zsjl?
= Hos=
y
10122913
20.353 mys
2: = 0.26(0358) = 0.094 mis
(Q)=2756(0004) = 0259 mis
Qe= Qh = 0.353 m/s

2002)
For the pipe aystem shown in the Fig (5 for al pipes and the flow in
Pipe 4is 12 fs, Determine the following
(@ the head lost i pipe tin fet
(@) the total head lost in terms ofthe totaldischargeQ, where Qis in cl,
Qa Xe (©) total headlostin feet.
Q=+*Q > Eq. 2)
b= hfs 2 Fa. 0)
HL=if+hithi=1m > Eq (8)

Wi = 90826(0.082)820101" _ aoe770
(05)
= 0.0826(0082)(640)02" p961592
if
Cy
ry
re

Q=Qi=Qu= 1240/5
149) msva
Qn Ave Al English Version
= Fp: 18 wyanqujyye
46371? LQ? infect ater is flowing atthe rate of 300 lit/sec from A to E as shown in the figure.
HL» rs spute the flow in each pipe in lit/sec and the total head loss. Assumef=
Head lotin pipe 1: oralpipes
ity £83210.(1500)2)°
(24/1
Hla 5.59 feet
‘Total headlost in termsof :
‘Total Head Lost, HL= HL; + Hl + He
Total Flow = Qi = Qi=Q
| Hien 4.637eC
(0.015)? (1500)Q”
cay"
| HL= 00388
y= 4637(0015 20000"
(247197
Hla = 00518 Qt
(Hla Hts)
4.637(0015) (4000)Q,* |, 4.637(0.015)* (5000)2,"
8/12)" (2/12
Q)=03034 Qs
| [+QhQ-9)
03034 QQ

1g #897(0015)40001076720)"
48712)
Hla = 0.2826 @
0388 0? + 0.2826 + 0.0518
HI 13732.
(©) Total head lost:
‘Total headlost = 03732(12)!= 5374 feet
yon 2O2OMSNCOORE serge
(029°
2
px LOONIE51992
(03)
y= 20260 eon waoe.
0.45)"
Inq.)
1269Q# = 387102
= 17870,
Ink, @)
| LAATQU+
Qs=27470,
Qi Qs
| Inky @
| Tera = 1269 a.7e709+ 5102747097
| Q:= 24610
| ink@) 2 0)
Qi= 246104 + 17470, + Qi 03 + > 54.0)
2, = 0.0576 ms 54.0)
(Qu = 2.461(00576) = 0.1418 mys the Eq. i)
(Qs =1.747(0.0576) = 0.1006 mY/s Hae hn tha+hs=15 > Eq. (6)
| 0," 274710.0576)= 0.1582 m5
|| Check:= f y= ae
S082
O14 Qs+ Qs
| Fy Bhanesotto + 00576 «03 (0K ee soszsemnye0? Tomar
| oblem 7=52 (CE Board ee
;
0.0826(0.02)(¢
Igo Te - stage
‘The total headlost from A to E in the figure shownis 15 m.
Hectpie: ‘Assumef= 0,02 for all pipes. ies ae =322308

y= OOOO
o at =org

gx OORADEDIEHOIDS?
a =114702
\ 6): Solution
ae a seas esi7208-+ 11470 ‘Note:
criginalTheaddition
pipe inorderal fopipe tncrese
should be laid in parallel (not in series)
But Qs= 0, From Eq. (1) the capaciofthesystem f
15210307 +31.7292 > Eq. 6)
In Bq,(4
31.72Q2 = 922007 + 429702
ButQs= Qe
31,72Q2 = 7520?
Q)= 06490:
InEq. 2 Original pipe: Pipe ©
‘i= 0:7 06190: Capacity, = 1 2
Qi 1619Q: Head ost, H= O20"
InEq. (6) 0.08261.0)?
Fa8) niansssoy'+317208 Cole rug
15=88902
{Qs= 0.69(0411) =0267 mys= Qe Required capacity,Q:= 150)
Q;=0678 mys=0s Head lost = H
= 2:9826(2/3)f)L 0.50)
Check:Qr= Qs + Qs= 0.411 +0267 = 0678 (OK) D:
[H=H] (ince they are laid in parallel)
See
0.082610; _ eae 0.0826((2/3)/)L(1.5Q4)*
. is tobe reinforced with a new one whose coefficient o
‘An existing pipeline : Dy Dy
friction is 2/3 ofthe old one. Ifthe length ofthe new pipe is equalto that DY aus
‘ld one and the additional required capacity is 150% ofthe existing cap Dy
How big the new pipe should be compared to the ok! one, Use the D ive
‘Weisbach formula? D,
‘Therefore, D;=1,08 timesDy

With velocity of 1 m/s in the 200-mm-diameter pipe in the figure shown,


calculate the flow through the system and the head H required. Assumef= 002
{forall pipes and neglect minor losses.
[Qr= Aver
Q= §(02)0) = 0.0314 mys
hy .0924(0.02)(300)0.0318)?
26,00) 200) =153m
4 02
From Ea. @)
a= hn = 1.53
lig = 9.0826(0,02)(300)03 153m
eee
(2:= 00866 m/s
From Eq, (1)
i+ Qs
Qs 0.0314 + 0.0866
(Qs= 0118 mys

Ins
10.0826(0,02)(600)04"
03°
= 08660, > Eq. (6)
| From Eq. @):
= Qi+ Os
0.118 = Q, + 0.866,
2 =0.0632 mys
From
Os Eq. @) A

From Eq. (6)


He Intl thn
Gi53 20826(0.02)(900)(0.118)* , eae
.0826(0.02)(600)(0.0632)?
05) 03
H= 338m
fa mtmer
Aon
7-55 (CE November 1983) ne 10291104
Three of different lengths and diameters connected in series as ope
Macha100 ters id If the roughness coefficient n = 00
disregarding minor losses, determine: ona 1200134: 10)0167
(@). the head loss in each pipe,
(H)_ the diameter ofan equivalent single pipe that could replace all e=0201 m= 304m
pipes, and
(© draw the approximate EGL and HGL. (9 EGL and HGL:

1500 _-_.
"0 m= —._
_—_——=SSS

Qi=Qr=Qs= 0.16 m'/s


(@) Head loss in each pipe:
cae ninFormal
ye SSEELO
p= HDHD)y 1810000)(0.16)*
(035)
n= 483m “re 18000 300 mm-=1500m 250 mm = 800m
-10.2940.012) Pasaniore? 1 Problem 7-56 9(CE November99)
5 (03) ‘The installation shown in the Figure is designed for filling tank trucks with
p= 3498 m Water. The 10-inch line has an over-all length of 100 feet. The 6-inch line A is 10
long. The 10-inch line Bis 40 feet long. The Darcy-Weisbach factorf, equals
hp __1025(0.012)?(800)(06)"
02575
(0.02. Neglect minor losses. Determine the total discharge which can be
delivered by this system when all the gate valves ae fully open.
y= 49.33,
(© Bauivalent pipe:
Qc= 0.16 m/s
hg== HL4.89hn
Ie + ha49.39
4134.98 +h
he 89.14
‘Le = 1,800 + 1,500 + 800 = 4,100 m
= 2020) 0.810604?
Hea" (6712) 2216/12"
Hla = 0.161 Qa?

2A, 0.810604?
2g 3226/12)
2
2A
Ae ae 0408 0 a
Inq.)
.125 02-0161= 0400 +20
0125 0 + 0564 Q,= 0 72.0)
Energy equation between © and B:
Fi-Hlia~Hlaa™ Ep
0+0480-Hlna-Hilaa= 3 +020 > Fa.)
gg 0.02(40) 0.810605?
(67/12) 322(6/12)"
Hs = 0.644 Qs?
mae
322(6/13)

Q=Q+Q F901)
InFg. (8):
800.125 Q2- 0.644 Or = 0403 Qe +20
0:15 0 + 1.047 Qyt= 60 7.6)
Oeyeuua SubB. =Ba,
0+0+80-Hli-Hla4= SA +0+20 > Fa.)
Tigbsetoe
0.125 +
2g
0.564 Oy
oie antes
e Pr 0.8106Q7
Qn= 1.362 Qs > Fa. (6)
2g x3gt gt
Hs! LL & _ 0.021100) 0.8106?
D 2g (10/12) 32.2(10/12)*
Hya=01250
0.0826(0.025)750)0,"
oa5* aoe
Q=161229,
Q.+ 1.64220, =1
Q,=03785 m’/s
‘another 200-mm-diameter pipe (f= 0.02), 200 m long. Determine the diameter
fan equivalent single pipe oflength 350 m and f= 0025 that could replace tap = 20828(002)(600)(03785)2
0 i 015"
perry HL=1870m
Solution Pil
aaroya or equivalent pipe:
aaa r=tm'ys
For the originalpipe system (two pipes in series Hle= Hl 1870m
Q=Qi=Qi=1 m/s Hg 2082HOOSN7EONN? _ ry
HL hn + ha ictae a
a 010826(0.015)(150)(1)* , 0.0826(0.02)(200)(1)* De= 0.218 m = 218 mm
ican Oa
FI 2B
aa
Forthe equivalent pipe: Inthe figure shown below, it is desired to pump 3411,000lit/day of water from
Qcolm/s stream toa pool. Ifthe combined pump and motor efficiency is 70%, calculate
Hie * 122281 m : the following:
010826(0.025)(350)(1)? _ (@) total pumping head in meters,
He De a () the powerrequired by the pump, and
Djs 026mm «726 mo (©, the monthly power cost ifelectricity rate is
the pump operates for 24 hours and take P6.00 per KW-r. Assume that
1 month = 30 days,
Tengih] Diameter Hazen tn]
‘A 150-mm-diameter pipe (f = 0.02), 600 m long, is in parallel with z 2h {gnn) G
‘diameter pipe (f= 0.025) 750 m long, Determine the diameter of an equiv a
single pipeoflength 70m andf~= 0.015 thatcould replace the two pipes ua
Solution
SetQ=1m'/s
Forthe given pipes (wo pipes inparallel):
+ Qie1
In=ha
Powerrequired by the pump (Inputpower:
Hla tn+hn > Eo.) Pes QyHA
Q=O+Q 383.2) = 012945(0.81)69.5)
ha=ho 28) = 27.98 Kilowatts
Hazen Williams Formula: Power input= P,/ Efficiency
y= 1067 =2708 (070
Power input = 38.614 kilowatts
Power cost:
‘Cost = Power input, KW x Time in hours x Power rate per kW-hr
= 38.614 kW x (20 «24 he)x (P6.00/ kW-he)
Cost = 166,812.48
{Qi 311,000 itday x (day/24 hes) «(1 he/3600sec) _ ——
Qh= 39.48 L/s = 0.03948 m/s fern
How many liters per second of water must the pump shown supply when the
Solving for Qs and Qs flow needed in the915-mm-diameter pipe is 1314 m/s? Assumef= 0.017forall.
From Eq. @) pipes.
rae 1248 10671915)0,""
10.67(1,525);" ,
Cig) (0a)*™ C40)0.15)"
Q= 07870:
From Eq. 2):
[Q=Q:+Qs
(0.03948 = Q: + 0.7870:
(Q:= 0.0221 m’/s
From Ea.(1): a
10.67(1,525)(0.03938)'* 1067(1,525)(0.0221)'
Hla = so 100)" a? (02) * B0C10) 2) (0.2)
HLq= 26801m
Energy equation between A and B:
Exc2 HLqa+ HA = Ex a 4 2B oy
Pag PA + q-Hl tHA= 2
Tye 2
0+0+47-26801+HA=0+0+50
HA=693m > Total pumping head
‘Atjuneton ‘
Inflow = Outflow
Q+Q= 2,405
Q,+035= 1.314 + 0.142
Qi= 1.106 m/s
11,106 Liters persecond
a
‘The turbine shown is located in the 350 mm-diameter line. If the turbine
efficiency is 90%, determine its output power in kilowatts.

qe 13iemis e

Q:=1314 m/s
ig 20820.017)2MOVESIAY 99 op
ae
Elevation B= 6.1 + ha
Elevation B’ = 6.1+922= 1532m.
hy Elev. B’ ~ Elev. C= 1532-122
y= 312
p= 2OS2HOOITIAZONS? 539
0406
= 0182 m/s
liq Elev, D- Elev. 5°
Iyg=246-1532=9.28
10826(0.017)(1829)2.7
x= 008° 29.28
a
Powerinput = Qi HE
= 0.2787(981)(6.425)
Power input = 14.83 kilowatts
Power output = Power input x Eficleney
= 14,83 x 0,90,
Power output » 13.347 kilowatts
~O eo
‘A 1200-mmtiameter concrete pipe 1.800 m long caries 1.5 m?/s from
reservoir A, whose water surface is at elevation 50 m, and discharges into two
‘concrete pipes, each 1350 m long and 750 mm in diameter. One of the 750-mim-
diameter pipe discharges into feservoir B in which the water surface Is at
elevation 44 m, Determine the elevation of the watersurface of reservoir Cinto
Which the other 750- mm-diameter pipe is flowing. Assume f= 0.02 for all
Q=023m/s pipes.
Se ee ‘Solution
TSO
Elev. D’ = Blew. C+ hg = 280+ 87.75
Elev. D’ =367.75m
fig Hie, D~ Elev, B36775-330
a= 775m 2
q~ SORSORYCEA wa7.75
1 00887 m/s
Atjunction D:

Sas
[Inflow = Outflow]
isascha
ON fy =2.18m
£.0826(0.018)1220)(0.2787)? 965m
n= nes Elev.P= Elev. A~/in = 50-218
Elev. P= 4782 m.
“hy + HE = Blew. A ~ Elev. D’
26825 + HE = 400- 367.75 p= Blev, P~Elev, B= 4782-44
HE=5425m hn=3.82m.
a onsen3500s" mam n= 24m
.ss(0.02\1500) yee
= - 0.6375m'/ is. Q1=1399m/s
05"

Now=Outfow] hanan 0240029410000;


(045)
=O: Os
135 =0.6575+0) (Q2=0488 m'/s
Q)=07125m/s Pa
(0.0826(0.02)(1-350(0.7125)* 4.77 Qs" 1.399 0.488 = 0,911 mys
Be 075
saa
ae
Elev. C= Elev. P’~ hp
bwcoe s5- MEOONONOA 554m
0.0826(0.018) (0.911)?
a comme
Hoge
“Three reservoirs A, B, and C are connected respectively with pipes 1, 2,
joining at a common junction P whose elevationis 366 m. Reservoir A
a
Determine the flow in each pipe in the figure shown and the elevation of
clevation 933 m and reservoir is at elevation 844 m. The properties of reservoir Cifthe inflow to reservoir A is 515 Lit/sec.
pipe are as follows: Ly = 1500 m, D; = 600 mm fi = 0.02; Ia = 1000 m, Ds*
‘mum,f= 0,025; Ls = 900 m, Ds = 500 mm, f= 0.018,A pressure gage aur
P reads 4950 kPa. What is the flow in pipe 3 in m’/s and the el
reservoir C.
Elev. F’= Eley, E"+ ha = 84.23 +737
Elev. 916m
‘ig = Elev. F’~ lev. D= 91.6 -90
hig = 1.6 m
p= PO £0.0826(00
AD3)(30
IB 0)252
Ia
m 0.5 cd
2)" 0.199 m/s > Flow in pipe 3
‘Atjunetion F:
{Inflow = Outflow)
= Qs4Qs
=0247+ 0.199
Qu= 0.446 mys > Flow in pipe 4
Jip = 20826(0.03)(200)(0486)? 5 oy
045°
Bley C= Bley F tig= 0164801
Elev. C= 99.61 m
2
j= SOONGNOST«475
lem 7-65
Elev. E'= Elev. A+ hy =80+ 4.23 Determine the flow in each pipe in the three reservoirs shown.
Hey. E'= 8423m
ig Elev. B- Elev. =90 - 84.23
I= 577m 5
ypu 2082)CODD? a 5.77
045
Q1=0.268 mY/s > Flow in pipe 2
‘Atjunetion E:
[Inflow =Outflow]
+= Qi
Qs=0515 - 0.268
= 0247mys > Flow in pipe
ie = 0:0826(0.03(900)(0.247)? 737m.
yx SemIHOI = e292
_-910826(0.03)(4000)0," 5
hp eee 30.25 Qi
Direction of flow:
‘The flow in each pipe is due to gravity. The flow in pipe 1 is ob
away from reservoir A and the flow in pipe is towardsreservoir C,
flow in pipe 2 is either away or towards reservoir B, To deter
direction of Qx, assume Qz = 0, then lz = 0 and the EGL for p
horizontal,

= 200 r orsonser
ha=1322Q2"x-30; Qr=0087/e=20
hp=2025Q2= 70-1 Qy=018270-2
Atjunction P:
Hlefiow = Outow)}
Qr+Q=Q)
00887./r + 0.087J5=30 = 0182./70=2
Vi +1.482V¥-30 =3.1V70-x square both sides
x +2964 Ux VE=30 +2.196(x-30)= 9.6170 - x)
2.964 Jz=30 = 738.58-12.806x square both sides
{8.785(3)(x - 30) = 545,500 ~ 18,9175 + 164.2
155215s?-
pa18653x + 545,500 = 0
p= 30.25Q5= 40; Qs=1.15m"/s
‘Since Qs > Qu the supply from reservoir A is not enough B.for p (2, = 0058750287 = 0416 mye
‘Therefore, cisawayfom reservoir Band P*is below reservoir (Qe 0.087Y50287=30 0,392 mie
(Qs 0.182 V70-B0287 = 0,808 mis
1+
DateQ=o
0392 =0808
fa08 0808 (OX)

Mn" 1275 Q2= 15; Q,= 1.085 m/s


p= 306 Q= 25; Qs= 0.286 m/s
w= 19579 QE = 45; Qe=0.048 m/s
Qs+ "0334 m/s
Since Qi > (Qs + Qy), the flow from pipe 1
Pipes 3 and 4. Therefore, sis towards reservisoirmore than enough to supply
B and is above reservolr

p= 208201002)600923" «1975.9.
aaah
p= LE2H002)20002" «1 54992
02°
= 2.0826(0.02)(450)0,7 =306. 02
ie 03
Mo0.08a26(0n.02)a(9003" 9579.98
CHAPTER SEVEN 4
Fluid love in Pipes.
Problem 7-67,
‘The pipe network shown in the figure represents a spray rinse system. Find 2
flow in each pipe. Assume C; = 120 for all pipes. %
canis

arms aims aims tm


‘Solution
‘Using Hazen-Williams formula:
In ™ 12.75 Q) Qr=0.28/x Ke api
Ig= 1589 Q2= 15-3; Qe 00254Vi5=¥ 1
hy=206Q?=40-2; Qu 00572/i0=% Pieri Ente
lg=19579Q#=60-x, Qu 0.00715 /60== PITT nd meeeaitis alas and
‘Atjunetion P:
10.67(400) 520 = = Ker=
Kay = TOMO)
Qi=Q+QrQe Tanang 1" Kee Ker Kew
Q1-Q:-Q-Qe=0
0.28 Vx - 0.0254 J15—x - 0.0572 V40=x - 0.00715 Y@-x = 0 3m
Solvexby trial and error
‘r= 4055
= 028JETS ~04994 mio
Qr= 0.0254 VI5-3.055 0.0878 m'/s
(Qs 00572 /40=3055 =03477 mij
(Qu 0.00715460-3055 = 0.0539mye
Checke
0.4894 = 0.0878 + 0.3477 + 0.0539
0.4894= 0.4894 (OK)
‘SEVEN
“Fluid Flow in Pipes
jon to be applied: Qer= 005+ 0014-0 = 0.068
ere wee 188
on EHO Qre=0.1-0.014 = 0.086
Qo=005+= om
Tabula Vas Ger=00s-0= 008
om -5 ‘Second Cycle (using the above corrected flow):
010 54 j K a
0:10
‘010
ard
4534 321 0.2119 85.845
‘010 454 520 [01681 125671
005 4075 321
320__| 001 40.713
005 00579 72139
2516 13 3a ono 50.688
05 2516 0.0860 39887
005 0075 0.0650 50.268
0.0500" 25.155
010500 25.155
=. 2K
185EKQ.=
Correction:
Loop I: AB, BG, GH, HA: ‘Loop Ii AB, BG, GH, Ha:
16364+73A5— 45-2647 _ yong
ee 18191 + 7.062 - 3587-23699 0.00329
T85(65.845+72139+ 0713+ 125671)
IBC, CF, FG, GB:
er ee Shy2088-ASA-735 «yoy Loop Il: BC, CF, FG, GB:
TB5(45 4075 ace 5.778+3.217 ~343~7.062
T5{(50.684 +50.264-+3988+772.135) ~ 0.0038
Ill: CD,DE, EF, FC:
Ee
ase 1.258 + 2.038 - 1.258 - 2.038 Loop Ill: CD, DE, EF, FC:
4075 one 4.2564 2038-3.217-1.258 0.00851
TR5(25155+ 4075+ 25155+
5026)
Corrected flow:
Qu=02+00119=02119
Onc =0.1 + 00119- 0.4 = 0.0979
Qcu=0.1 -0.0119=0.0881
Qu.=02-00119 0.1881
Gre04 +0014=0.114
yn 0:2119+ 0.00329 = 0.21519
Se =o + 000529 OS= m/s 009799 m/s 7-68
‘Qeur= 0.0881 - 0.00329 = 0.08481 m/s OU having sp. gr. oF 0.9 and viscosity v = 0.0002 m/s flows upward through an
‘Qin = 0.1881 - 0.00329 = 0.18881 m/s Welined pipe as shown, Determine the flow rate.
Qnc = 0.114 + 0.0038 = 0.1178 m/s
Qcy= 0.064 + 0.0038 -0.00451= 0.06329 m/s Ans:7.64 L/s
Ore = 0.086 0.0088 = 0.0822 m/s
(Qc = 0.05 + 0.00851 = 0.05451 m/s
Qor = 0.05 + 0.00451 = 0.05451 m/s
(Qtr0105 0.00451 = 0.04549 m/s

oasis

Problem 7 - 69
Gasoline at 20°C (sp. gr. 0.719, = 0.000292 Pas) flows at the rate of 2 L/s
through a pipe having an inside diameterof 60 mm. Determine the Reynolds
umber.
‘Ans: 104,400
Problem 7 - 70
‘If 140 L/sof oll lowsthrough the system shown, determine the total head lost
between points B and C.
Ans: 118m
tube 15 mlong is toconnec two large tanks whose differencein water level
“Whats the yrs rada ofrectangular duct310m by 500m 2m. The tube sto carry 5L/s of low. What minimum size of tube is necessary
toassure laminar flow condition? i
Problem 7-72 Ans:S5mm
‘Air at 1500 kPa absolute flows in a 25-mm-diameter tube. What is Problem 7-76
‘maximum laminar flow rate. Use density of air = 14.04 kg/m?and x 0.00008
Pave The water system in a suburban area consists of an old 200-mm pipeline 760 m
long which conveys water from a pump to a reservoir whose water surface {s
107 m higher than the pump. Water is pumpe d at the rate of 0.07 m3/s,
Determine the horsepower saved by replacing the old
Problem 7-73 ase pipe. Assume the valueoff as equal to 0.033 and 0,022,pipe with a new 250-mm,
respecti
1nd new pipes. Neglect losses of head except friction head, vely,forthe old
Water district which serves
wl « sububan communiy
supply the aes rom an exitingneds rsrvle,
an
tration main which
ine wl be 83 le ong. ‘Ans: 2282 hp.
Problem 7-77
Paint issues from the tank in shown at Q = 45 f/h. Assume laminar flow.
it eghect BRITO
‘The proposal to bulla singe ine of new sot clean at ron pipe. D
47 ()ibasbued tobe
Detrmine he0.021. The presentJemand
minimumameter is MCD.be we
pipe thatcould Ans: 84

eres
particular installation.
@ What isthe velocity of flow in the pipe.

re ne
eae aee 7-4
Prse a
ae
ae
‘pressure drop overa 30-m length ofpipe? (0) Determine the kinematic viscosityofthe paintin ft/s
‘Ans;0.002544
‘capillary tube of inside diameter 6 mm connects tan A and open cont
shownin the Figure. The liquid in A, B, and capillary CD is water
“specific weight of 9780 N/m and viscosity of 0.0008 kg/(ms). The
345 kPo. Assume laminar flow.
{@) Determine thehead loss inthe pipe in termsofthe discharge Q.
() Determine the discharge in L/s. ‘Two types of conduit are use to convey water, the open channel and the
pressure condult (pie)which was dicussed in Chapter 7. An open chanel
‘one in which the stream is not completely enclosed by solid boundaries and
(0) Determine the Reynolds Number. therfore asa fee surface subjected only to atmospheric pressure The ow fn
sucha chanpel i eased notby some external hen, but rather by the gravity
componentalong the slope of the channel. In an open channel Gow, Ue
hydraulic grade line is colnlden t with the steam surface since the presnire at
the surface is atmospheric. The flow in open channels may either be uniform or
soneunform,

‘The specific energy (H) is defined as the energy per unit weight relative to the
bottom of the channel. Itis given by:
‘here § is the slope of the energy grade line and Lis the length or run.
hhead loss balances theloss in height ofthe channel.
From Darey.Weisbach elation:
ne iT heDee
whereD =4R
ee
aR 2
hebree
LO fot
alee
eee hy nS
ep

2
(ns)?

For a given channel shape and bottom roughness, the quantity (8x/f
constant and can be denoted by C. The equation becomes,

‘These equations are called the Chézy formulas, first developed by the F
‘engineer Antoine Chézy in 1769, The quantity C is called the Chézy
varies from about 30 m"3/s for small rough channels to 90 m/s fo
smooth channels.
|A great deal of hydraulic researchers correlated C with roughness, shap
slope of various open channels. Among them were Ganguillet and Kut
11869, Manning in1889, Bazin in 1897, and Powell in 1950, q
n= roughnesscoeffident SeeTable 8-1
‘m= Bazincoeficient, SeeTable8-2
R=hydraulicradius
‘e= roughness in meter
Re= Reynoldsnumber
= slope ofenergy gadeline
UNIFORM FLOW (S = S,)
“Table 8-4: Values of tobe used with Manning Forma ‘The simplestofall penchanel problem theuniform flow condition. For the
‘ow to be unitor, the vlc, depth of flow, and coseseconal tea offot
| «condit
aanpi nt ofte seam must be constant le =e» dredsAy Ay For is
ion the stream susface i parallel tothe channel bed
and the energy grade
Sects 10] “bars line i parallel tothe stream surface, and
Seman taee ‘0010-0014 Tine Ss equal to the slope ofthechannelbed Sy
aiid sever pe aio {9912
italfurs, soo ‘aout oats =
Gone, precastsutoce
Cement mortar oir 001s
oot] |a0is wins Disa Sree =a Seen =f
5
arkures, unplaned 001s
dont 0007 .
‘Gomanriay riage ‘oni
“rere, once 12 ome wa
rk wih cementmora ‘012 pax
Gastronnew aaa
Sige) EGR) (CIEE

Cenentbe Sac
Riveed see 087
‘yi the unit weightofthe liquid, R is the hydraulic radius, and 5 sf 7 derive Walp ppertoes foe most ficientsectors, nirinase tate
‘lope ofthe EGL, and for wniform flow or for S<1/10(0<57*),S= S. with the cross-sectional area constant.
Rectangular Section
NORMAL DEPTH
‘The normal depth d, isthe depth at which uniform flow will occur in an op Perimeter, P= b+ 24
channel. Normal depth may be determined from Chézy formula with § = ‘Area, A = bd
“The resulting equation usually requires a trial-and-error solution. be asd
A
paAvra
a
MOST EFFICIENT CROSS SECTIONS (MES)
{Also known as the most economical sections, these are sections which, f0
given slope S, channel cross-sectional area A, and roughness 1, the rat
discharge isa maximum.
From Manning formula,
gz al ens
It ean be seen that with A,1, and S constant, Q is maximum when the hyd
radius R is maximum, and since R= A/P, then R is maximum if is minimun
‘Therefore the most efficent section is the one that have the least wet

Tet us supp hak you aeTherequired to design a relanguar canal to have 9 Goss]
‘sectional area of 8 sq). possible dimensions (wat » depth)ae a flows:
‘b= Bm, d= im;perimeter
Fleer=.
(eee?
pee
An bd + Pad tan9 (d)] 2 3
oe2dsecO-2dtan 0+ 2d tand *
A=bi+dtand te ee
Mdm A-dtand
t= 4 -dtano > Ea. Q)
aay ‘Therefore, the mosefficent trapezoidal section (including the rectangle) has its
top with()equal tothe sum of the sides (23), whichis« proportion fora half
Pa aund+2sc® j hexagon,
> 19,09) ‘
2p i From Eq. (2)
2 vondaato + aiecd1an0=0
BA smo eamctn0
A
e =2sec8 -tan8
A= Qsec 0 -tan0)d
InEg.@) ‘This shows that the best of all efficient trapezoidal section isthe half-regular
px (2sec8: 2 + @secO-tan6)d ‘hexagon (allsides areequal).
P=2@secO-tand}d > Ea. (4)
Triangular Sections
Re AP _ @sec6~tano)d?

From the figure:


mb
pape2y arensi Perimeter, P = 2d sec (0/2)
A Aw Vax 2d tan (0/2) xd
Aréun(/2
but b= 4 dan tano/2)=
b=24sec0-dtanO-dtan z
b=2dsec0-24tan mo EE
‘VELOCITY DISTRIBUTION IN OPEN CHANNEL
‘The velocity distributionin wide openchannels is given bythe expression:

where: y= depth ofwater in channel nm.


‘= velocity ata distancefrom channel bed
= von Karman constant havinga value of0.40forclear water
(orsedimentladen waterits value may be aslow as 0.20)
= meanvelocity of flow
‘S=slopeofthe EG. i

ALTERNATE STAGES OF FLOW


‘The channel shown in the figure below carries water ata depth of dand a mean
velocity of V.
en D> Sat ‘Thetotal specific energy in thechannel isH =
z= 45°
solvingfor0 = JF)
and the discharge is, Q= A J2g(H=@)
“Therefore, the most efficient triangular sectionis the 90° V-notch.
If the equation will be plotted (as shown in the igure atthe right side), itcan be
seen that when d= 0, Q=0and when d= H, Q= and by substituting values of
din terms of H we can establish a curve.

AgrT t
Circular Sections
acationice ae e e
te a
e n
h
‘of the diameter. See PROBLEM 8 23.
fy gencscrge icyte ese ied
depths arecalled altersate stages, andare spoken as the frnguil or

CRITICAL DEPTH, d.
Fromthe figure shown in the previous page tha there iscertain depth d
for a given total specific energy H, the discharge is maximum, This
‘called the critical depth and is defined as the depth at whichfora given total
‘ischarge i maximum, or conversely, the dept atwhichforageenflow, the
‘nergy 1s minimum. is value ean be obtained by differentiating the follow
equation: Replacingq= of
#= (od?/s
ana ae
Critical Depth on Rectangular Section 2 (Froude number, F)
Forrectangularchannel, thedischarge per meter widthcanbeexpressed as:
9-4 This shows thatthe Froude number for critical depth is equal to 1.
where: =unitflowinm/spermeterwidthofcanal= of
aia.
‘uniform flow atcritical depth is known as the
Th eqn ein slope x de etaiguercamel
he slope quired to

Critical Depth on Any Section


For any section, the critical depth can be computed by the following der NON-UNIFORM OR VARIED FLOW ($+S,)
formula: Uniform flow rarely occurs in natural
‘width, and slope along the channel, Thestreams
Mannin
because of changes in depth,
can be applied to non-uniform flow with accuracygdepend
equation for uniform flow
reach L taken. ‘Thus along stream should be divided intoentseveral
on the length of
reaches of
varying length such thatthe change in depth is roughly the same within each
reach,

mes
wR3:
secton ome
From the figure shown:
ce thiesuesae
Be ae SL
SL-SL= (Boab a) ;
‘where A and/or B, if variable, must be expressed in terms of d.
‘Hydraulicjumpisan abrupt increase in depth ofrapidly lowing water. Flow at
the jump changes from a supercrit
accompanying lossofkinetic energy. ical to a subcritical stage with en
‘A hydraulic jumpis the only means by which
Jess than critical to greater than critical toa the depth of flow canchange from
channel.

He Evai +

LL= lengthofeach
S,= slope ofchannel bed >
5:6 52 slope oftheenergygradeline atsections 1 &2,
respectively,computed usingManning'sformula

‘Consider thatina shorintervaloftime, themass mshown in Figure 8-2 moves


from section I to section2. In changing postions water losses momentum
Theunbalancedforce (F-F)must be equal tothe, therateofchange of momentum
FE
‘Momentum, F#=me = mon
Fam2=2h, where m= W/g
pa ot
ty
pu DeVol O=my
SCRE
butVol/t= Q, then;

Qe,
pina)
pada
a
fa
tWA 4,

2 $l did

where 9 unit flow = Q/b oF Q= 96


‘This equation can be rearranged to an explicit expression for the di
before and after thejump as ae oe

For rectangular sections:


Qn ab
Anbd
hna/2
b=be=d
2
biss/2)- bis/2)= = (
‘of the inability of liquids to resist shearing stress, the free surf
ly uniform flow is always normal to the resultant of the forces acting on
"Water. When water flows in a curved path, its surface assumes a
‘normal tothe resultant of the forces of gravity and radialacceleration. flows uniformly in a rectangular, concrete, open channel that is 10 m,
4 depth of 3m. The channel slope is 0.0025. Using n = 0.013, find the
velocity, flow rate, and the boundary shear stress,
Solution
ve Laisa
A= 108)=30m? s
P=10+3Q)=16m?
R=A/P
R=30/16=1875m
a 1875"0.0025)7Wa
aon
= 5.648 mys
TO Aoy
Q= 305.848)
Q= 17545 myo
fe=1R5]
+=9810(1.875)0.0025)
1= 4598Pa
z
Determine the uniform flow through a trapezoidal concrete lined
side slope of 3H to 4V and bottom width of 2 m ifthe depth of flowcanal having,
is 2m. The
‘channel is laid on a slope of m per 2 kilometer. Use n = 0.013.
on Len sta
P=2+205)=7m reAA
R=A/P=7/7 1008
Reim ae
$= S,= 3/2000
soon n-onsaim
Q= Mars anmoon5) 20038 <10sseay 52
Q= 2085 mo ‘$= 0.00084. > Siope
(0) Shearingstress:
Blam 8=3 May 2003 ‘Saks
vl
-981(0.8868)(0.00094)
{A trapezoidal channel has a bottom width of 6 m flow and side slopes of 2 hori 0,00818 kPa
tol vertical, Ifthe depth of flow is 1.2 m and the is 20.40 m?/sec, Sy= 8.18 Pa
(a), Compute the specific energy.
(#). Compute the slope ofthe channel if = 0.014.
()__Compute the average shearing stress at the boundary.
‘Solution ‘The section of a storm drain tunnel is as shown. During the heavy storm, the
water surface is 2.5 m above thesemicircular section. If n = 0.02and the slope of
| ‘the channel is 0.009, calculate thedischarge.
| ar,
| Q=204m/s
| ‘A= 6(.2) + ¥5(24)(1.2) x2
| -A%10.08 me
| fo= =22)
| °* 7008
| = 20238 m/s
@ Speicenergy= 5 +4
= 20038? 44
2981)
Specific energy, H=1409m
4rA=470,- frtsino
Q=AL Rens sin =0.785
A=55(25)+ 425"
A= 1620? SolveO by trial and error:
O= 101.185"
P=25(2)+ 150) + 5(1.25)
Pat1sm
R=A/P= 1620/1193 101.1854(¢/180")- sin 101.1852) = 0.785 (OK)
R=1358m Then,
2 1620 hy (.358)-7(0.009)72 pm 30.6960" 101.185")
180°
P=271m
R=A/P=0:9806/271
R= 0365 m
coefficient = 0.012, was found to be 7/8full. Determinethedischarge Q= (9806)fe (03657/40.000)/2
the pipe.
Qr13%mys
Solution
es!
rectangular, concretechannel, 15 m wide is to carry wateratthe rate 22m/s,
If the channel slope is 0.00025, determine the normal depth of flow. Use
roughness coefficient n = 0.013.
Solution
Q-at Res
Amst
Re A/P= 154/(15+ 26)
220154 ods [eS] s (0.90025)
Qn adRnse
Am FAnat 154_}*
1.206 «if 294]
= Fx Fx 067
A=09896m? Solve ford bytrial and error,
d= 1187 m
AsAwa-A= far
+ AY= Arcee Aries
“feqiired vertical drop ofthe channel bottom per kilometer of length.
0013.
eae, ea For concrete lining, n= 0,013,
onal gsi
D=2r 1,60; r= 0.80 m
For semi-circular section, R= r/2
Q=¥4m(08)%% ay (48)?(hy)?
Q= 1.187 mys

Water flowsin a triangular V-notch steel channel, ata


velocity of1.2 m/s, Find the normal d
soon Solution
=a rose vel Ragin
in
pee
A= 35)(4)- 422)
‘A YQu tanoo"yd
‘Am @tan60?
Pa35+2+ Pa+15 P= 24ec60"
P=9828m R=A/P
R=A/P= 12/9828 = dtan60"/2sec60?
ReiZim R= 04g
12> ahr 04880):4@0017)1/2
Q= (12)ahs (1.22179 812 =25 ‘d= 0,601 m (normal depth)
5 0000562 oF 0562mperkilometerlength
8-10 (CE November 2001)
trapezoid ial canall with sides making an angle of 35° to the horizontal
‘base width of 6 m. The channel is laid on a slope of0.002 with n = 0.013.
discharge of75 m/s, what isthe normal depth?
Substitute bin @2) to (ly
Solution (729- 2828d)d + 2m 697
Am 64+ Yd)ADBA)2 7294 ~ 18288 6:97
‘Ax 64+ 1428E 1.828. -7.29d +697 «0
P=6+ 2017434)
Pm 6+ 3486
pe AA, Steaaadt
64+ 142d?
P 6e3d8ed 4 300-mmediamete
| g-atrese tequed to cay waterthee O N aye BePe 50 a
| msesa
|
f
= +

2/3.
ae Gar TEBE) ABT
1_(6a+r428d?
J ye
Cs
flow. Useroughness coefficientn= 0.013

ss
=A} pnsiay
Cage

.
004=x hyRY(2
| By trial and error, d= 2m AR= 0011627 &
Aarne oon
;
sit #7/P=000125 Sag)
/P2/ = 0.001627 raise both sides

‘A trapezoidal flume with side slope of 1:1 carries 225of (0/s of water at am
velocity of 3 ft/s. The canal is laid on a slope 0.0002 with n = 00
Determine the normal depth ofthe canal in fet.
‘Solution
Q=Av
225" A@)
Am 75 = 697 m?
Anbi+e=697 > E50)
oe Beg
p= 3ft/s=0.915 m/s From the igure:
1 R¥(0.0002)"
0.915 —— A Aner Arto
m9 ain
Aa Yersing
R=0956m.
‘A=¥ir @yfe—sind) = 0.09125feain6) For the mainchannel
x -02570 aan AL RISE
p= SE
A=172)8.6) + 120.6) + 40.641)62)
From Eq (1) A= 6812 m8
fo031250 —sinMP 6 ootas P= 3.6(8/1)+ 12 26(08/1) = 25:86 me?
025 rh R= A/P = 68.12/25,86
(@ xis -sino)? ee R=2634m
e Quan (68:12) « ar (2:684)?/9(0.0008)¥2
Qeun™ 90.01 m/s
Solve for@ by tial and error:
o= 14046" For thefod plane:
Then; ram=A Reg
0s 9/2) = 025 -)/0.25 A=00) = 60ne
‘cos (140.46/2)(0.25) = 0.25 -d P=60+1=61m
=01654m
‘R= 60/61 = 0.9836 m

‘Problem 6-13 (CE Board November 1988) Qe = (60) hg(0.98867(0.0005)2


‘A flood occurs in a main channel having a trapezoid al section (side (Qnue = 20557m/s
both sides: 2H to 1V) and base width of 12 m. The depth of flow in this
an almost horizontal plane
{8.360 m and the flod spills out overflood on one
60 m with an overflow
the main channel. The width ofthe andplaintwoistimes
Total Qusu.0 20587+ Quine
flow, Q= +Quin
foflm. If= 0025 forthe main channel as large forthe 0 O=110367mys
‘section, estimate the discharge ifthe be slope for both is 0.00030.
Solution
‘The water cross-sectionalare inthe figure,
J teranmvas n= 002s —ap shown is 1836 m, and the wetted
is 1256 m. If the flow ls 45.69,
‘m/s and n= 0.015, findthe slope,
QR At Rasie
A= 1836 mt
P=1256m
R= 1836/1256
Re1462m
45.69 = 18,36(1/0.015).46275"2

‘An earth canal carries water at a depth of 1.8 m. The canal is 6 m wide on
bottom andhas sides sloped 1.5 horizontal to 1 vertical. S = 0.0002. Using.

Tmo
dhe 1427460
4136+ 14mn= 1 +2075
1979+316n- 2 <0 mulpiybthsideby
1079n + 11412-1=0
114? +1079" -1=0
a = 10793 (1079—a@a14Iy—)
214)
+ Rese n= 0.0253
Aw TE19)-15.66m8 (©) Value ofm (Bazin)
Pete eee Cn tS
ue
R=A/P=1566/1249
R=1254m 4154=
1
= arb (1.258)7(0.0002)9
v= 0.657 m/s 4154+97:1m= 87
m= 1.205,
d to carry 1.5 m?/s at
‘An open channel is to be designe0.012.
™ channel material has an value of Find the monte
emicincular section.
solution
o=atms
For semicircular section of radius r
Am sor)mr/2 pay
RaA/P
15 Dar) « ahr@/274(0.007)"7 17=A08)me
A225
=0.2174
For MES, R= d/2 and x= 2y,
fe=29)
b+ 2(4d/3)= 2f54/3}
TUTE
Problem 8-37
Find the most efficient cross section for Problem 8-16 for rectangular se Am bis 1 (@yad/3.2
Solution See
For mostefficient rectangula section: ee
bo2d Las=1.63m
Ante
Reds Rad/2

Then, fom Live sin 5


15= @) abe (4/27406007)7" 08+ abe ower
a= 017006
d= 0515 mor51$ mm
2002)
A triangular channel with mostefficient proportion discharges water at the rate
‘A trapezoidal canal has the following flow and channel properties: 0 cf m/s, Assuming n = 0018 and S =0.021 calculate the normal depth of fow
{7 m/s; mean velocity of low = 80 cm/sec ; roughn ess coefficient =
slope =4H13V.Usingthemosteficient trapezoidal cana, whaslope
rested?
e proportio
Note:Thtrlangular n for most efficient tiangular sectionis that ofa
V-notch.
Top width = Sum ofsides
“Top width,x bd
Agree
a Sum ofsdes=d-+ a2
P= 2(42)= 2828
pare deal mood
se
red4 £ . nosed 2
Ri [Q=A0)
ooh 25 AQ)
Anne
q ase a9 (ony?
rea 1,
1127360 [Aandi]
= (v2 y+ 050
a=361m
b= 3.61V2
pezoidal canal has one side vertical and
trapezoidal : 45%. If
the other slanting best1m
is 25 m/sand its mean velocity is 1 m/s.
‘ through thea canal tcaaeeloele eruiod
discharge
Zs ‘Problems @= 21 (CE Board November 1595)
A wapezoidal flume of mosteffc proportion has a base widt
fall discharge is 3.m°/s. If the ient
as
h of 1.5 m. Its
‘ectangular section, by how muchsame material were used for a most efficient
would the discharge be decreased in m/s ?
Solution
‘ Q=AlRnsa
n
a= Asano
; Let K= (1/n)sv
Using the Manning's formula: Q= KAR”
v= LRwgn Using the mostefficient of all
ane trapezoidal section
5-2] ron {all sides equal)
‘Since n and v are constant (v= 1m/s), $ is minimum when R is m an 22805
hencethe section mustbea Most EfficientSection, then: as
A=2925 mt
Redj2
R=13/2
= K2925)(0.65)""
k=1367
For the mostefficient rectangular section:
aod
‘Using the same material means the same perimeter as the trapez
section then
Pab+2d=2d+2d=15(8) Q-a2 Ragin
d= 1125 m.
b= 2(1.125) Q= (6514) phy GA) ””2001)"
b= 2.25 me Q=2181m'/s (design capacity)
‘Ax b= (2.25)(1125) Using the mostefficient rectangularsection with Q = 21.81 m/s & §= 0.001
‘A= 253125 m?
Re d/2= 1125/2
R= 05625 m
Q= KAR® = 1.367(2.53125)(0.5625)"”?

Raa/2
Ane
= Cahs (4/27240.001)7 = 21.81
an=7416
4=2087m
Note: For the same material (perimeter),rge channel slope, and b=2d = 2(2.018) = 4.174m_
roughness coefficient, the ratio ofthe discha through the most
efficient trapezoidal sectio n (the half regula r hexago n) and Savings permeter length:
‘mostefficient rectan gular sectio n is: Excavation = Volumeojat- Volumenstent
Ecavaton == 030perlengthtal
(65\LANG)- (41742

EeTess erane) ietreeeae2eom7


m deep lined with smooth LUning= O98utprmeterlengthof exon
"A rectangular canal, 65 m wide and 1.4What savings in earth excav
‘0,013) has a hycrautic slope of 0,001. have been effected by using #
lining per meter length of canal couldn but
i ‘canal sectio adhering to the same di
5(2r-6)@x-0+ sin)? (cos0-1) +2(2x-0 + sin6)*8= 0
(@r-0+ sind)? [52s-9)cos 8-1) +2@x-8+sind)]= 0
5(2ncos0 -2x-@ cos 6 +8) +4x-28+2sin8 = 0
10x cos 8 - 10x-58.cos@ +50+4x-28+2sind = 0
10xcos -6-58 cos8 +38 + 2sin8 =0
Solve for by trial and error (not shown here)
0-576"
‘Then:
d= D/2+ (D/2) os (0/2)
d= D/2[1 + cos 675"/2)]
d= 0.938D

Ma.
Determine the maximum flow through a 1.2-m-dlameter concrete culvert which
Inlaidona slopeof 0.009. Use n = 0.013.
“Solution”
‘The maximum flow though the circular section occurs when the depth of
flow d is 0.938 ofthediameter D.

3
Q= Kivi? 160-0) + sinoplaay? So-eeel
= - KO 2x8)+sind)°/?
a

ac [@a=6)+sino}*/?
ts (2n-0)°*
AQ ¢ 2x OF§1Gx-0)+sind}?/*(-1+ cos)
® [25-97
Pelee omePAeeeBED a
[2x-9P?P
Solvefor Q: (duringuniform flow, d= tm)
Q=A. Rs

Rw 1.102/3.166 = 0.348 m 021)


(21) oop
1

2-421 Rinse Q=2611 mys


= 1.102.
a ahs (0348)270.009)17 aaa
At section 1;
= 3979mi Aisne
RimAy/Py
Ry =15/(12+201.25)=1.08m
Uniform flow occurs in a 12-mwide rectangular channel having a bed me Q/Ar
(00021 and n = 0019 atadepth of 1m. Ife partial closure ofthe gate near the y= 2611/15 1.781m/s
raises the level at thatby 050m, how far upstream isthe point where the a3
flowis 125:m? Fe 014m

Re
rain _ 6sgr9
(0.197074)?
103*7?
Atsection
Ar= 12015),
Ars 18m
Re Aa/Py
R= 18/(12+ 2015) =1.20
m= Q/As
M2 26.11/18 = 1.45 m/s
ot
0.107 m
2g
g.= 0019/2145)?
aa eae
= S25: _ auoriosoom
m= O/ AL
0) = 205/2.066 = 0.992 m/s
0.05 m
5,» Come
sje (0.018)%(0.992)?
(056)"
Atsection2:
r was constructed 0s (0/2) = 05/1
{A reinforced concrete drainage outfall 2 m in diamete
es to the sea as shown in = 120"
‘uniform slope of 2 per thousan d and discharg
figure. During the recent heavy downpou r,the peak flow in the drainage: B= 360°-0
was 205 m/s. Assumi ng steady flow and n = 0,015, determi ne the d B= 240" os ;
between manholes 1 and 2
Arn CY ycypsin 20"
v= 2527 m2
J o= 2050
pe MeNeAOT
180°
P=419m
Re=Ai/P
Ry=2.527/4.19 = 0.603 m
w= Q/Ar
y= 2.05/2527 = 0811 m/s
22!
28 2 0.034 0.084m

o (00157(08117
Tae "= 00002905
5
An cures + Sa)sin 151,04°
Ay= 2.066m? Bye ca vawonsumee
2 7
5 =0.00038175,
p= 2(1)(20896")
30°
P=3617m, Me HB
Ryn Ay/P na liars
Ry = 2066/3647 = 0.565 m Cas
Atsection 1:
Av=45015)
Ai= 675 m8
R=Aym
Ri=675/(45+215)=09m
me Q/Ay
At an
‘A rectangular concrete channel 45 m wide iscarrying water. 300away, = Q/6.75
pointthedepth of water is 15:m and at adownstreampoint = 01480
m. If the channel bed slope is 0.0010,find the appr
depth of flowis 1.17
flow rate. Usen= 0.013. 3 ooo
Solution

Ry 5.265/(45 + 261.17)= 077m


m= Q/Ar
= Q/5.265 0.199
22 =a08
eae:

$0 (0.013)?(0.19Q)?
CORES = ooo000864c

§ = Si+S2 _ +
0.000008260.00000864
2 a
5 = 0.000006459:
2 From
$a vy edie SL SL
= 2 Sna-mese
2g 2g 2g
2M 45 L=d-d+SL Eq)
0.001802- a.01129 + (0.000006450")(300) eee.
iaeeecres= 1.5 -1.17 0.001(300)
2 Q=1552mys
20
Uniform flow occurs ata depth of 240 m in a 3.60-mwide rectangular chan
The channel bed slope 0.008and n= 0015. Neglecting head los, what lg
‘of bump, extending over the entre channel widths required to create a citeal |
“The cteal depth for rectangle chaneli depth at the bump. Also determine
through the bump,
the behavior of the stream as it
passes
a feos
£ Solution
Len ve 1 RSI"
41-1627 o/spor eter vt oats 36x24 13 Vong
0015 36+2x24,
v= 2.88 m/s.
Q= Av™ (3.6x24)(2.88) = 24.9 m/s
Poe ae
Froude number, Py=0/ gd
-width. Determine the minimum specific energy.
Since the flow is at the upper stage, water flows down to the lower
Solution stage as it passes through the bump (seethefigure below)
“The minimum specific energy occurs at critical stage:
Solving for critical depth: Unit ow, q= Q/b= 24.9/3.60,
Unit flow, q = 6.92 m/s perm width
7 ast
aie ¢ ee Heap
d= 1273
ope stage
He ee
Gyrod) Lower
veq/de
= 45/1273 =353 m/s
2
foomere
Heng
= G537/(@981) +1273
Hag "191m owte wha aeathe wpesae
oO Hue @/2)de = @/2)(1.273) ie a (i
Hoa = 191m
ee & a
Critical depth, d= 1.696 m (©) Caitca depth
= g/d 692 11.656 49=Q/b= ad
= 408 m/s = 1.905(15)
4 28875 m/s perm width
[Neglecting losses, the EGIL is horizontal, then from thefigure above: ant - (CE
2 2 evi 981
Do gae sath ‘d= 0.94m (ertieal depth)
2g 2g
O87 "24" 24- ony
HOF 1596 (0 Critica slope
apemezon "1"
Spay ove Eons
= 4/d=28575/094
oR v= 304m/s
Re=A/P.
= 3(0:94)/[3 + 2(0.94))
R= 0578 m
1 (0578p:
‘0018
‘S.= 0.00324 (critical slope)

sablem B= 3T ‘Determine the critical slope for a rectangular smooth concrete fume (n = 0,013)
3-m-wide rectangularchannel having
‘Water is flowingatadepth of 15 m inastageof 45 m wide whichis to carry 45 m’/s per meter width ofchannel.
= 0.013 and S = 0.0009, (@) What isthe flow?(b) Determine the crt
depth, and (¢) determine the critical slope. Solution
ove san
1) Sagetow
Solution
7
v= Lanse den YES here 9 = 45 m/s per meter width

oe 1
0.013
[oa n y”
3+ 2(1.5),
erty es Rem AP
Froude No, Fem 5 = ear = 45(1.279)/145 +2127)
Re=0813m_
Froude No, Fy=0.497<10 (subcritical)
mg/d
2.8 45/1273 =3535 m/s
Problem 8-34
3.535 = hy (081379527 Water flows at the rate of 25 m/s in a trapezoidal channel having a base
S.= 0.00278 ‘of 3 m and side slope of AF13V. If the canal is. lined with concrete (n = 0.0
determine the (0critical depth, (6critical velocity,and (the critical slope,
o-35
‘A parabolic canal has a top width of 2 m and depth of 2 m,
critical depth and critical velocity when the flow is 3 m*/s.
Solution
oo AS }—_——1'»
$B ein
Koi
Q=3m'/s el : (0) Ceiteal depth
; ok Paw
| @.
7 laroe
B eP 34 ae
Q=25in'/s
ches pn O48 (343
ZB
fo ae : E Am dct e/3
oa 2
By squared property of parabola: Solve for dby trial and error:
ey
(B/2e d, = 1.523 m (critical depth)

(@)Criticalvelocity, v.
v= Q/Ac
‘Ac= 3(1523)+ 4(1.523)/3 = 7.662 me
19. = 25/762
1%, 3.263 ny (critical velocity)
as=1548
h = 1.115 m (critical depth) (9 Gaitcal slope, §,
tem ©1 Rang
Critical velocity2:
Q=Av.=3Biv. Rea 4/2 = ae 7.662.
R= 20.115) 1° SB SBA
B= 1893m R=0919m
3= 3499)0.115)% 3.263 = (1/0.013)(0.949)” 5,72
2.2703 mys (critical velocity) 'S,=0,00193 (critical slope)
(8-35 (CE May 2002) q Flow stage:
flowsathe rate of 350 cfs in a trapezoidal canal having base width of
Attia
eae =
BQ?

‘B= 2(1.155)=231 m
A= (L1557=1384m?
BQ? __ 2314)?
;
3A" 98101334)"
30)1507>10 tert flowtte vertag
Atcritical stage, ¢. 8
= 380
Blnval’
Am 12d + YH) x2 124+ 28
grs22tys
0 area
92.2 12+4d sees.
By trial and errorusing the choices, d= 2564 feet Note: ‘The normal depth (1.125 m)isess than thecritical depth. This shows
{hatat normal condition, thechannel isflowingat lowerstage,
sabidas
‘A tiangular lume having sideslope of 1:1 cases water athe rte ofm/s “7
bottom of the flume is on a slope of 0.005 with n = 0.013. Determine the stage 0 Determine the flow in trapezoidal channel having base width of 4 m and side
flow Whats thecritcaldepthforthegivenlo? slope of 1:1 ifthecritical depth is 23m.
Solution
Solve for thenormaldepth:
Qeaveat rns
An iQdyd= B
fence wut ors
ReA/P=a/[2as2}
R=03544
Q=# (/o.o13y0354d7(0.005)12— 4
a= 147
d=1155m normal depth
eesA= V(8.6+4)23) = 14.49 me Bycontinuity condition
Aso Awe
: cunt. 409}03)
meow bi)»
Q= 5891 mys Inq, (1):
OW (you O27/da)* , 4,
a 2esn 7°" Gea *#*03
ees mavngwitha vl o£03 m/s one a depth09m ina rect 007206060
5
channel, It approaches a smooth rise in the channel bed of 03 m. What.
the etimated depth be afer therie? 49-060s6i2 + 0005720
‘Solution Solve for d by trial and error:
Determinethestage of ow: 4=059tm
Bvn0/ fet
=03/ 5103) 1982)
Fy 0101 <10 (Upper stage) Water is flowing ina 3m-wide rectangular channel ata depth of 360 mim_A
‘Therefore the water moves down as it passes the smooth rise 1300 mm, Determine thedischarge, eh
Solution
Tipata ee
Es Aidly +)
£ 2
2 0360139036413)
# 2
4q=1952m"/s per meter width
O-e
=19520)
= 5856 mye
Co
Water is moving in a S-m-wide rectangular channel at a depth of 1.0 m and a
Me oye Be eavon 54.0) Froudenu mber Fx=J.
of flow and (b) Froude
thewaterundergojumpwhatsthe) do
number after the jump? ee
BP Te
=
VO Ea
2) 99mn/s (velocity before thejump),
auh,
4=99{1) = 99 m/s per meter width
Theny
@ diddy +42)
2
(09)? _ Leds(1+ ds)
‘981 2

= 2475 m/s

ee aa

hydraulic jump occurs in a trapezoidal channel with side slope of 1:1 and
‘width of 4 m. If the upstream depth is 1.0 m and the downstream depth i
'm,compute the(¢) discharge andthe() power lst inthejump,

ttheeanoat
tih E (Re (ee
section;
Ay= 46 +4)(1) =5 me?
Aah= 4C)(05) + HQ)YE/2)x2 ‘A hydraulic jump occurs in a diamond-shaped closed conduit as shown in the
Ay=233 figure. the conduit is horizontal andthe water depth just upstream of the jump
‘Atsection 2:
4s 600 mm. the conduit is completely filed with water downstream ofthe jump,
‘As= 8+ 4)(2)= 12me Pressure gage readings are as shown, Compute the flow rate.
‘Agla=2Ah

Ask.= 10578
In Eq. (1):
os7s-20~
8
Q=2647 mys
Power lost:
From the figure:
era
2g 28
ne = BBM
B= BE 5200/5
a529m/
22
a 125m
1.428:
2g meine
P = 2.
A Ba. 2.206 m/s one
?
220248 m
2g
1428+ 1=0248 +2+ HE

Power lost = 4674 kilowatts


An uQNQ=2
Agha = x4(5/3)
Aalg =2/3
Ah; =054(02) =0108 aye (1 (1
/3-0009= G8! 1
eae 981 le 3)
“Atsection 2: 2-009)
Ar=2h:= 1.08 m? 3-027 = 00619| <=002
Fighn= 0714 +06=1314m a }
28+ 0.02712 = 0.68822 -0.09)
Aah, = 1.08(1.314) = 1.419 38-0715: + 0.06192 =0
‘Solve x by trial and error:
x= 0.858 m.
PowerLoss:
SE: aie Bowen
2
‘A hydraulic jump occurs in a triangular flume with side sloped 1:1. The
ate i 0.45 m’/s and the depth of water before the jumpis 0.3m.Find the di
‘after the jump and the power loss in the jump
Solution
fimo
=06113 m/s
ons
ey santo m

Pare oe 2-2) 1274+03 «0019+ 0858+ HL


ae & & Aa,
HL=0697 m
Q=0.45m/s. P= QyHL = 0.45(0.81)(0.697)
A= 1409)03) P =3.077 KW (power loss)
Ay = 0.09 m?
‘Aji = 0.00.1)
Ah = 0.009
Forthe jump:
determine; (a) the depth of flow at the foot of the spillway and after t Baily
“Jump, (6) the velocity of flow after the jump, (c)the energy after the jump, and f 2
{i the head loss inthe jump. 2.53)? _ 014234,(0.1423+4,)
oseen?
‘Solution 42 + 01424-1785 =0
ds 454m (depth after thejump)
© erm9/de=3.53/4.154
= 085 ays (velocity after thejump)
oe
mm a

Hy= 4.19 m (energy after the jump)


‘The discharge per meter width ofcrest is: @ HL=H-th= 315-419
4g" ColEB HL=27.31m (head lost)
| gn L92(1.5)2
= 3:59:m'/s per meter width
(@) Neglecting head 1osson the spillway: ‘A trapezoidal channel with a bottom width of 3.m and side slopes of 2
horizontal to 1 vertical has a horizontal curve with a radius 30 m without
15430= 50 +ai ‘superelevation. Ifthe discharge is 22 m?/s and the water surfaceofatthe inside of
the curve is 15 m above the channel bottom, find the water surface elevati on at
97 Vad = 958 the outside of the curve. Assume the flow id subcritical.
9353/4
- O58/ay 4,
15°88)
0635 54,
a15= 98
a
SelvinMhSO1zS
ordy aad
(depthatrorthefotofthepinay
vB Problem 8 - 47
yr dedi stream has a speed of3.6m/s and is 60.cm dep. Find theangular spr
Am Wet 15)Q4+ 8) the wave ct up by atamalar obstruction?
r=30m Solution
v=Q/A
v= 22/fife +15) +8)) any
vm Ad/(x+15)(2x +8)
“4
erent
o15= LEPTSNB)
981030)
x15 = Lore
(@+ 157Qx+8)
6578 (x-15)r+ 15}12x +8)
By trial anderror:

‘A small boat moving in shallow still water where thedepth is15 m;


‘makesan angleof&* withthelineofmotion. Compute thespeed ofthe boa:
Problem 48
‘What flow can be expected in a 1.2-m wide rectangular, cement-ined channel (1
= 0.015) laid on a slope of 4 m in 10 km, ifthe water flows 0.6 m deep? Use
Kutter’ C.
‘Ans: 0425 m/s
Problem 8 - 49
AA circular sewer pipe 1.6 m in diameter is laid on slope of 2m per kilometer.
‘The pipe is made of concrete with n = 0,013.Determine the discharge when the
pipe is two-thirds full.
‘Ans: 2.73 m?/s
A rectangular channel has a width of 3m, If the discharg
{is 2.1.m/s and coefficientofroughness n = 0.015,compute etheflowing in the canal
critical slope.
‘Ans: 0.00414:
Problem 8-56 /
A trapezoidal channel With side slope of 1:1 has a bottom width of 2m. Ata
critical depthis 2.24 m, what the velocity of flow?
Ans:3.79 m/s.
Problem 8- 57
For a wide open rectangular channel, ifthe critical velocity is 4.5 m/s, what is
the critical depth?
‘Ans: 206m

Hale I A
‘Ans: 1627 m/s per meter
Problem 8-59
Water flows in a 3-meter wide rectangular channel. Ifthe critical depth was
A triangular channel with most efficient propo ‘measured to be 0542 m,, what is the maximum flow in cu. m/sec?
‘of 2m?/s, Assuming n= 0.018 and $= 0.0021 calculate the normal depth off a Ans: 375°
in meters.
Problem 8- 60
What are the best dimensions for a rectangular brick channel (n = 0015)
Problem 8; 53
designed tocarry 5 m/s of waterin uniform flowwith S,= 0.001?
rectangular flume 3m wide and 1.5 m deep has a carrying capacity of 45. Ans: 1.27 mx 254m
m/s. If the same material were used for the mostefficent trapezoidal
‘by how much would thecarryingcapacity increase in cu, m/s? Ans: 1.22: trebenine at
‘Waterflow in a wide channel approaches a 10-cm-high smooth bump at 15 m/s
and a depth of 1 m. Estimate (a) the water depth over the bump and (b) the
Problem 8- 54 ‘bump height which will cause the crest flow to be critical.
If water flows uniformly at the rate of 45 cubic feet per second in a ‘Ans: () 85.9 cm; (6) 197 mm
Aon wid wth = 4 feetat depo 2 fe what type of ow 81
ns:
‘undergoes a jump, compute (a) the depth after the jump,(2) the velocity:
after the jump, (€) the Froude numberafter thejump, (d) the head lost, and ()
the percentagedissipation. ‘
‘Ans: (a) 3.46 a (6) 289 m/)
(©) 0496; (0625me; () 14
Hydrodynamics deals with the study of the motion of a fuid
Problem 8 - 63
A rectangular channel has b =3m and d= 1m. Ifn and S,are the same, whati Interactions ofthe fluid with its boundaries. This chapter will discussand of the
the forces
developed by moving fluids as isis important
the diameter ofa semicircular channelthat will have the same discharge?
Ans: 2.67 m such objects as pumps, turbine, apap, rocketsin the analysis and design0
and many other hydraule
Problem 8 - 64 ‘The basic principles used in hydrodynamics are the Newton's laws of mation |
‘Acday tle V-shaped channel (n = 0.014) has an included angle of 70° and ‘which canbestated as follows! Bae |
|. A body at restwill remain tobe at rest orin motion wil remain in
‘with auniform velocity along a straight line until acted pon byan external ||
85 m°/s. Compute the (a) critical depth, (b) the critical velocity, and (c)
critical slope for uniform flow.
fa
IL A particle acted upon by an unbalance force system has an acelertion in
line with and directly proportionaltothe resultant ofthe force system and
inversely proportional tts tase (Lawoffer) A
UL Ineveryactiontheres always an equaland oppositerection. (Latof
slope of 24 mm/m. Find the depth and rate offlow for critical conditions.
‘Ans: depth = 514 mm; q= 1.15 my FORCE AGAINSTFIXED FLAT PLATES
Consider the jet of fluid shown to strike a flat plate held normal tothe pathof
Problem 8-66 the jet. Before contact, a mass of fluid M; moves with an intial velocity of2)
‘A trapezoidal canal with side slopes 2:1 has a bottom width of4 m and cart and asistrikes the plate its velocity in thex direction ism»whichis less
flow of2 m/s, Calculate the critical depth ‘This change in velocity is caused by the force exerted by the plate on the th.
stated by Newton'sfist law ofmotion,
For instantaneous closure, the pressure increase reaches up to the pipe entrance
Consider the pipe line shown leading from a reservoirA to thevalve at B. If 0 at A where it dropsinstantly to the value it would have for zero flow.
valve is suddenly closed, the lamina of liquid next tothe valve will
‘compressed by the rest of the column of liquid flowingagainst it. At the sa
time the walls of the pipe surrounding this lamina will be stretched by Rapid Closure (t.< 2Ue)
pressure ‘The next upstream lamina will then be broug! I is physically impossible for a valve to be closed instantaneously (= 0). Fora
rest, and s0 on. Thecessationof flow and the resultingpressure increase rapid closure (t'< 2L/c) the maximum pressure near the valve would
‘along the pipe as a wave with the velocity c which is given by the follo stil be
‘equations:
For rigid pipes:
'No matter how rapid the valve closure may be, so long as isnot the idealized.
instantaneous case, there will be some distance x from theitintake within which
the pressurerisecannot extend all the way to the reservoir intake.
Problem9-1
‘A hose and nozzle discharges a horizontal water jetagainst a nearby vertical
plate. The flow rate ofwater is 0.03 m/s,and the diameter ofthenozzle tip is
30mm. Findthe horizontal force necessarytohold plateinplace.
where: cls of prem wavetnmy/s
ofarly MidiPa
ofhe«10Pa)
Ersbulkmodula
(Porwatera30°C, E4225
motuflay
=i= ppe ofthe pipe wal in Fa
ds tnteraltikes
ameterin ofpipein mn
1Sanghof
da velo ofpipeflownm
tem time of closure in seconds

inm/s
Water flows from a large tank through an orifice of75 mm diameterand against
‘8 Block, as shown in the figure. ‘The water jet strikes the block at the vena
contracta. The block weighs 220 N, and the coefficient offriction between the
block and the floor is 0.5. The orfice’s coefficient of discharge is 0.60, andits
‘coefficient ofcontraction is 0.62. Whatis the minimum height to which water
‘mustrisein thetankin order tostart the block moving tothe right?
Problem 9-4
Flor motion to impend to the right, F=f= N= 0.55(220) = 121N Ajet ofwater wasissued horizontally ina 50-mm-diameter orifice, with €=0
Fay oy. ‘nd C. =0.9, under ahead ofSm. What isthe force available in thejet?
8 Solution
10 Q/a, whereas the area ofthejet atthevena contracta
CxA F
0.62% § (0.075)
0.00274 m? osGah
Q/o.o027a 2 = 090% JRE) = 8.91 m/s
v= 365Q
fei} 659)
p= 2 = CARR
g = 0.60% § (0.05)%« /2(881)(6)
Q= 00117 m/s
+= HONTESDgs
F=104.247N
.0182 = 0.60% § (0.0757(23h
T=2am Ss
y=h+o3 small ingot and platform rest on a steady water jet as shown. If the total
y=24+03=270m weight supportedis950N, whatis theetvelocity inm/s?

Problem 5
[A jet of water flowing, at the rate of 012 m/s hits a vertical building
horizontally at close range with a force of 7000 N. Determine the diameter o
‘theetin mm,
Solution
F=pQu
=9Qx2
FopQxe
F=pQya

D=0051 m=51 mm
Problem 9-8 d
f Issuing vertically from a nozzle having a diameter of25 mm,flows ou If jet of any fluid of an area A and with a velocity o is deflected through
‘ahead of30 m. If C= C,~ 098, determine at what constant height
et of water support a load of 200 N verticallyfrom the nozle ip? inglewithoutanyangemapitadethe vy, proveat he
force acting on the vane is F = 2pAv*sin (8/2) 7% in
Solution ‘Solution
oie? 2gy >a.) Qu Av . Is
‘11 Co/2gh = 0.98.J2s(20) Fam pQ(0v- Ua)
m=238m/s Fe™ pAv(o-v cos)
F=pQu=200 i= pAv'(1~ cos 8)
Q-CAPe Fy = pQ(y- Pm)
Q= 098% $ (0.025) /2520) B= pAo(0-vsind)
Q=00117m/s Fy= pAvtsin@
200= 1000(0.9117)02 Fe AFAR
naim/s
In Fa. (0) F= cos0))?
(17.1= @38)?-20-8Dy
FapAvt fi-zend-ecos'0+an78
obiem = Butsin’9 + cos=1
‘A small diameter jet of water with a velocity of 15.8 m/s strikes a large, fx F=pAvt J2=2e080
flat plate which is normal to the axis of the jet. The force of the water on t
plate is 280 N. What force is exerted on the plate if the jet velocity is doub From therelationship:
sin @/2)= fest
[FpQu] 280= 1000)(Q)058) Y2=2c086 = 2sin 0/2)
Q=001772m'/s Fm pAv? x 2sin (0/2)
‘When thevelocity is doubled, thedischarge i also doubled, F = 2pAv* sin (0/2)
= 158Q)
p=36m/s
Q= 0017720) =
Qo.0a544m/s ‘A jet having a diameter of50 mmand a velocity of 12 m/sis deflected through
an angle of 60° by a fixed curved vane. Determine the horizontal and vertical
F= (1000)0.03544)91.6) ‘componentsoftheforceexerted bytheetonthe vane. Neglectfriction,
Fa1320N
(0.059012)
(= 00236 m/s

atin
FepQ(ou-vm)
= 1000(0.0236)(12-12 cos60°)
F,=1QNtotheright
F,=pQ(eu-es)
Fy= pQy- Pm)
= 1000(0.0236)(0- 12 sin 60°) = 1000(0.05)15cos30* -15 cos60°)
Fy 27452 to the right
Fy = 245 or 245 N downwards
Fy== 1000(0.05)(-15
9Q6-2m) sin30*- 15
sin60°)

iaetualud tatal enyaaeiieomteltykenapeal


Solution horizontal jt of water flowing freely in the atmosphere is deflected through
Quam
= $0.05)102) eR® fan angle of 9°by a fixed curved vane, Ifthewaterthas a diameterof 40 mm
and a velocity of8 m/s, whatis the force required toholdthe vaneinplace?
= 00236m/s &

Fr=pQ (0-7)
= 1000(0.0236)[12- (12)]
F,= 5664 tothe right
Q=0.010 m/s
Fr=pQ(eie-Pa)
= 1000.01) -0)
F,= 80N to the right
down from the horizontal. The jt is deflected upwards 60° with the
‘siteaves thevane. Determine the X and ¥componentsofthe forceexerted by
thejetonthevane.
ps fiPaR? R= 12741 (force on the spring)
= (oor i ‘By Hooke's law:
5
Fe11314N Remax
12741 = 15008
Rx F=11344N acting at 45° with the horizontal 4 ‘Ax = 0.085 m or 85 mm (compression of the spring)
Using the formula derived in Problem 9 - 8 Ql, - 0m)
F=2pAv?sin (@/2) 1000(0.029}(0-15 sin)
= 2(1000) § (0.04)? (8) sin (90°/2) i Eps One.N,
Pe1374N 3 Ry 907.6N (Forceon the wheels)

Forthe cart in the figure shown, compute the force on the wheels caused
denectingthe etand thecompression ofthespring,ifthespringsstifiness i 15
by ‘The waterjet in the figure, moving
woot feea” GisToethenents
splitter so that
oe
at 12: is divided

; direction ofthe
ae 4 flowinahorizontalresultant
plane
force on this single stationary blade. Assume ideal
5] Denson t

‘Solution
‘The vertical component of the force exerted by the jet is supplied by t
{round actingonthe wheel, whl the horizontalcomponent is supplied by todas
Distcirri Fe= pQu(eu =n)* pQdou 00)
onan Fy= pQs(0y =) * Qe)
= ¥ 005705) QAv
Q= 0.029m/s 3 = $ (0.15102)
Q=0212 m/s
Fria Qr= $o=00707 mis
Problem 9-15
‘The water tank in theigureshown stands oma frictionlesscat and feeds ajet 0
diameter 50 mm and velocity 12 m/s, which is deflected 50° by a vane,
‘Compute the tensionin thesupportingcable,

F, = 1000(0.0707)12-(12 cos60%] + 1000(0212)[12- 12.0s 60°]


F,= 2548.6 tothe right
‘Scot, THE TRTGTON WW Whe cable Te equal to the Ke
Fy 1468.43 N o 1468.43 N downwards componentofthe force exerted by the jt on the vane.
Te: PQ(l%- Mix) force reaction
Fs \BP+RF Q=A0
= \iasHi6y? + (468.57 = F(005702)
F= 29379. Q=0.0236m/s
tan= Fy/Fs
Since the jet source and the vaneis on the same body,then there isno initial
tang = 1468.43/2518.5 ‘momentum. The initial absolute velocity v1 = 0
= 30° T= 1000(0.0236)(12 cos 50° -0)
‘The force onthe splitter is 2937.9 N acting 30° down to the right T=182N
Fre pQ(Ou- 0)
Q=Au=Alo-v)
tyne tuc080 =a + (0-1)c0s0
‘Solution t= e080 +0 vc088
P= 0.6080 + v1 cos 0)
Fy pQu- Pa)
OFA tem frceré-ettecor
= $ (0.075712)
F,= pA(o-e/)|v-0.c0s0-¥/(1 cos 0)]
F,= pA(o-v/[ ol ~cos0)-/(1~cos )]
Q™ 0.053 m/s Fi. pA(o- 0) cosYo-v)
‘Since the jet upon leaving the vane stays in the tank, then there is no final Fr=pA (L=c0s0) (0-0? or pA (.=cost)
‘momentum and the absolute velocity v= 0.
Fy PQ(Pry- Pay)
F, = 1000(0.053)(12-0) Fy = pAu(0- usin @)
Fr = 636 N (force exerted by the jet) Fy=-pAut sin 0=-pa(e—v)sin
aes
‘A jet having a diameterof50 mm and a velocity of15 m/s is deflected by a vane
mo-a7 ‘which is curved through an angle of60° and which is moving with a velocity of
jetofanySuidofanareaA andwithavelocityvstrikesasinglebody moving {m/s in thesamedirection asthejet. Determine the Xand Ycomponents of the
thesame direction and with a velocity of v/, flows over it without friction loss, {forceexerted by thejet and itsdirection and velocity asitleavesthevane,
leaves withareatvevelocity inthedirectionof,provethat
Fr=pA(l-cos0)o-/andF,= pA@-v}'sin®
Using theformula derived in Problem 9- 17:
F.= pA (1=cos0)(0-/
= 1000 §(205) ~cos 60")15 - 6)
F,=7952Nto theright
Fy=-pA(o=vsind
‘= -(1000) $ (0.08)4(15 = 6) sin 60°
F, = 137.73.Ndownward
Tae OF9 ;
a...
u=15-6=9m/s i A horizontal nozzle discharges 100 L/s witha velocity of2& m/s. Thejt strikes
oe an unsymineticaltwocuped vane tats moving in he sre recto athe
jet ata velocity 6 m/s,Half of the jet is deflected upwards 90° and the other
Q= $0057)= 001767 m/s half is deflected downwards 45%, Determine the X and Y components of the
oeeean forceexerted by theet onthe vane.
mal) ‘Solution
0
‘Dy, =6+ 90s 60" =1051
tyusinoo"
Dy=9sin60° =7794 m/s
T= PQ(ou-va))
F, = 1000(001767(15-105)
‘F, = 7952N to the right
LF) = pQ(iy- Pm)
y= 1000(0.01767)(0-7.794)
F, "137.72 N downwards
= Joa? +02,
ere PO(n = va) + PO(14-0)
= (05079F j Fy pQi(0y- v9) + HOS)
te @=Au~ Qu/e weve monster
u=24-6=18m/s
= 0.1(18)/24 = 0.075 m/s
Qs = Qs = Q/2 = 0.0375 m/s
"= 1000(0.0375)(24 - 6) + 1000(0,0375)(24-18.73) Work = pQu(t ~cos Bo"
= 872.625 tothe right 1000(0.0664)(15)(1 -cos150*)(20)
Work = 3771.23 Nena
, = 1000(0.0375)0- 18) + 1000(0.0975)(0- (12.73)]
F, = 1971625 N oF197.625 N downwards
Board)
300 mm x 200 mm reducingbend in a level pipeline carries water atthe
Fe Gt rate of 280 L/s. Determine the components ofthe force necessary to hold the
A jet issued at the rate of 195 L/s and a velocity of 35 m/s strikes a blade ‘bendin position.. The pressure at the entrance is 345 kPa and the friction inthe.
‘moving in the same direction at 20 m/s, ‘The deflection angle of the blade is bend is negligible
150°, Neglecting friction in the blade,determine the X and Ycomponents of the
forceexerted by theet on the blade. Find also the workdone on the vane, Solution
Solution

aan

wen
u=35-20= 15 m/s,
t= -1.c0830"
v= 20- 15c05 30° =7 m/s
sin30"
bay 115sind0"=75
y= m/s
Q= Au
= Qu/o= 0:155(15/35) Solve frpsbywaiting the energy equation between1 and 2
= 0.0664 m/s A=B y
Fy= pO(eer) = 1000(0.0664)95-7) a Bag
F,=1860 N tothe right 2g oe Aaea og
Fy= PQ(0-2) = 1000(0.0664)(0-7.5)
05+ ~
2S gosZe.am
F, = 498 N or 498 N downwards pe=31313 kPa
Fi=pAn345x $03)
F-24291
Fix pels=313.19% $027
Fe 9at kPa
2439-984costs?-R= O7ACSDa91 conts?-396) axaeate
onsen 3,
R,= 1678 KN to the left f
compose “The forces acting on the gasket are; the compressive force due to 6 bolts
25,Deyo) tightenedat890Neach, andthetensile force yaffected by theuid flow.

‘The 180° pipe return shown discharges salt


Water, with a density of 10,104 N/neinto the ‘0.085, TB m/s
2 Sage
atmosphere at a constant rate of 85 L/s, The
pressure in the water at section A is 48 kPa,
‘The flow area of the pipe at A is 15400 mm? 3F,= & (oy-09
tnd that of the discharge end is 7600 mm If
teach of the 6 flange-bolts is tightened to a B-Re Mey-oy)
tension of 890 N, determine the average
‘pressure on the gasket between the two
flanges. The flange area in contact with the
79-ny= SEOcap-55
‘gasketis 6700 mm. y= 201 Ndownwards
InEq.(2):
Fy8906)-2201
=309N
‘A 50-mm-diameter nozzle is attached to a 300-mm-diameter pipe by flange
bolts. What willbe the total stress in the botswhen the pressure athebases
S17kPa, Assume C=090.
‘Solution

1%, = 0.0278(28.959) = 0.805 m/s


Q= Aro = §(0:3)%0.805)
= 0.0569 m/s
Be = Heo)
‘The bolts in the flange will resistthe horizontal trust R, due to water flow
therefore the total stress in boltsis equal to Ri. 4 A-Re= 2X, -o)
¢
5171403/1-R, = 225708)axo590805)
Re = 34.94 kN

‘The total tension in the bots b 4.94KN


Problem 9 - 25
diameter is connected to a600-mun-diameter water main by a ‘Water under a pressure of 450 KPa absolute, flows with a velocity of 3
Fil "The velocityathe the pressure is 420
8 m/s and onthe reducer.
200 ma pipe bingxerted through a rightangle bend having a uniform diameter of 300 mm. ‘The bno
| Nepoctng lose, fin the resultant forcee by water lies in horizontal plane; water entersfrom west and leaves towards the
‘Assuming no drop in pressure, what is the magnitude and direction of
tt, resultantforce acting upon the bend?

corm Solution

ns eeetcee
Neglecting head lost, py = px
impr450 kPa abs
‘ae 5s 450-1013
Tee
x
pastor pom pr= 87 KPa gage
Fe Fe 387« $037
Qs Asma (n/4)0378) Fis P= 2465kN
Q=0.353 m*/s
Q- 70378)
t=OyDiPo: Q=0212 m/s
= 00/60076) = 1.25m/s
Bia caer ee P= Shel
fe
ot aA B-Re= Benn)
Bear
te S2RE8) o-3)
op 38 22g, ao
Bg’? , m 9at Ry=252864N
pre as72kPa
Fre piA= 31.72% $0.6 25 Leo
Fy=1221kN
Fim pohe= 420 $03)? By-fem oy-my)
Fn 2971N,
= Lees-ol
AeR-B= Senn) (R= R7R71
-0353(9.81) R= (25.286)? +(25286)
raan-r-297= SEED 61.25) R=3576KN
R=SLLEN
2 : -2
tan = = 23286/25286=1
0 mase
2
§ Sean
77 5(0.05)" Y -
‘The resultantforce actingupon the bend is 35.76 kN actingin the direction S nmg-B BR 5e.q
45° E
‘Sum-up pressure head from 1 to2n meters ofwater:
pee Pg.
Ba -2(F) egy
Find the pull onthebolts in the igureshown. Assume ideallw. “
Pa P< 04mof water
tay,
In Eq. (1):
12,994 Q*= 0.4
= 0.00568 m’/s
y= os a
"your
OE “2
> sare
Solve for pr:
Energy equation between 1 and 3 neglectinghead lost:
He Hagel.
iy
4 er
sey | ps. C157
2981) ~ 2981)
pHa
981
a

Solve for Q:
“Energy equation between 1 and 2 neglecting head lost:
Ey=B:
ME PU fe Pee
ag as ae
e275 0.0571 R= SMSREE599.1157)
R= 01726 KN
R=126N [Mom 0] F(t) = Fu?)
Fo (6031)(2) = 120.62 Ibs
Thetotal pull on the bolts is therefore 172.6
pag
lem9-27 (CE 2007) Flow occurs over a spillway ofconstant cross-section. The depth of flow atthe
“The jets from a garden sprinkler are T-inch in diameter and arenormal tothe 2= back of thespillway is 1.20 m and after the spillway is 0.20 m. Determine the
feetradius as shown in the Figure. The pressure at the baseofthe nozzle is 60, horizontal force on the spillway per meter of spillway width, Assume ideal
si. Neglect the velocity head at the base of the nozzle, Use coefficient of flow.
‘velocity Cy = 0.80 and coefficient of contraction C, = 1.00, 3
(@ Determine the velocity of flow ofthejt in ft/se.
(@) Determine the force exerted bytheetonthesprinkler,
(© Determine the force F applied on each sprinkler pipe 1 foot from the

Energy equationbetween
‘Oand A neglecting losses: t
[Eo EAL ‘
B02 5 PO +29 Assumingidea low (neglecting head lost
eo Hie Ha
Pay aay ae yo
oe
Bae
dy 2F ty
ee
Ge ey ton ce $00 Lo/0.2x0F ,45. (Q/02x1F 5.
28 25
ealVl tee
Prey,9278 54een) ; am/s
208585
on 8S 074875 m/s
ea
= (nC Aaca
Qm (1x08) $(4)? C448) = 0412 17/5 me
a = 4.4925 m/s
yh, Ar=9810(i) (1.21)
068.2 N an 2.0226
Boy Tholsas
Fy= yligA=9810(98)(021) mo 2.0226Fasay 787 RV
Fy=1962N
PFu=pQe- on}
PERpaleo)
“R-R-Fa=pQ(n-0)
7,063.2 ~ R~196.2 »
Fi-R-Fr™ pQ(r-)
9810(@)@x1)-R-9810(28)(0.35 «1=1000(20229)6,779-1.0119)
1000(0.8985)(4.4925 - 0.74875)
R= 350824 Npermeter R=9376N permeter

Problem 9-30
submari ne is running submerged in sea water (sp. = 1.03) ata speed of24
Water flows
pathrough a sluice gate having an opening, of 0.60 m above the kph The projected area normal to the direction ofgr.motion being 6 square,
channel bed. The upstreamdepth being 2 m and the downstream depth being, meters, Ifthe drag coefficient is 0.10, determine thedrag force.
(0.35 m. Determine the horizontal thrust of water on each meter with of the
sluice gate.

28
v= 24 kph + 366.667 m/s
6.667?
Fo= 0100981 «103)6) 7
Fo= 13.735kPa

ae
« torpedois launched with a sufficient power so that after it has reached
steady state ofmotion its velocity in salt wateri 16 m/s. (a) What speed woulda
it attain in fresh water if resistance is the same, and (2) what speed wouldthe
Assuming idealflow (neglectinghead los) torpedo attain infresh water ifthepower isthe same?
HH
dae ey ‘Solution
2g 2g (@) The resistance on the torpedo is the drag force
lavas 2. iapassxf eas A
Fo= Cova
O28
Q=20226 m/s
Since the resistance isthe same:
Fin fresh water = Foin alt water
Fo=pe Conte
corate
oe
981 vf = (9.81 x 1.0316 35.0 = 0.209610 109)1(00571
anaes 07 658.nys
(@ Power= Fox
Power infresh water = Power insalt water
‘copa iof eagee2 If the velocity of flowina 600-mmdiameter cast iron pipe (E= 8.29 x10" Pa) be
Aggie ciuagy changed in 0.25second from 06 m/s to 0 m/s by closing a valve 300 m froma
wep ene? reservoir, what probable increase in pressure due to. water hammer will be
981 op = 0.81 x 10319 ‘expected close the valve? The pipe wall is 20 mm thick.
y= 1816 ays
eo = 1 where Ex = 207x 10°Pa (for water)

‘Time for the pressure wave to travel round trip:


Tine, P= 2
Time, = 2 055sec
= (98107.85)$ x(0.05) :
W=4032N : seine Since the time ofclosure (, = 0.25 sec) is less than T; i isa sudden closure,
and the pressureincrease i:
BE=YwV pepcav
= (9810 x 1.03)$ w(0.05? “ ‘Po = 1000(1088)(0.6 0)
BE=529N r= 652,800 Pa = 652.8 kPa
Fp +5.29- 4032
Fo=3503N
9-34
the adequacy of a 600-mm-diameter pipe of length 2.5 km with a In the figure shown, the elasticity and dimensions of the pipe are such that the
‘of 20 mm if itis capable of carryinga maximum stress of 124 MPa. celerity ofthepressure wave is 950 m/s. The flow is initially 1 m/s. Find (a)
Pipe is to carry a flow of 0.85 m/s under a static head of200 m ofwater, the water-hammer pressure for instantaneous valveclosure, (b) the approximate
fan. an additional water hammer pressure caused by a quick-closing-valve water-hannaner pressure atthe valve if itis closed in 4 seconds, () the water-
Tocatedatitsend. Use Es =2.07x 10”Pa for waterand E =200,000 MPa for steel, ‘hammer pressure at the valve if it is manipulated so that the flow rate drops
almost instantly from 1 m/s to 04 m/s, and (f) the maximum waterchammer
Pressure at @point in the pipe 100 m from the reservoir if the flow rate is
reducedfrom05m/s to0 m/s in second.

P.=3,771,000 Pa =3,771 kPa


Static pressure:
pe th=981300) ae it =a
p= 2919 kPa
‘Total pressure, p=371 +293 ‘Velocity of flow, v = 0.884 m/s
Total pressure, p= 6714 kPa Celerty, c= 950m/s
(@)Instantaneous closure:
m=peo
= 1000(950)(04884)
‘Py = 839,800 Pa = 839.8 kPa

$= 10071 MPa < 124 MPa (Time ofclosure, t=4 seconds


‘Time forpressure wave to travel round trip:
Therefore, thepipe is adequate tocarry thepressure ne 2S 2700)
© 950
T1474 see
‘Since {> T, itisa slow closure, then:
Since the point 100 m from the reservoir falls within xo, the pressure
Pe ea it ea ‘expected at this point is less than the maximum pressure (at points beyond
(0) Instantaneous partial closure: 475m). The pressure at this ms point can be computed
pny ratio and proportion,
" Pespcdo
40-01-22

"1 second <T (rapid closure)


Waterhammer pressureat points beyond x: Problem9-36
rears ‘The bend shown isin a horizontal plane. A Bid weighing 8615 kN/mé
tne bend wile vocof38 m/sanda
Maximumpressure ps ~100(950)(0.442)
‘Maximum pressure,p,= 419900Pa =4199kPa felon om steerer erepressureof0 ee
La. Neglecting
‘Ans: 104KN,
fe

st.

TO, 1200 ©
Properties of Fluids & Conversion Fa
9-37
inteneatantaarr Appendix
San
a Properties ofFluids
Problem9+ 38 & Conversion Factors
‘Determine the magnitude and directionofthe resultant forceexerted on the
double nozzleshown, The axesofthepipes and both nozzles le in ahorizontal
plane. Bothnozzleetshave water velocity of12 m/s.
‘Ans: 28 N upwardotheright
‘8 64°withthe horizontal

1997
104 1.925 1.37
12-1917 ia
a 140 [1.908 5.
176 1.88610. TAL
90 0: iss [tars |
0295 212 1.859 [0.591
Suggested curvefitsforwater in ange 0.5< 100°C:
(kg/m?) = 1000- 0.0178|T-4°C}'7 £02% (Tin °C)

In oe ~-1.708 - 5.302 + 7.0032


He
wherez= 223 (rink)
Properties ofFluids & Conversion
PapertGren gistand 20
a i oe
ten Sam eee
Sear a
e os saa
Table A=2:Veiety 20 gst fAat at ‘ Sa
i oat
a
nat5 alia [ee¢ lake ee tee?
Sea ow~
saint idm ot coer saa [oes
Tap ast [ose a [aie ae et n
179 [at] 433, 251_|358_[ 1 Ta Biri 16
Fo zu 3 f a 37210" 285.1
195 479) 212_| 4.06. S58 aaa
2a7_|_230 o_| 454, = Tat
238
See [265 182 sie)
2 ia
275 408,
BAe 13 375 Z =ra = FACES
# eee 07 10" “728 2k
355 [7 089 [74
“ascbeaine nS fo se: ‘TableA=4: Properties ofConemenGases at 1am and 20°C
=e
eRe
Power law:
| Sutherland law: #2
is Ut) (eg) ORs sense
with Ts278°K, po 1.71% 108 Pa,gd Tin kelvins.
a
0.
are} 00"
* Giticalpont
APPENDIX,
j ofFluids & Convesion Factors 1 Properties ofFluids ConversionFactors
(Continuation) 4 ConversionFactors from BG toIUnits (Cos A
‘Dery, Soundpeed j Tomerron To mater |
pat
06
"ome
96
aaa Hi x
Peer
sss
pies
oo33—[ [978
29a vay We
mn mi
a oe
oor!
ost}
ret on
307 EE
ves Be e780
iSoro 36 veune #i
wus) mt
:™ ton
Sis zy
a fhidounce (vs) mt 29574x 104
Conversion Factors from 86 to I Units
nek Sy
wht
eA
we
iSos0 its
_ = Tezt Table - A= Or Covers Fecor

oe ™Mi } ee
MOR
Taesn=16009 Te = y= 231607 = 0.007054?
saga?
t310— oneree 11S ems
Energy
batt
Reet
gin 4 1892 13 us, once = 29574 19°
E }* Lyd = 3 t= 0.914 m
1 nso (0) = 1.0 10%m US. quatt () © 9.4695 4:10 ee

Foe & n sae Ti i


= 3
ot N x io
9.8067 Talay = 32.174ibm = 14.594
Toms oaeg ne TAT 6.082908
Pic ieee zraTee 10
5 ° 25a:
m(ganas)
cai)6
im {m3510"
18570
1 sutton21000
Tiare
= 2000 =907185
SWS ace et)
= 500= 10,000
=
He sin ia
Tastow in
sa ca
Ta 5350»
Taso j| Tate
Te" O30 :
bs als 459910 + Hin?
Siminiinecaden= 048704 mys [ince oaae
Foner ns
iS Wwwv 13588 as0
asm | amn= tes7ene y
mae wi aA ‘701 Tosimivs
cane
= ms bene
013 eae Taran Coane alumflow
Seca en
ear fa
Mae ise
57510 | tbe rae Goes La PRay
oT eee as
Sc
Speci eat
Sostensa SS
at a
Nn Lab
ies of Fluids & Conversion Factors INDEX
‘Conversion Factors (Continuation) A Cohesion, 7 *
“Pressure Force) Acoustical velocity,9 Colebrook equation, 369 iy
“Tibi= 47.88 p2 TR = 4HOD2 N= 1608 Adhesion, 7 Compressbility, 8
‘in? = 144 xh? = 6895 Pa 1 kat 2.2046bt = 9.806656 ‘Conservation of mass, principle of, 242
‘atm = 2116.2bt! = 14696 mafia? 11 U, (short) ton = 2000 br ‘Adiabatic condition, 10 Continuity equation, 242
= 101,325 Pa Lome = 1.0% 10° N ‘exponent, 10, ‘Continuous flow, 242
1g (a2)= 3575 Pa 1 ounce (avis) (on) = 0.27801 N Absolute pressure, 29
bare 10 «10° Pa Adiabaticcondition, 10 ‘Contraction ofthe jet, 284
Alternatestagesofflow,491 ‘Convergingtubes, 292
‘Arch dam, 82 Crest, 297
Power. Archimedes’principle, 8 Critical depth, 492
Tiers 1355623 Thp = S50 bis = 745.7 W Atmospheric pressure,2. Critical slope, 494
1 Btu = 152 cal = 1055.0563 Ribs = 1.3558 W Critical velocity, 375, 495
= 78.17 Reet . Culvert, 253
1 Kwa nou (i) = 3.6 « 10°)
B
[raassisene
7 Tart = sistant
157.08 Wi Barometer, aneroid, 30 D
4 om 160185 Bazin formula, 301, 483 Dam, 80
1g 1000kg Bends, 390 analysis of, 4
Kinematieviscosity Bends channel, 500 types of, 81
TR= 0.000025606 mis Bernoull’s energy theorem,246 -Weisbach formula, 381
THORS SH = Ta = OOOO Boiling, 10 int
Borda's mouthpiece, 298 Differential manometer, 4
eae fees “Absolute Temperature sealo readings Bourdon, 29 Discharge,241
OMe + 32 Boyle's law, 9 Displaced fui, 88
Te (PNTGPM = Te4 273.16 ‘Bulk modulus ofelasticity, 9 Diverging tubes 292
or |[-—"Thermaeonduetvig= Buoyancy,88 Drag, curve, 297
Drop-down
TAAug") = 046723 N-VGegH) [BRTRR) = 1.7307 WICK) Burstingforce, 96
u/iboR)w 4186.8,/(65) Buttress dam, 83

c
Canal lock,81
Capillary,
Centr, ofbuoyancy, 99
ofpressure, 73
Charleslaw,9
(Chézy formula, 482
coeficientof482
Cipolltt weit,904
Coeficient,ofcompressibility,8
ofcontraction,277
offriction, 86
of velocity, 277

You might also like